Sunteți pe pagina 1din 187

Universit

e Paris-Sud Orsay
L2 S3 & S4 PMCP

ANNALES DE PHYSIQUE
Phys202r & Phys206r
Ondes, Corde vibrante, Acoustique,

Electromagn
etisme, Optique ondulatoire

Partiels et examens
2002-2006
ANNALES DE PHYSIQUE
Phys202r & Phys206r
Ondes, Corde vibrante, Acoustique,

Electromagn
etisme, Optique ondulatoire

2002-2006

Mise `a jour de ce fascicule sur le site


www.deugs3smr.u-psud.fr/

Premi`ere version : 26 septembre 2005


Cette version : 28 juillet 2006

Jean-Jacques LABARTHE
Laboratoire Aime-Cotton
www.lac.u-psud.fr
Bat 505 CNRS II
91405 ORSAY Cedex
Tel. : 01 69 35 20 49
Fax : 01 69 35 21 00

Ce fascicule regroupe les sujets de partiels et dexamens donnes en S3 Physique de la preparation au


magist`ere et aux concours de physique ( prepa ENSI , DEUG S3 SMR , L2 S3 PMCP )
pendant les quatre annees universitaires 2002 `a 2006. Ils correspondent, dans le nouveau L2 PMCP, au
Phys202r du S3 et au Phys206r du S4.
Les sujets des 1ers partiels portent sur les ondes, la corde vibrante et lacoustique.
Les sujets des 2es partiels portent sur les ondes (effet Doppler) et lelectromagnetisme.
Les sujets dexamen comportent en plus de loptique ondulatoire.
Les textes ont ete rediges par Jean-Jacques Labarthe avec la collaboration de Jacqueline Ridard et Marta
De Frutos.
2 `
TABLE DES MATIERES

Table des mati`


eres

1. Premier partiel 2002-2003 5


A. Mesures acoustiques dans lair . . . . . . . . . . . . . . . . . . . . . . . . . . . . . . . . 5
B. Corde fixee aux deux extremites . . . . . . . . . . . . . . . . . . . . . . . . . . . . . . . 6
C. Corde inhomog`ene . . . . . . . . . . . . . . . . . . . . . . . . . . . . . . . . . . . . . . 7
Corrige . . . . . . . . . . . . . . . . . . . . . . . . . . . . . . . . . . . . . . . . . . . . . . 9
Complement . . . . . . . . . . . . . . . . . . . . . . . . . . . . . . . . . . . . . . . . . . . . 14

2. Deuxi`
eme partiel 2002-2003 16
A. Questions de cours . . . . . . . . . . . . . . . . . . . . . . . . . . . . . . . . . . . . . . 16
B. Etude dun fil cylindrique . . . . . . . . . . . . . . . . . . . . . . . . . . . . . . . . . . 16
C. Onde plane electromagnetique . . . . . . . . . . . . . . . . . . . . . . . . . . . . . . . . 19
C+. Questions supplementaires . . . . . . . . . . . . . . . . . . . . . . . . . . . . . . . . . 20
Corrige . . . . . . . . . . . . . . . . . . . . . . . . . . . . . . . . . . . . . . . . . . . . . . 21

3. Examen 2002-2003 29
A. Communication avec un sous-marin . . . . . . . . . . . . . . . . . . . . . . . . . . . . . 29
B. Pouvoir separateur dune lunette astronomique . . . . . . . . . . . . . . . . . . . . . . 33
Corrige . . . . . . . . . . . . . . . . . . . . . . . . . . . . . . . . . . . . . . . . . . . . . . 35

4. Examen (2e session) 2002-2003 40


A. Ondes . . . . . . . . . . . . . . . . . . . . . . . . . . . . . . . . . . . . . . . . . . . . . 40
B. Propagation dondes acoustiques . . . . . . . . . . . . . . . . . . . . . . . . . . . . . . . 40
Partie non posee `
a lexamen . . . . . . . . . . . . . . . . . . . . . . . . . . . . . . . 43
C. Induction . . . . . . . . . . . . . . . . . . . . . . . . . . . . . . . . . . . . . . . . . . . . 45
Corrige . . . . . . . . . . . . . . . . . . . . . . . . . . . . . . . . . . . . . . . . . . . . . . 46

5. Premier partiel 2003-2004 53


A. Onde progressive . . . . . . . . . . . . . . . . . . . . . . . . . . . . . . . . . . . . . . . 53
B. Ondes progressives en sens opposes . . . . . . . . . . . . . . . . . . . . . . . . . . . . . 53
C. Reconnatre les ondes . . . . . . . . . . . . . . . . . . . . . . . . . . . . . . . . . . . . . 54
D. Battements . . . . . . . . . . . . . . . . . . . . . . . . . . . . . . . . . . . . . . . . . . 54
E. Corde sol dun violon . . . . . . . . . . . . . . . . . . . . . . . . . . . . . . . . . . . . . 54
F. Vibration forcee dune corde . . . . . . . . . . . . . . . . . . . . . . . . . . . . . . . . . 55
`
TABLE DES MATIERES 3

Corrige . . . . . . . . . . . . . . . . . . . . . . . . . . . . . . . . . . . . . . . . . . . . . . 58

6. Deuxi`
eme partiel 2003-2004 64
A. Effet Doppler dun sonar . . . . . . . . . . . . . . . . . . . . . . . . . . . . . . . . . . . 64
B. Reflexion et transmission . . . . . . . . . . . . . . . . . . . . . . . . . . . . . . . . . . . 65
C. Condensateur elastique . . . . . . . . . . . . . . . . . . . . . . . . . . . . . . . . . . . . 66
D. Induction magnetique dans un cadre en mouvement . . . . . . . . . . . . . . . . . . . . 67
Corrige . . . . . . . . . . . . . . . . . . . . . . . . . . . . . . . . . . . . . . . . . . . . . . 69

7. Examen 2003-2004 74
A. Questions de cours . . . . . . . . . . . . . . . . . . . . . . . . . . . . . . . . . . . . . . 74
B. Fentes dYoung . . . . . . . . . . . . . . . . . . . . . . . . . . . . . . . . . . . . . . . . 75
C. Guide dondes hyperfrequences . . . . . . . . . . . . . . . . . . . . . . . . . . . . . . . 76
Corrige . . . . . . . . . . . . . . . . . . . . . . . . . . . . . . . . . . . . . . . . . . . . . . 80

8. Examen (2e session) 2003-2004 89


A. Onde harmonique stationnaire dune corde . . . . . . . . . . . . . . . . . . . . . . . . . 89
B. Induction . . . . . . . . . . . . . . . . . . . . . . . . . . . . . . . . . . . . . . . . . . . . 90
C. Interferom`etre de Michelson . . . . . . . . . . . . . . . . . . . . . . . . . . . . . . . . . 91
Corrige . . . . . . . . . . . . . . . . . . . . . . . . . . . . . . . . . . . . . . . . . . . . . . 92

9. Premier partiel 2004-2005 95


A. Onde triangulaire . . . . . . . . . . . . . . . . . . . . . . . . . . . . . . . . . . . . . . . 95
B. Ondes dune membrane . . . . . . . . . . . . . . . . . . . . . . . . . . . . . . . . . . . . 97
Corrige . . . . . . . . . . . . . . . . . . . . . . . . . . . . . . . . . . . . . . . . . . . . . . 99

10. Deuxi`
eme partiel 2004-2005 105
A. Questions de cours . . . . . . . . . . . . . . . . . . . . . . . . . . . . . . . . . . . . . . 105
B. Effet Doppler . . . . . . . . . . . . . . . . . . . . . . . . . . . . . . . . . . . . . . . . . 105

C. Etude dun condensateur . . . . . . . . . . . . . . . . . . . . . . . . . . . . . . . . . . . 106
Corrige . . . . . . . . . . . . . . . . . . . . . . . . . . . . . . . . . . . . . . . . . . . . . . 109

11. Examen 2004-2005 117


A. Ligne coaxiale . . . . . . . . . . . . . . . . . . . . . . . . . . . . . . . . . . . . . . . . . 117
B. Franges dYoung et observation dune etoile double . . . . . . . . . . . . . . . . . . . . 121
Corrige . . . . . . . . . . . . . . . . . . . . . . . . . . . . . . . . . . . . . . . . . . . . . . 123

12. Examen (2e session) 2004-2005 131


A. Induction . . . . . . . . . . . . . . . . . . . . . . . . . . . . . . . . . . . . . . . . . . . 131
B. Reflexion dune onde plane electromagnetique . . . . . . . . . . . . . . . . . . . . . . . 132
C. Pouvoir separateur dune lunette astronomique . . . . . . . . . . . . . . . . . . . . . . 133
Corrige . . . . . . . . . . . . . . . . . . . . . . . . . . . . . . . . . . . . . . . . . . . . . . 136

13. Premier partiel 2005-2006 141


4 `
TABLE DES MATIERES

A. Ondes de types S et A . . . . . . . . . . . . . . . . . . . . . . . . . . . . . . . . . . . . 141


B. Onde harmonique stationnaire dune corde . . . . . . . . . . . . . . . . . . . . . . . . . 143
Corrige . . . . . . . . . . . . . . . . . . . . . . . . . . . . . . . . . . . . . . . . . . . . . . 145

14. Deuxi`
eme partiel 2005-2006 150
A. Magnetostatique : bobinage torodal, inductance . . . . . . . . . . . . . . . . . . . . . . 150
B. Ondes electromagnetiques . . . . . . . . . . . . . . . . . . . . . . . . . . . . . . . . . . 152
C. Blindage electromagnetique . . . . . . . . . . . . . . . . . . . . . . . . . . . . . . . . . 154
Corrige . . . . . . . . . . . . . . . . . . . . . . . . . . . . . . . . . . . . . . . . . . . . . . 156

15. Examen 2005-2006 163


A. Ondes electromagnetiques dans le milieu interstellaire et distance dun pulsar . . . . . 163
B. Propagation guidee : ligne `
a rubans . . . . . . . . . . . . . . . . . . . . . . . . . . . . . 165
C. Dispositif interferentiel des trous dYoung . . . . . . . . . . . . . . . . . . . . . . . . . 167
Corrige . . . . . . . . . . . . . . . . . . . . . . . . . . . . . . . . . . . . . . . . . . . . . . 170

16. Examen (2e session) 2005-2006 178


A. Corde fixee aux deux extremites . . . . . . . . . . . . . . . . . . . . . . . . . . . . . . . 178
B. Reflexion dune onde plane electromagnetique . . . . . . . . . . . . . . . . . . . . . . . 179
C. Induction . . . . . . . . . . . . . . . . . . . . . . . . . . . . . . . . . . . . . . . . . . . . 180
D. Appareil photographique . . . . . . . . . . . . . . . . . . . . . . . . . . . . . . . . . . . 180
Corrige . . . . . . . . . . . . . . . . . . . . . . . . . . . . . . . . . . . . . . . . . . . . . . 182
5

1. Premier partiel 2002-2003


Enonc
e
29 octobre 2002
duree : 2 heures 30
sans documents, calculatrices autorisees

Les parties A, B et C sont independantes, mais il vaut mieux traiter le probl`eme C apr`es le probl`eme
B. Bar`eme indicatif : A=5, B=7, C=8.

A. Mesures acoustiques dans lair

AA
AA
haut-parleur microphone

AA O x0 x

Fig. 1: Mesures acoustiques dans lair.

1) Un haut-parleur alimente par une tension sinusodale de frequence = (1000 10) Hz cree dans
lair une onde plane progressive le long de laxe Ox. La surpression au point x et `a linstant t est de la
forme
p(x, t) = p0 sin(t kx + 0 ). (1)

Expliquer le sens de lexpression onde plane progressive.

2) Un microphone place au point x sur laxe gen`ere une tension u(t) = u0 sin(t ) proportionnelle
a` la surpression p au point x. Un dispositif mesure le dephasage . Des mesures pour diverses positions
du microphone montrent que le dephasage depend de x suivant la loi

u a = (18,4 0,2) rad m1 .


= ax + b o` (2)

En deduire la valeur numerique de la vitesse du son c c.

3) On deplace le microphone le long de Ox `a la vitesse (algebrique) constante v. A linstant t = 0, le


microphone se trouve en x0 .

3.a) Quelle est la position x du microphone `a linstant t?


6 1. PREMIER PARTIEL 2002-2003

3.b) Montrer que dans cette experience le microphone gen`ere une tension

ud (t) = u0 sin( t ) (3)

de frequence differente de et exprimer en fonction de , v et c.

3.c) On mesure
= (1,20 0,01) Hz. (4)
Exprimer la vitesse v du microphone en fonction des grandeurs mesurees. Calculer numeriquement vv.
Preciser sur un dessin le sens de deplacement du microphone.

4) En realite, londe generee par un haut parleur nest pas une onde plane. Decrire londe reellement
creee ; son amplitude est-elle constante?

B. Corde fix
ee aux deux extr
emit
es
On consid`ere une corde de longueur L = 63 cm et de masse lineique = 2 102 kg m1 . La corde est
fixee en ses extremites (les points x = 0 et x = L de laxe Ox) et tendue avec la tension T0 . La vitesse
des ondes transverses de la corde est c = 500 m s1 .

1.a) Determiner la tension T0 en fonction de et c. Donner les dimensions de T0 , et c et verifier


lhomogeneite de la formule obtenue.

b) Calculer la valeur numerique de la tension T0 .

c) Ecrire lequation donde de dAlembert verifiee par le deplacement s(x, t).

2) On consid`ere londe stationnaire


 x 
s1 (x, t) = A sin cos(1 t). (5)
L
a) Montrer que lequation (5) est solution de lequation donde de la question 1.c) pour une valeur de
1 que lon determinera litteralement en fonction de L et c puis numeriquement. Determiner litteralement
et numeriquement la frequence 1 et la periode T1 de cette onde.

b) Dessiner laspect de la corde vibrante.

c) Expliquer le qualificatif stationnaire donne `a cette onde.

3) On consid`ere londe stationnaire


 
2x
s2 (x, t) = A sin sin(2 t). (6)
L

Montrer que lequation (6) est egalement solution de lequation donde de la question 1.c) pour une valeur
de 2 que lon determinera litteralement et numeriquement. Determiner litteralement et numeriquement
la frequence 2 et la periode T2 de cette onde.

4) On consid`ere londe
     
x 2x
s(x, t) = A sin cos(1 t) + sin sin(2 t) (7)
L L
7

o`
u 1 et 2 ont les valeurs determinees ci-dessus.

a) Montrer que le signal (7) est solution de lequation donde de la question 1.c).

b) Cette onde est-elle stationnaire? Cette onde est-elle periodique?

c) On pose
s
f (x) = s(x, t = 0) et g(x) =
(x, t = 0). (8)
t
Representer graphiquement f (x) et g(x) en fonction de x (0 x L).

d) Expliquer comment un experimentateur peut creer londe (7) en preparant la corde `a linstant t = 0.

5) On desire augmenter de 5 % la frequence du mode de vibration de la corde considere dans la question


2).

a) Montrer que cest possible en modifiant la longueur de la corde, sans modifier la tension. Calculer
la nouvelle longueur de la corde.

b) Donner une autre facon dobtenir la meme modification de la frequence sans modifier ni la longueur
de la corde, ni sa masse lineique. Calculer la nouvelle valeur de la grandeur modifiee.

C. Corde inhomog`
ene
On designe par corde B la corde vibrante du probl`eme B : sa longueur est L = 63 cm ; sa masse lineique
= 2 102 kg m1 ; elle est fixee en x = 0 et x = L de laxe Ox et tendue avec la tension T0 ; la vitesse
des ondes dans cette corde est c = 500 m s1 .
On designe par corde B une seconde corde de longueur L, homog`ene, de masse lineique = o`
u
= 10, tendue avec la meme tension T0 et fixee en ses extremites.

1.a) Quelles sont les frequences propres de vibration, rangees dans lordre croissant, 1 , 2 , 3 , . . . de la
corde B ? Calculer les valeurs numeriques de 1 , 2 et 3 . Vous pouvez utiliser les resultats du probl`eme
B pour repondre `a cette question.

b) Determiner la vitesse c des ondes dans la corde B . Quelles sont les frequences propres de vibration,
rangees dans lordre croissant, 1 , 2 , 3 , . . . de la corde B ? Calculer les valeurs numeriques de 1 , 2
et 3 .

ene C
Corde inhomog`

On utilise maintenant une nouvelle corde C , toujours de longueur L = 63 cm, tendue avec la meme
tension T0 et fixee en ses extremites (les points x = 0 et x = L de laxe Ox). La corde est constituee
L
de deux demi-cordes G et D de longueurs L/2 attachees bout `a bout en x = . La demi-corde G
2
L
(0 x ) a la meme masse lineique que la corde B de sorte que la vitesse des ondes dans cette
2
L
demi-corde est toujours c = 500 m s1 . La demi-corde D ( x L) a une masse lineique =
2
a celle de la corde B .
identique `
a la question 9 on ne consid`ere plus que la corde C et on se propose de
Dans la suite du probl`eme jusqu`
determiner ses modes propres de vibration (ondes stationnaires).

L
2) Ecrire lequation donde verifiee par le deplacement s(x, t) (on distinguera deux cas: (a) 0 x
2
L
et (b) x L).
2
8 1. PREMIER PARTIEL 2002-2003

3) On consid`ere une onde de la forme




sg (x, t) = G sin(kx + g ) sin(t + ) L
si 0 x
s(x, t) = 2 (9)
L
sd (x, t) = D sin(k x + d ) sin(t + )
si xL
2
o`
u G et D sont des constantes reelles positives ou negatives.
Quelle est la relation entre k, et c? Quelle est la relation entre k , et c ? En deduire lexpression de
k en fonction de k et .
Justifier, sans aucun calcul, que la phase `
a lorigine est la meme dans sg (x, t) et sd (x, t).

4) Ecrire les conditions aux limites en x = 0 et x = L. En deduire g et d .


Il y a plusieurs solutions pour g et d . Expliquer pourquoi on peut se limiter `a choisir une quelconque
de ces solutions.

5) Justifier pourquoi londe (9) doit satisfaire aux conditions


       
L L sg L sd L
sg , t = sd ,t , ,t = ,t (10)
2 2 x 2 x 2
pour tout temps t.

6) En utilisant les equations (10), montrer que k doit satisfaire `a une equation qui secrit seulement
en termes de k, L et .

7) On admettra que les solutions strictement positives de lequation



tg 10
tg() = (11)
10
sont, rangees dans lordre croissant, 1 = 0,6269, 2 = 1,498, 3 = 2,382, . . . Calculer les valeurs
numeriques des trois frequences de vibration les plus basses de la corde 1 , 2 et 3 .

8) Dans le mode de frequence 3 , calculer les valeurs numeriques de k et k . Comparer les longueurs
dondes associees `
a ces valeurs avec la longueur L de la corde. Dessiner laspect de la corde vibrante `a
un instant donne pour ce mode de vibration.

9) Comparer les frequences de vibration n , n et n de meme rang n des cordes B, C et B pour un


entier positif n quelconque.
9

Corrig
e

A. Mesures acoustiques dans lair

1) Onde plane : la surpression p(x, y, z, t) = p(x, t) est la meme sur chaque plan x = Cte (la surpres-
sion ne depend ni de y ni de z). Ces plans x = Cte sont les surfaces donde (plans donde).
Onde progressive : londe se propage `a vitesse constante sans attenuation ni deformation le long de
laxe Ox dans le sens de x croissant. En effet, la surpression est de la forme p(x,t) = F (t x/c) avec
F (u) = p0 sin(u + 0 ) et c = /k.

2 2
2) Le dephasage depend de x par = kx 0 = x 0 . On a donc mesure a = . La valeur
c c
de b, qui nest pas utilisee dans la suite, est b = 0 . Do`
u

2
c= = 341,5 m s1 . (12)
a
c a
Lincertitude relative sur c est = + u c = 7,1 m s1 . En conclusion, la vitesse
= 0,02. Do`
c a
du son mesuree est
c = (341,5 7,1) m s1 (13)

a linstant t est x = x0 + vt .
3.a) La position du microphone `

3.b) Le microphone gen`ere une tension ud (t) = u0 sin(t kx + 0 )= u0 sin(t kx0 kvt + 0 ) =
v
u0 sin([ kv]t kx0 + 0 ) = u0 sin( t ) avec = kv = 1 et = kx0 0 . Il voit
c
donc la frequence
 v
= 1 (14)
c

differente de (effet Doppler).

v av
3.c) Les equation (14) et (12) donnent = = . On a donc v = 2 =

c 2 a
v ( ) a
0,410 m s1 . Lincertitude relative sur v est = + u v = 0,0079 m s1 .
= 0,019. Do`
v a
En conclusion, la vitesse du microphone est
AA
AA
haut-parleur microphone

AA
v<0
v = (41,0 0,8) cm s1 . x
(15)

Le signe indique que le microphone se deplace vers le haut-parleur.

4) Londe emise est plutot une onde spherique inhomog`ene (du moins `a des distances du haut-parleur
grandes devant ses dimensions). Lamplitude constante p0 doit etre remplacee par une amplitude qui
decrot inversement avec la distance r du point dobservation au haut-parleur. Si on designe par h
labscisse du haut-parleur et en se limitant `a observer londe sur laxe Ox en x > h, une expression plus
realiste de la surpression est
A
p(x, t) = sin(t kx + 0 ).
xh
10 1. PREMIER PARTIEL 2002-2003

B. Corde fix
ee aux deux extr
emit
es

1.a) La formule de la corde vibrante donne T0 = c2 . Les dimensions sont [T0 ] = M LT 2, [] = M L1


et [c] = LT 1 ; T0 = c2 est bien homog`ene.

1 2s 2s
1.b) T0 = 5000 N . 1.c) 2 2
2 =0.
c t x

2 s1 2 2 s1  2
2.a) On calcule = 1 s 1 et = s1 .
t2 x2 L
  
2  1 2 c
Lequation donde 1.c donne s1 = 0 do`
u on tire 1 = = 2493 rad s1 .
L c L
1 c 1 2L
La frequence est 1 = = = 397 Hz et la periode T1 = = = 2,52 ms .
2 2L 1 c

2.b) La figure de gauche represente la forme de la corde aux instants tn = nT1 /16 pour n entier.
La periode T1 = 2,52 ms est tr`es petite devant le temps de persistance des images retiniennes, la corde
apparat comme sur la figure de droite.

0 L 0 L

2.c) Stationnaire : tous les points de la corde vibrent en phase ou en opposition de phase (comparer
` onde progressive). Mathematiquement, s1 (x,t) est factorise en un produit dune fonction de t et dune
a
fonction de x.

2c
a la question 2. On obtient 2 = 21 =
3) La resolution est similaire ` =
L
c T1 L
4886 rad s1 , 2 = 21 = = 794 Hz et T2 = = = 1,26 ms.
L 2 c

4.a) s = s1 +s2 est la somme (superposition) de deux solutions de lequation donde. Lequation donde
etant lineaire, cest aussi une solution.

4.b) Londe s nest pas stationnaire. Londe s est periodique de periode temporelle T1 : s(x, t + T1 ) =
s(x, t).
11

4.c)
 x  f (x) g(x)
f (x) = A sin
L
represente la forme de la cor-
de `a linstant t = 0 et 0 L 0 L
 
2x
g(x) = A2 sin
L

est la vitesse du point ma-


teriel dabscisse x `
a linstant
t = 0.
4.d) Lexperimentateur peut mettre la corde en mouvement en deplacant transversalement chaque
point dabscisse x [0,L] de la corde de f (x) et en lui communiquant la vitesse transverse g(x) `a
linstant t = 0. Ces conditions initiales determinent compl`etement le mouvement de la corde, et, comme
s(x, t) y satisfait, lexperimentateur a ainsi cree londe s(x, t).
s
c 1 T0
5.a) La frequence est 1 = = . On peut laugmenter de 5 % en diminuant la longueur L
2L 2L
de 5 % `a tension constante. La nouvelle longueur est L = 60 cm.

5.b) On peut augmenter la frequence de 5 % sans modifier la longueur de la corde en augmentant la


tension de 10 %. La nouvelle tension est T0 = 5500 N.

C. Corde inhomog`
ene

1.a) Le mode de vibration de la corde B considere en B 2.a) est le mode fondamental de frequence 1 =
c
. Les frequences propres de la corde B sont les harmoniques n = n1 de la frequence fondamentale
2L
1 : 1 = 397 Hz , 2 = 794 Hz , 3 = 1190 Hz , . . .

1.b) La vitesse c des ondes dans la corde B est


s s
T 0 1 T0 c
c = = = = 158 m s1 .

c 1
La frequence du mode fondamental est 1 = = = 125 Hz. Les frequences propres de la corde
2L
B sont les harmoniques n = n1 de la frequence fondamentale 1 : 1 = 125 Hz , 2 = 251 Hz ,
3 = 376 Hz , . . .

1 2s 2s L 1 2s 2s
2) Lequation donde verifiee par s(x, t) est 2 2 = 0 pour 0 x et =0
c t x2 2 c2 t2 x2
L
pour x L.
2
L
3) Londe sg est solution de lequation donde pour 0 x lorsque = kc . Londe sd est solution
2
L kc
de lequation donde pour x L lorsque = k c . On a donc k = soit k = k .

2 c
On cherche les modes propres de la corde sous la forme dondes stationnaires, cest-`a-dire sous la forme
dune onde s(x, t) = f (x)g(t) produit dune fonction f (x) qui ne depend que de x et dune fonction g(t)
12 1. PREMIER PARTIEL 2002-2003

qui ne depend que du temps t. Le facteur g(t) = sin(t + ) doit etre le meme pour les deux moities de
la corde. La phase doit donc aussi etre la meme pour les deux moities de la corde.
Nota : la fonction f (x) est

G sin(kx + g ) si 0 x L

f (x) = 2
L
D sin(k x + d ) si
xL
2

4) La corde est fixee en x = 0 et x = L. Les conditions aux limites en x = 0 et x = L sont donc


sg (0, t) = 0 t et sd (L, t) = 0 t. On doit donc avoir sin(g ) = 0 et sin(k L + d ) = 0. On peut prendre
g = 0 et d = k L soit


sg (x, t) = G sin(kx) sin(t + ) L
si 0 x
s(x, t) = 2 (16)
 L
sd (x, t) = D sin k (x L) sin(t + ) si
x L.
2
Les autres solutions de sin(g ) = 0 et sin(k L + d ) = 0 donnent la meme expression de s(x, t)
(eventuellement en modifiant les signes de G et D). Par exemple, g = donne sg (x, t) = G sin(kx +
) sin(t + ) = G sin(kx) sin(t + ) qui est de la forme (16) apr`es le changement de G en G.
   
L L L
5) La condition sg , t = sd , t qui exprime que le deplacement est continu en x = est
2 2 2
justifiee par le fait que la corde reste dune pi`ece.
   
sg L sd L s L
La condition ,t = , t exprime la continuite de la derivee en x = . Il y a
x 2 x 2 x 2
L
raccordement en x = des deux morceaux sg et sd . On peut justifier cette condition de la facon
2
L
suivante. Si ces deux morceaux formaient un point anguleux en x = , la resultante des forces de
2
L L
tension appliquees au tr`es petit troncon de corde < x < + aurait une composante transverse
2 2
non nulle dans la limite 0. Ce troncon de corde ayant une masse qui tend vers 0 lorsque 0, il
serait soumis `a une acceleration infinie, ce qui est impossible.

L
6) Ecrivons que (16) verifie les conditions aux limites en x = :
2

 
kL k L
G sin = D sin (17)
2 2
   
kL kL
Gk cos = Dk cos (18)
2 2
   
1 kL 1 k L
Le rapport de ces equations donne tg = tg soit
k 2 k 2
   
kL 1 kL
tg = tg . (19)
2 2

kL
7) Posons = . Pour = 10, lequation (19) secrit
2

tg 10
tg() = . (20)
10
13

kc c
La frequence de vibration correspondant `a une solution de cette equation est = = . Les
2 L
frequences de vibration n (n = 1, 2, . . . ) de la corde correspondent aux racines strictement positives de
n c
lequation (20) rangees dans lordre croissant : n = Les trois frequences de vibration les plus basses
L
de la corde sont donc 1 = 158 Hz , 2 = 378 Hz , et 3 = 602 Hz . Les frequences propres de la corde
C ne forment pas une serie harmonique.
2n
8) Lequation (18) determine D en fonction de G pour le mode de vibration n. Posons kn = =
c
2n sin (kn L/2) sin(n )
. On a D = G = G . Londe du mode de vibration n est donc de la
L sin ( kn L/2) sin( n )
forme
L
A sin( n ) sin(kn x) sin(2n t + ) si 0 x
s(x, t) = 2 (21)
 L
A sin(n ) sin kn (x L) sin(2n t + ) si xL
2
o`
u A et sont arbitraires.

mode n kn = 2n /L kn = kn n = c/n n = c /n
1 1,99 m1 6,29 m1 3,16 m= 5,0 L 0,99 m= 1,6 L
2 4,76 m1 15,0 m1 1,32 m= 2,1 L 0,41 m= 0,66 L
3 7,56 m1 23,9 m1 0,83 m= 1,3 L 0,26 m= 0,41 L

Le mode n comporte n + 1 nuds de vibration. On peut observer les longueurs dondes differentes
n = c/n et n = c /n = / 10 qui sont respectivement les periodes spatiales de sg et sd dans les
moities gauche et droite.

s s s

0 L 0 L 0 L

mode 1 mode 2 mode 3

9) On observe, en utilisant les valeurs calculees, que

n < n < n (22)

pour n = 1, 2, 3. Les frequences propres s


dune corde homog`ene de masse lineique de longueur L tendue
c n T0
avec la tension T0 sont n = n = . Pour n, L et T0 fixes, ces frequences decroissent quand
2L 2L
augmente. On a donc, en correspondance avec > ,

n < n (23)

pour tout n, et on sattend `


a ce que les inegalites (22) soient vraies pour tout n puisque la masse lineique
de la corde inhomog`ene est comprise entre et .
14 1. PREMIER PARTIEL 2002-2003

Compl
ement

Dans ce complement, nous donnons une demonstration des inegalites (22) pour tout n. Presentons tout
dabord une determination graphique des frequences n . Dans ce complement, x ne designe plus labscisse
des points de la corde, mais une frequence.
Les frequences propres n (n = 1, 2, y
. . . ) de la corde C sont les abscisses y = f1 (x)

x > 0 des intersections des courbes


Lx
y = f1 (x) = tg (24)
c
1 Lx
y = f2 (x) = tg (. 25)
c
1 3 4 2
Lintersection pour 2 est en dehors de 1 1 2 3 2 4 5 6 7 5 x
la figure.
Les frequences propres i (i = 1, 2,
. . . ) de la corde B correspondent ` a
f1 (2m ) = 0 et f1 (2m1 ) = (m =
1, 2, . . . ). Les frequences propres j
(j = 1, 2, . . . ) de la corde B corres- y = f2 (x)

pondent `a f2 (2m ) = 0 et f2 (2m1 )=
(m = 1, 2, . . . ).

y
La frequence propre n de la corde 1

C se trouve dans les intervalles de


y = g1 (x)
frequences i , i+1

et [j , j+1 [ x3 x4
0 x1 x2 x
(on pose 0 = 0 = 0) avec
y = g2 (x)
   

i = n et j = n (26) 1
1 1

o`
u x designe la partie enti`ere de
x. Nous pouvons pour determiner
ces intervalles remplacer les fonc-
tions f1 (x) et f2 (x) par des fonc-
tions en dents de scie variant entre
1 et +1, respectivement g1 (x) et
g2 (x), en faisant correspondre ` a
une branche de la fonction tg un
segment de droite.

Nous definissons ces fonctions en dents de scie par :


x 2m x
g1 (x) = = 2m si x [2m1 , 2m+1 [ (27)
1 1

x 2m 

g2 (x) = si x 2m1 , 2m+1 (28)
1

pour m = 1, 2, . . . Le nombre dintersections des courbes y = f1 (x) et y = f2 (x) dans lintervalle


i , i+1

ou [j , j+1 [ est le meme que le nombre dintersections des courbes y = g1 (x) et y = g2 (x)
dans le meme intervalle. Soit xn (n = 1, 2, . . . ) labscisse de la n-i`eme intersection des courbes y = g1 (x)
et y = g2 (x), pour x > 0, la suite xn etant croissante. Les nombres cherches i et j de (26) sont donc
   
xn xn
i= et j = (29)
1 1
15

(mais xn 6= n ). Labscisse xn (n = 1, 2, . . . ) est aussi labscisse de lintersection des droites dequations


(voir les cas n = 1, 2 et 3 sur la figure)
x x
y= et y= 2n. (30)
1 1
 
1 1
On a donc xn + xn = 2n et
1 1
2 n
xn = (31)
1+ 1
Les valeurs i et j cherchees sont donc
  Tableau des valeurs de i et j.
2 n n i j conclusion
i= = 1.519 . . . n (32)
1+ 1 1 0 1 [1 , 2 [ [0, 1 [
et   2 3 0 2 [3 , 4 [ [0, 1 [
2n 3 4 1 3 [4 , 5 [ [1 , 2 [
j= = 0.4805 . . . n . (33)
1+

Il resulte de (33) que j + 1 n et donc que j+1 n (n crot avec n). Comme par definition de j
n [j , j+1 [ on en deduit n < n . Lequation (32) donne i n ; do` u i n ; avec la definition
  i
de i, n i , i+1

, on obtient n n . Comme est irrationnel, f1 (i ) = tg est different de
2
f2 (i ) qui vaut 0 ou . La frequence i nest donc pas une frequence propre de la corde C . Legalite
i = n est donc exclue et les inegalites (22) sont donc valables pour tout n.
Remarque : comme xn et n appartiennent `a un meme intervalle [i , i [ de longueur 1 , on peut ecrire
avec une erreur dau plus 1
s
2 n n 2 T0
n = (34)
1+ 1 2L 1 +
soit s  2
n T0 1+
n avec c = . (35)
2L c 2
Les frequences de vibration de la corde C ne seloignent pas plus de 1 de la serie harmonique des
frequences de vibration dune corde homog`ene de masse lineique c ( < c < ) de memes longueur L
et tension T0 .
16 `
2. DEUXIEME PARTIEL 2002-2003

2. Deuxi`
eme partiel 2002-2003


Enonc
e
6 decembre 2002
duree : 3 heures
sans documents, calculatrices autorisees

Les parties A, B et C sont independantes. Bar`eme indicatif : A=2, B=13, C=5.

A. Questions de cours

1) Quest-ce quun champ de vecteur conservatif ? Donner sans demonstration des proprietes dun tel
champ de vecteur.
Quest-ce quun champ de vecteur `
a flux conservatif? Donner sans demonstration des proprietes dun tel
champ de vecteur.

2) Quappelle-t-on conducteur parfait ? Expliquer pourquoi les champs dune onde electromagnetique
sont nuls dans un tel conducteur.

3) Quest-ce quun dielectrique lineaire homog`ene isotrope? Quelle est la signification precise des mots
dielectrique, lineaire, homog`ene et isotrope?

B. Etude dun fil cylindrique


Ce probl`eme utilise deux syst`emes de coordonnees. Les notations sont indiquees sur la figure. Le point
M a les coordonnees cylindriques (r, , z) ou cartesiennes (x, y, z). Les tri`edres orthonormes directs
associes sont respectivement (~ur , ~u , ~uz ) et (~ux , ~uy , ~uz ).

I. R
egime stationnaire

~ est
Le fil est parcouru par un courant continu dintensite I = 1 A. On suppose que le champ electrique E
constant et uniforme dans tout lespace (conducteur et vide environnant) et que la densite de courant
~ = E~ est constante et uniforme dans le conducteur.
17

On consid`ere un fil conducteur homog`ene rectiligne cylindrique


daxe Oz, de longueur infinie, de rayon a et de conductivite .
La permeabilite 0 et permittivite 0 du conducteur sont iden-
tiques `a celles du vide. Les valeurs numeriques des constantes z
~uz
sont
~uz ~u
a = 1 mm ; M
= 108 S m1 ; O ~uy ~ur
0 = 4 107 H m1 ; ~ux
0 = 8,85 1012 F m1 . y
x r
Lorsquon demande dans la suite une reponse en fonction des
constantes definies en preambule, cela signifie que la reponse
attendue secrit avec tout ou partie des constantes introduites Fig. 2: Coordonnees.
ci-dessus.

1) Determiner la densite de courant ~, le champ electrique E,~ la densite de charge et le champ


~
magnetique B en tout point M (r, , z) de lespace. On donnera les reponses dans le syst`eme de coor-
~ on
donnees cylindriques en fonction de I et des constantes definies en preambule. Pour le calcul de B,
montrera par des considerations de symetrie que
~ , z) = B(r)~u .
B(r, (1)

2) Tracer la courbe B(r) en fonction de r.


Verifier que les relations de passage des champs electrique et magnetique sont satisfaites `a la surface du
conducteur.

3) On consid`ere le produit scalaire


~
s = ~ E. (2)
Quelle est la signification physique du produit scalaire s?
En quelle unite mecanique sexprime-t-il?
Exprimer s en tout point de lespace en fonction de I et des constantes definies en preambule.
Determiner la valeur numerique de s.

4) On consid`ere le vecteur
~ B
E ~
P~ (r, , z) = . (3)
0
Quelle est la signification physique du vecteur P~ ?
En quelle unite P~ sexprime-t-il?
Determiner le vecteur P~ en tout point M de lespace, interieur ou exterieur au conducteur. On exprimera
P~ dans le syst`eme de coordonnees cylindriques en fonction de I et des constantes definies en preambule.

5) Exprimer dans le referentiel cartesien P~ (x, y, z) = (Px , Py , Pz ) au point M (x, y, z), interieur ou
exterieur au conducteur. On exprimera les composantes cartesiennes Px (x, y, z), Py (x, y, z) et Pz (x, y, z)
de P~ en fonction de x, y, z, I et des constantes definies en preambule.
Calculer la divergence
~ P~ .
= (4)

Quelle relation existe-t-il entre s et ?


Quelle est la signification physique de cette relation?
18 `
2. DEUXIEME PARTIEL 2002-2003

z
A B
h

6) On consid`ere le rectangle R de sommets O, A, B et C situe dans le


plan Oxz. Les longueurs des c otes OA et AB sont respectivement h et
r. Calculer le flux B (r) du champ magnetique `a travers le rectangle
R oriente suivant ~uy dans les deux cas r a et r a. On donnera la
C
reponse en fonction de r, h, I et des constantes definies en preambule. O r x
Fig. 3: Rectangle.

II. R
egime quasistationnaire

Le fil est maintenant parcouru par un courant dintensite

I = I0 eit (5)

en representation complexe. On suppose que la densite de courant ~ reste uniforme dans le conducteur et
que les expressions de B~ et B (r) obtenues en regime stationnaire pour une intensite I constante restent
valables lorsque lintensite I est de la forme (5). En cas de besoin, on pourra admettre que le flux B
est pour r a
0 I0 hr2 it
B (r) = e . (6)
4a2

1) Lorsque r a, montrer que


dB (r)
v= (7)
dt
est de la forme
v = v0 ei(t) (8)
o`
u v0 > 0 et sont des nombres reels et determiner v0 et .
Quelle est la signification physique de v ?
Calculer numeriquement v0 lorsque lintensite I varie `a la frequence 50 Hz pour I0 = 1 A, h = 1 m et
r = a.

2) Montrer que la circulation C de la densite de courant ~ le long du contour = OABCO formant le


bord du rectangle R est donnee par
dB (r)
C = . (9)
dt
dB (r)
En utilisant lexpression de v = obtenue `a la question precedente, donner C en fonction de I0 ,
dt
, h, r, t et des constantes definies en preambule.

3) Deduire de la question precedente que la densite de courant ~ ne peut pas etre uniforme dans le
conducteur.

4) Pour evaluer cette non-uniformite, nous supposerons que ~ reste parall`ele `a Oz et est donne en re-
pre-sen-tation complexe par
~ = j0 (r)eit ~uz (10)
o`
u j0 (r) est une amplitude complexe qui depend de r.
19

Montrer que la circulation C sexprime alors en fonction de


f (r) = j0 (r) j0 (0) (11)
et en deduire f (r) en utilisant lexpression de C obtenue `a la question 2).

5) Verifier que pour les valeurs numeriques donnees precedemment on a



f (r)
j0 (0) 1. (12)

Lhypoth`ese faite au debut de la section II que la densite de courant etait uniforme dans le conducteur
est-elle justifiee dans ce cas?
On modifie la frequence du courant (5) sans modifier son amplitude. La frequence est lune des valeurs
= 1 Hz, = 102 Hz, = 104 Hz ou = 106 Hz.
Pour quelles de ces frequences lhypoth`ese que la densite de courant est uniforme dans le conducteur
est-elle justifiee?

C. Onde plane
electromagn
etique
On consid`ere la propagation dans le vide dune onde plane electromagnetique de frequence = 3 107 Hz.
La vitesse de la lumi`ere dans le vide est c = 3,00 108 m s1 . On utilise un syst`eme cartesien Oxyz de
vecteurs unitaires ~ux , ~uy , ~uz . Le champ magnetique de londe au point ~r (x, y, z) `a linstant t est donne
par
~ r ,t) = B0 cos(t kz)~uy
B(~ (13)
o`
u B0 est une constante.

1) Exprimer la longueur donde dans le vide en fonction des donnees et calculer numeriquement sa
valeur. Dans quel domaine du spectre electromagnetique cette onde se trouve-t-elle? Citer le nom dun
appareil qui peut creer une telle onde.

~ r ,t) de londe. Tracer sur un meme schema le syst`eme


2) Donner lexpression du champ electrique E(~
~
cartesien Oxyz, le vecteur donde k et les champs electrique E(~ ~0,0) et magnetique B(
~ ~0,0) `a linstant
t = 0 et `a lorigine O du syst`eme.

3) On designe par C un cercle de rayon a = 5 cm et de centre O. Ce cercle est le bord du disque D.


Le vecteur unitaire ~n est perpendiculaire au disque D. Le vecteur ~n se trouve dans le plan Oxy et fait
langle avec Ox. Le cercle C est oriente dans le sens direct autour de ~n.

~uz Boucle C

O ~uy y

Boucle C

~n
O ~ux
~uy

x

~ux ~n Projection sur le plan Oxy
20 `
2. DEUXIEME PARTIEL 2002-2003

~ le
Demontrer, en utilisant une des equations de Maxwell, que la circulation e(t) du champ electrique E
long de C sexprime en fonction du flux (t) du champ magnetique `a travers le disque D par
ZZ
d(t) ~ r ,t) ~n dS.
e(t) = o`
u (t) = B(~ (14)
dt D

~ r ,t) par sa valeur en


4) Justifier que lon peut remplacer dans lequation (14) le champ magnetique B(~
~ ~0,t).
O, B(
En deduire que e(t) est de la forme
e(t) = A sin(t) (15)
et determiner A.

5) Un dispositif mesure la moyenne temporelle




W () = e(t)2 . (16)

Calculer W () et tracer sa courbe en fonction de pour 0 .


Determiner les valeurs de pour lesquelles W () est minimum ou maximum. Preciser quelles sont alors
les positions du disque D par rapport au champ magnetique B.~

C+. Questions suppl


ementaires du probl`
eme C non pos
ees `
a lexamen

6) Au lieu de londe (13), on consid`ere maintenant une onde dans le vide de meme frequence et de champ
magnetique
~ (~r,t) = B0 sin(t kz)~ux + B0 cos(t kz)~uy ,
B (17)
o`
u est une constante (0 1).
~ (~r,t) de londe.
Est-ce une onde plane? Determiner lexpression du champ electrique E

7) Pour chacun des cas = 0, = 1 et 0 < < 1, tracer lallure de la courbe decrite au cours du temps
~ (~r,t) lorsque lorigine du vecteur est fixee, pour ~r donne.
par lextremite du vecteur E
En deduire la polarisation de londe dans chacun des cas.

8) On utilise le meme dispositif qu`a la question 3)


Etablir lexpression de e(t) qui remplace lequation (15) dans le cas de londe (17).

9) Calculer W () (defini par lequation (16)). Tracer sur un meme diagramme les courbes de W () en
1
fonction de (0 ) correspondant ` a = 0, = et = 1.
2
21

Corrig
e
A. Questions de cours

~ r , t) un champ de vecteur defini dans tout lespace. Un champ de vecteur conservatif peut
1) Soit E(~
etre defini par une des quatre proprietes suivantes. Le champ de vecteur satisfait alors aux trois autres
proprietes.
~ derive dun potentiel, cest-`a-dire quil existe un champ scalaire (~r, t) tel que
1. Le champ E

~ = .
E ~ (18)

~ le long dun chemin AB ne depend que des positions des extremites A et B du
2. La circulation de E
chemin.
~ le long dun chemin ferme est nulle :
3. La circulation de E
I
E ~ = 0.
~ dl (19)

~ est nul :
4. Le rotationnel du champ E
~ E
~ = 0. (20)
~ r , t) un champ de vecteur defini dans tout lespace. Un champ de vecteur `a flux conservatif peut
Soit B(~
etre defini par une des quatre proprietes suivantes. Le champ de vecteur satisfait alors aux trois autres
proprietes.
~ derive dun potentiel vecteur, cest-`a-dire quil existe un champ de vecteur A(~
1. Le champ B ~ r , t) tel
que
~ =
B ~ A.~ (21)
~ a
2. La flux de B ` travers toute surface fermee orientable S est nul :
ZZ
~ ~n dS = 0.
B (22)
S

~ `
3. La flux de B a travers une surface orientable avec bord ne depend pas de la forme de la surface
mais seulement de la forme du bord.
4. La divergence du champ B ~ est nulle :
~ B
~ = 0. (23)

2) Un conducteur parfait est un conducteur dont la conductivite est infinie. Le champ electrique est
nul sinon la puissance par unite de volume dissipee par effet Joule,
dPc ~ = E 2 ,
= ~ E (24)
d
serait infinie, ce qui est absurde. Lequation
~
B ~ E
= ~ (25)
t

~ = ~b(~r)eit
donne pour B
~ =
i B ~ = 0
~ E (26)
22 `
2. DEUXIEME PARTIEL 2002-2003

~ = 0 pour une onde de pulsation 6= 0.


et B

3) Di electrique : Milieu isolant qui ne conduit pas le courant electrique.


Lineaire : La polarisation dielectrique P~ en un point du milieu est une fonction lineaire du champ
electrique E~ au meme point. On peut ecrire cette relation `a laide dune matrice 3 3 (ij ) :

Px xx xy xz Ex
Py = 0 yx yy yz Ey . (27)
Pz zx zy zz Ez

Homog`
ene : Les proprietes du milieu sont les memes en tout point.
Isotrope : Toutes les directions sont equivalentes.
Dans un dielectrique lineaire homog`ene isotrope la relation (27) est une proportionnalite

P~ = 0 E
~ (28)

avec la susceptibilite dielectrique constante dans tout le milieu.

B. Etude dun fil cylindrique


I. R
egime stationnaire

~ montre que ~ est constant et uniforme dans le conducteur. Lintensite du courant


1) La relation ~ = E
2
est I = a j. On a donc

I
~u si r a
~ = a2 z (29)
0 si r > a

et, pour le champ electrique,


~ = I
E ~uz . (30)
a2

La divergence de E ~ E
~ etant nulle, la densite de charge = 0 ~ est nulle :

= 0. (31)

~ est perpendiculaire `a ce plan :


Le plan OzM est plan de symetrie. On en deduit que B
~ = B(r, , z)~u .
B (32)

Linvariance par translation parall`element `


a Oz et par rotation autour de Oz impliquent que B(r, , z)
ne depend que de r :
B~ = B(r)~u . (33)
Le theor`eme dAmp`ere applique ` ~ le long
a un cercle de rayon r et daxe Oz donne la circulation de B
de en fonction du courant enlace par :

0 Ir2
si r a
2rB(r) = a2 (34)
I si r a.
0

On a donc
23

B(r)

Ir 0 I
0 2 ~u
si r a 2a
~ = B(r)~u = 2a
B (35)
0 I ~u

si r a.
2r

2) Les champs E ~ et B~ sont continus `a la surface du


conducteur. Les relations de passage sont donc verifiees
sil ny a ni charge ni courant surfacique.
O a r

~ est la puissance par unite de volume dissipee par effet Joule. Il sexprime
3) Le produit scalaire s = ~ E
en W m3 . On a dans le conducteur
2
~ = I ~uz I ~uz = I .
s = ~ E (36)
a2 a2 2 a4
En dehors du conducteur s = 0. En conclusion, nous avons
2
I = 103 W m3 si r a
s = 2 a4 (37)

0 si r a.

4) Le vecteur P~ est le vecteur de Poynting. Le flux de P~ `a travers une surface est la puissance electro-
magnetique qui traverse cette surface. P~ sexprime en W m2 .
r r
~ B ~

si r a 2 2 si r a

~ E I 0 I a2 I a
P = = ~
u z ~
u = ~
u r (38)
0 0 a2 2 1
si r a 2 2 a2

1
si r a
r r
soit
I 2r

~ur si r a
P~ = 2 2 a4 (39)
2

I
~ur si r a.
2 a2 r
2

5) Comme
x
x~ux + y~uy r p
~ur = = y x2 + y 2
r , r= (40)
r
0
p
on a dans le conducteur (pour x2 + y 2 a)

I 2x

2 2 a4
P~ =

I 2y (41)
2 4
2 a
0
24 `
2. DEUXIEME PARTIEL 2002-2003
p
et en dehors du conducteur (pour x2 + y 2 > a)

I 2x

2 2 a2 (x2 + y 2 )

~
P = I 2y . (42)

2 2 a2 (x2 + y 2 )
0
p
On calcule dans le conducteur (pour x2 + y 2 a)

Px Py Pz I2 I2 I2
= + + = 2 4 2 4 = 2 4 . (43)
x y z 2 a 2 a a
Pour obtenir en dehors du conducteur, calculons
   
x 1 2x2 y 1 2y 2
= , = (44)
x x2 + y 2 x2 + y 2 (x2 + y 2 )2 y x + y2
2 x2 + y 2 (x2 + y 2 )2
do`
u par addition
   
x y 2 2x2 + 2y 2
+ = 2 = 0. (45)
x x2 + y 2 y x2 + y 2 x2 +y 2 (x + y 2 )2
On en deduit quen dehors du conducteur
~ P~ = 0.
= (46)

Remarque. Le calcul de est plus simple en coordonnees cylindriques. La divergence du vecteur


~ = Ar ~ur + A ~u + Az ~uz
A (47)

est donnee par


~ A
~ = 1 (rAr ) + 1 A + Az . (48)
r r r z
Cette expression appliquee au vecteur (39) redonne les equations (43) et (46)

I2
~ P~ = si r a
= 2 a4 (49)

0 si r a.

En comparant (49) et (37) on a en tout point la relation

= s ou encore ~ P~ + ~ E
~ = 0. (50)

Cette relation exprime la forme locale de la conservation de lenergie. En effet, la forme locale de la
conservation de lenergie sexprime par le theor`eme de Poynting
u ~ ~ ~ = 0,
+ P + ~ E (51)
t
u
o`
u u est la densite denergie electromagnetique. En regime stationnaire, = 0 et (51) secrit (50).
t

~ = B(r)~u a` travers le rectangle OABC est (~u = ~uy pour les points
6) Le flux du champ magnetique B
du rectangle) Z r
B = h B (r ) dr (52)
0
25

Pour r a, lequation (35) donne


Z r
0 Ir 0 Ihr2
B (r) = h 2
dr = . (53)
0 2a 4a2
Pour r = a
0 Ih
B (a) = . (54)
4
Pour r a Z r
0 I 0 Ih 0 Ih r
B (r) = B (a) + h dr = + ln . (55)
a 2r 4 2 a
En regroupant ces resultats

Ihr2
0
si r a
B (r) = 4a2 (56)
 
0 Ih 1 + 2 ln r

si r a.
4 a

II. R
egime quasistationnaire

1) v est la force electromotrice dinduction qui apparat dans le contour rectangulaire OABCO.
Le flux B est pour r a
0 I0 hr2 it
B (r) = e . (57)
4a2
On a
0 I0 hr2 i(t/2)
v= e (58)
4a2
damplitude et phase
0 I0 hr2
v0 = et = . (59)
4a2 2
Numeriquement v0 = 3,1 105 V.

~ la circulation C de la densite de courant sexprime en fonction de la circulation de


2) Comme ~ = E,
~
E: I I

~

C = ~ dl = E dl. (60)

La circulation de E~ sexprime en fonction de B en utilisant le theor`eme de Stokes et les equations de


Maxwell :
I ZZ ZZ ~ ZZ
E~

dl = ~ E)
( ~ ~uy dS = B
~uy dS =
d ~ ~uy dS = dB (r) .
B (61)
R R t dt R dt
dB (r)
On a donc C = = v et dapr`es (58)
dt
0 I0 hr2 it
C = i e . (62)
4a2

3) La circulation dun vecteur uniforme A~ le long du circuit est nulle :


I I
~
~

A dl = A dl = 0. (63)

| {z }
=0
26 `
2. DEUXIEME PARTIEL 2002-2003

La circulation de la densite de courant le long du circuit netant pas nulle dapr`es (62), la densite de
courant ~ ne peut pas etre uniforme dans le conducteur, contrairement aux hypoth`eses faites au debut
de la section II.

4) La circulation C est
Z Z Z Z





C= ~ dl + ~ dl + ~ dl + ~ dl = h(j0 (0) j0 (r))eit = hf (r)eit . (64)
OA
| AB{z } BC
| CO{z }



=0 (~
dl) =0 (~
dl)

En comparant `a (62) on obtient


0 I0 r2
f (r) = i . (65)
4a2

5) On evalue j0 (0) en supposant la densite de courant uniforme :

I0
j0 (0) . (66)
a2
Lequation (65) donne alors

f (r) 0 r2 0 a2 0 a2

j0 (0) = 2 104 s (67)
4 4 2

La densite de courant est approximativement uniforme lorsque



f (r)
j0 (0) 1. (68)

Cest le cas `a la frequence = 50 Hz de lenonce. Lhypoth`ese faite au debut de la section II que la


densite de courant etait uniforme dans le conducteur etait donc justifiee. Cette hypoth`ese reste justifiee
aux frequences = 1 Hz et = 102 Hz mais ne lest plus aux frequences = 104 Hz et = 106 Hz.

C. Onde plane
electromagn
etique

c
1) La longueur donde est = = 10 m. Ce sont des ondes courtes. On peut creer une telle onde avec

un emetteur radio a
` ondes courtes.

2) On sait que E, ~ ~k forment un tri`edre direct et que B = E/c. Connaissant B


~ B, ~ et ~k = k~uz , on en
deduit que le champ electrique de londe est
~ r ,t) = cB0 cos(t kz)~ux .
E(~ (69) z

k
On peut remarquer que le champ electrique sexprime par
O
B

~ r ,t) = cB(~
E(~ ~ r ,t) ~uz (70) y

E
x
en fonction du champ magnetique.
27

3) Le theor`eme de Stokes applique au disque D de bord C (lorientation de C correspondant `a ~n) donne,


~
avec lequation de Maxwell ~ E ~ = B ,
t
I ZZ ZZ ~
e(t) = ~
E

dl = ~ E)
( ~ ~n dS = B
~n dS.
C D D t

En changeant lordre de la derivation et integration dans la derni`ere expression on retrouve la loi de


linduction de Faraday :
ZZ
d(t) ~ r ,t) ~n dS.
e(t) = o`
u (t) = B(~ (71)
dt D

z a

4) Sur le disque D, z varie entre a et a. On a donc |kz| = 2 2 0,06 1. Le champ

electromagnetique reste donc presque uniforme sur le disque D et on peut y remplacer le champ ma-
gn ~ ~ ~
RR e-ti-que B(~r,t) par sa valeur2 en O, B(0,t) = B0 cos(t)~uy . Lequation (71) donne alors (t) =
D B0 cos(t)~uy ~n dS = B0 a sin cos(t) et

e(t) = A sin(t) o`
u A = B0 a2 sin . (72)

W
5) La moyenne temporelle est W0



A2
W () = e(t)2 = A2 sin2 (t) = .
2
Do`
u, dapr`es lequation (72)

B02 2 a4 2 0
W () = W0 sin2 o`
u W0 = . (73)
2 2

~ est alors parall`ele au disque D et le flux est


W () est minimum pour = 0 (mod ). Le champ B
~ est alors perpendiculaire au disque D et le
nul. W () est maximum pour = (mod ). Le champ B
2
flux || est maximum.

C+. Questions suppl


ementaires du probl`
eme C

6) Le champ B ~ est celui dune onde plane de vecteur donde ~k = k~uz . Le champ electrique sobtient par
~ = cB
lequation (70) E ~ ~uz :

~ (~r,t) = cB0 cos(t kz)~ux cB0 sin(t kz)~uy .


E (74)

7) Pour chacun des cas = 0, = 1 et 0 < < 1, on peut tracer lallure de la courbe decrite au cours
du temps par lextremite du vecteur E~ (~r,t) lorsque lorigine du vecteur est fixee, pour ~r donne. Cela
permet den deduire la polarisation de londe dans chacun des cas. La polarisation de londe sobtient
aussi avec les representations complexes E x = cB0 ei(tkz) , Ey = icB0 ei(tkz) , en formant le rapport
Ey
= = i.
Ex
28 `
2. DEUXIEME PARTIEL 2002-2003

y
y

x O x x x x x
O O
y
y

= 0, = 0 est reel : po- = 1, = i : polarisa- 0 < < 1, Arg[] = /2 : polarisation ellip-


larisation rectiligne sui- tion circulaire droite ; tique droite (les axes principaux de lellipse
vant Ox ; sont x Ox et y Oy).

8) Comme `a la question 4) on a
ZZ
(t) = (B0 sin(t kz)~ux + B0 cos(t kz)~uy ) ~n dS
D
a2 (B0 sin t ~ux + B0 cos t ~uy ) ~n = B0 a2 ( cos sin t + sin cos t) (75)

et
e(t) = B0 a2 (sin sin t cos cos t). (76)



1 1
9) En utilisant sin2 t = cos2 t = et hsin t cos ti = hsin 2ti = 0 on calcule la moyenne
2 2
temporelle




W () = e(t)2 = (B0 a2 )2 sin2 sin2 t + 2 cos2 cos2 t
2 sin cos hsin t cos ti)
1  1  
= (B0 a2 )2 sin2 + 2 cos2 = (B0 a2 )2 (1 2 ) sin2 + 2 .
2 2
Ou, avec W0 donne par lequation (73),
 
W () = W0 (1 2 ) sin2 + 2 . (77)

W
Pour = 0 (polarisation rectiligne), on retrouve les resultats W0
=1
de la question 5)
Pour = 1 (polarisation circulaire), W () = W0 est une = 12
constante independante de . 1
2 W0
Pour 0 < < 1 (polarisation elliptique), W () est minimum
=0
pour = 0 (mod ) et maximum pour = (mod ).
2 0


2
29

3. Examen 2002-2003


Enonc
e
14 janvier 2003
duree : 3 heures
sans documents, calculatrices autorisees

Les parties A et B sont independantes.


Bar`eme indicatif : A I=2, A II=7, A III=7, B=4.

A. Communication avec un sous-marin

ionosph`ere

onde G

onde S
emetteur
ocean
terre
sous-marin antenne

Ce probl`eme examine quelques aspects de la propagation dondes electromagnetiques dun emetteur


terrestre vers un sous-marin en plongee dans un ocean lointain. On envisage dutiliser les frequences
1 = 40 Hz bande ELF (extremely low frequency) ;
2 = 4 kHz bande VLF (very low frequency) ;
3 = 400 kHz bande MF (medium frequency).
La propagation de londe a dabord lieu dans le guide donde forme entre la surface de la terre ou des
oceans et lionosph`ere. On fera lhypoth`ese simplificatrice que les parois du guide sont parfaitement
conductrices. Cette onde, nommee dans ce probl`eme onde G (guidee), est etudiee dans le cadre de
lapproximation de loptique geometrique (partie I) puis de lelectromagnetisme (partie II).
Londe dans locean, nommee dans ce probl`eme onde S (sous-marine), est ensuite etudiee dans la partie
III.
Les parties I, II et III peuvent etre resolues independamment les unes des autres.
30 3. EXAMEN 2002-2003

Notations et valeurs num


eriques
Le tri`edre orthonorme direct associe au syst`eme de coordonnees cartesiennes (x, y, z) est note (~ux , ~uy ,
~uz ).
Les valeurs numeriques des constantes sont
0 = 4 107 H m1 ;
0 = 8,85 1012 F m1 ;
c = 3,00 108 m s1 ;
h = 75 km ;
R = 6400 km ;
Pour leau de mer
= 4,0 S m1 ;
r = 80.

I. Etude g
eom
etrique
ionosph`ere
A
h

E
l F
terre

On etudie la propagation de londe G entre la terre et lionosph`ere dans le cadre de lapproximation de


loptique geometrique. On fait les hypoth`eses :
la terre est une boule parfaitement conductrice de rayon R = 6400 km ;
lionosph`ere est un milieu parfaitement conducteur situe au-dessus de laltitude h = 75 km ;
les rayons de londe sont rectilignes dans la basse atmosph`ere (assimilee au vide) entre la terre et
lionosph`ere et se reflechissent suivant les lois de Snell-Descartes sur les surfaces de la terre et de
lionosph`ere.
Lemetteur radio E, situe `a laltitude 0, rayonne londe G dans toutes les directions. Le recepteur F est
egalement situe `a laltitude 0.

1) On envisage le type de propagation o`


u le recepteur recoit londe emise apr`es une seule reflexion en A
sur lionosph`ere.
Montrer que ce type de propagation na lieu que si la distance l de lemetteur au recepteur est inferieure
` une distance lc .
a
Determiner lc en fonction de R et h (on donnera une expression simple compte tenu du fait que h R).
Calculer la valeur numerique de lc .

2) Faire un schema qui montre que londe G peut atteindre un point quelconque de la surface terrestre
en suivant un chemin qui comporte plusieurs reflexions sur lionosph`ere et la terre.
31

II. Electromagn
etisme

1) Ondes
electromagn
etiques dans le vide
On consid`ere deux ondes electromagnetiques P1 et P2 se propageant dans le vide. Les ondes P1 et P2 sont
des ondes planes progressives harmoniques polarisees rectilignement, de meme frequence , de longueur
donde et de vecteurs dondes respectifs
~k1 = ~ux + ~uz = k~u1 (1)
~k2 = ~ux ~uz = k~u2 (2)

o`
u k, , sont des constantes reelles (k > 0, > 0, 0) et o`
u ~u1 et ~u2 sont des vecteurs unitaires.
~
Soit (0 < ) langle que fait k1 avec ~uz .
2
Les champs magnetiques B~ 1 et B
~ 2 des ondes P1 et P2 sont respectivement, en notation complexe,

~1 (~r, t) =
B
~
B0 ~uy ei(tk1 ~r) (3)
~2 (~r, t) =
B
~
B0 ~uy ei(tk2 ~r) (4)

a) Exprimer , k, ~u1 , ~u2 , , et en fonction de , c, , ~ux et ~uz .

b) Determiner la representation complexe E ~ (~r, t) du champ electrique de londe P1 . On ecrira la


1
E1x
reponse sous forme de vecteur colonne E 1y ecrit de la facon la plus simple possible en fonction de
E1z
x, z, t, c, B0 , , , et .
Representer sur une figure les vecteurs ~k1 , E~ 1 (~0, 0) et B~ 1 (~0, 0) ainsi que langle .

~2 (~r, t) de londe P2 .
c) Memes questions (y compris la figure) que b) pour le champ electrique E

d) On consid`ere londe P resultant de la superposition des ondes P1 et P2 . Determiner, en represen-


tation complexe, les champs electrique E~ = E
~1 + E
~2 et magnetique B
~ = B ~ ~
1 + B2 de londe P . On ecrira
fx (z)
les reponses sous forme de vecteurs colonnes du type Aei(tx) fy (z).
fz (z)

2) Propagation guid
ee

z
La propagation de londe G ` a grande distance l h ionosph`ere
est etudiee dans un mod`ele de Terre plate : h
le demi-espace z 0 est un conducteur par-
fait qui represente la terre ; basse atmosph`ere
le demi-espace z h = 75 km est un conduc-
teur parfait qui represente lionosph`ere ; O x
la region 0 z h entre ces conducteurs est terre
vide et represente la basse atmosph`ere.
32 3. EXAMEN 2002-2003

On se propose de montrer que londe P obtenue en 1.d) peut, par restriction `a la region 0 z h,
decrire une onde guidee entre les deux conducteurs.

a) Justifier que les conditions aux limites sont :


~
E(x, y, z, t) est parall`ele `
a ~uz x, y, t pour z = 0, h ;
~
B(x, y, z, t) est perpendiculaire ` a ~uz x, y, t pour z = 0, h.

b) Montrer que les conditions aux limites sont verifiees pour les valeurs = n donnees par

cos n = nU avec n = 0, 1, 2, . . . , N 1 (5)

o`
u U est une constante que lon exprimera en fonction de et h.

c) Pour chacune des frequences 1 = 40 Hz, 2 = 4 kHz et 3 = 400 kHz, determiner le nombre N
dans lequation (5) et les valeurs numeriques, en degres, des angles 0 et 1 (sils existent). On rappelle
que 0 < 90 .

d) Pour quelles frequences le guide donde est-il monomode? Donner la reponse de facon litterale, puis
numerique.

III. Propagation dans loc


ean

~ D,
1) Ecrire les equations de Maxwell macroscopiques dans un milieu en termes des champs E, ~ B,
~ H~ et
des densites de charges libres et de courants libres ~.

2) Le milieu considere est de leau de mer. Cest un milieu lineaire, homog`ene, isotrope et non magnetique,
de permittivite relative r = 80 et de conductivite electrique = 4,0 S m1 dans lequel

B ~
~ = 0 H, D ~
~ = r 0 E et ~
~ = E. (6)

Montrer que pour une onde de frequence , avec 1 < < 3 (1 = 40 Hz, 3 = 400 kHz), on peut
negliger un terme dans les equations de Maxwell et les ecrire, en notation complexe, sous la forme :

~ E
~ = i0 H,~ (7)
~ H
~ = ~
E, (8)
~ E~
= , (9)
0 r
~
~ H
= 0. (10)

3) On fait lhypoth`ese que le milieu (locean) occupe le demi-espace z 0. On se propose de montrer


quil peut exister une onde S definie dans tout le milieu (cest-`
a-dire pour tout z : z 0) par

~ r , t) = ~ux E0 eit e(q+i)z


E(~
(onde S) (11)
~ r , t) = ~uy E0 eit e(q+i)z
H(~

o`
u E0 , q et sont des constantes reelles et une constante complexe.

a) On suppose q 6= 0 et = 6 0. Londe S est-elle une onde plane ? Est-elle homog`ene ? Dans quelle
direction et dans quel sens se propage-t-elle? Est-elle une onde progressive?
33

b) Montrer, sur la base darguments physiques, que q > 0 et > 0. (Aucun calcul nest demande.)

c) En quelle unite est-il mesure ? (La reponse doit tenir en un seul mot.) Justifier la reponse.

4.a) Verifier que londe S est solution de (9) et (10) sil ny a pas de charges libres dans le milieu
( = = 0).

b) Determiner les constantes q, et pour que londe S soit solution des equations (7) et (8). (Indi-
1+i
a = q ; la reponse secrit en fonction de , et 0 ; on peut utiliser i = .)
cations : on arrive `
2

5) Applications num
eriques pour la fr
equence 1 = 40 Hz

a)
Determiner la longueur donde e dans le milieu ;
comparer e ` a la longueur donde dans le vide ;
peut-on envisager dinstaller une antenne demi-onde (cest-`a-dire un fil rectiligne de longueur egale
`a 0,5 fois la longueur donde) `a bord du sous-marin et pour lemetteur terrestre?

~ est
b) Lantenne du sous-marin permet de detecter londe S si lamplitude du champ electrique E
1
1 V m . Londe G engendre dans leau de mer une onde S dont lamplitude au niveau de la mer est
E0 = 1 V m1 .
Jusqu`a quelle profondeur d le sous-marin peut-il recevoir cette onde?

6) Repondre `
a la question 5.b) precedente pour les frequences 2 = 4 kHz et 3 = 400 kHz.
Expliquer en quoi il est avantageux, pour les communications avec les sous-marins, dutiliser les trois
frequences envisagees plut
ot quune seule.

B. Pouvoir s
eparateur dune lunette astronomique
On consid`ere une lunette astronomique dont lobjectif est constitue par une lentille mince convergente,
supposee parfaitement stigmatique de distance focale f = 900 mm. La lentille est limitee par un dia-
phragme circulaire, centre sur laxe optique, de diam`etre D = 60 mm. La lunette est precedee dun filtre
qui ne laisse passer que la lumi`ere dont la longueur donde dans le vide est voisine de 0,65 m. La
lunette est placee dans lair dindice na = 1.
A laide de cet objectif on desire observer une etoile double, assimilee `a deux sources ponctuelles S1 et S2
situees `a linfini, de meme intensite, separees par langle 1. La source S1 se trouve sur laxe optique
de la lunette. Les images des deux etoiles S1 et S2 dans le plan focal image de lobjectif de la lunette
sont designees par I1 et I2 respectivement.

1) Faire un schema representant lobjectif, son centre C, la distance f , laxe optique, les images I1 et I2 ,
langle ainsi que le trajet de trois rayons lumineux R1 , R2 et R3 issus de S2 . De plus :
la source S2 `a linfini ne figure pas sur le schema ;
utiliser un angle plus grand quen realite pour que cet angle soit clairement indique sur la figure ;
representer deux surfaces dondes et de la lumi`ere issue de S2 , avant que la lumi`ere natteigne
lobjectif et apr`es que la lumi`ere lait traverse ;
le rayon Ri (i = 1, 2 ou 3) coupe les surfaces dondes et aux points Ai et Bi respectivement.
34 3. EXAMEN 2002-2003

2) Questions sur le sch


ema

a) Quappelle-t-on lentille parfaitement stigmatique?


Comment cette propriete apparat-elle sur le schema?

b) Enoncer le theor`eme de Malus concernant loptique geometrique. Quelles proprietes du schema


deduisez-vous de ce theor`eme?

c) Donner la definition du chemin optique le long dun chemin allant dun point A `a un point B.

d) Soit [Ai Bi ] le chemin optique le long du rayon Ri . Quest-ce qui permet daffirmer que les trois
chemins optiques [Ai Bi ] (pour i = 1, 2, 3) sont egaux?

e) Exprimer en fonction de et de f la distance d separant les images I1 et I2 . Calculer numeriquement


d en m pour = 1,2 104 rad.

3) En fait, lorsque la lunette est dirigee vers une etoile, assimilee `a une source ponctuelle S placee `a
linfini, on observe dans le plan focal image de la lentille une tache circulaire centree en I, appelee tache
dAiry.

ES ()
La figure ci-contre represente leclairement ES () observe
le long dune droite I du plan focal image. Le premier
minimum nul, autour du centre brillant, correspond `a un
f
rayon = 1,22 .
D
a) Quel est le nom du phenom`ene physique observe ?
Calculer numeriquement, en m, la valeur du rayon du
disque dAiry dans le plan focal image de lobjectif pour
la lunette etudiee.

I

b) La lunette est `a present dirigee vers letoile double. Soit E(x) leclairement observe dans le plan
focal image de lobjectif sur la droite x x qui passe par les points I1 (dabscisse x = 0) et I2 (dabscisse
x = d). Representer lallure de leclairement E(x) lorsque

) d ) d = ) d .

c) Quelle est, en radian, le pouvoir separateur de lobjectif, cest-`a-dire la valeur minimale de la


distance angulaire separant les deux composantes dune etoile double pour que les deux etoiles puissent
etre separees `a laide de la lunette etudiee? On supposera que les deux images sont encore distinctes si
le maximum central de la tache dAiry correspondant `a lune des deux etoiles concide avec le premier
minimum nul de la tache dAiry correspondant `a lautre etoile.
Peut-on esperer, avec cet objectif, separer les deux composantes S1 et S2 lorsque = 1,2 104 rad?
35

Corrig
e

A. Communication avec un sous-marin


I. Etude g
eom
etrique

1) La portee maximale apr`es une reflexion L A 2)


E E
en A correspond au rayon EA horizontal. Le
F
theor`eme de Pythagore pour le triangle rec-

h
R+
tangle CEA donne (R + h)2 = R2 + L2 do` u C
R
L2 = 2Rh + h2 2Rh et lc 2L soit

lc = 8Rh = 1900 km. F
Si l < lc on peut recevoir londe qui suit le tra-
C
jet EAF avec une reflexion au point A sur la
bissectrice de langle ECF .
Remarques. La propagation directe suivant la ligne droite EF passe dans la terre et est impossible
avec lhypoth`ese que la terre est un conducteur parfait.
Lionosph`ere est reflechissante pour les frequences . 30 MHz ; les ondes sont (partiellement) reflechies `a
des altitudes entre 60 et 400 km. Les proprietes reflechissantes de lionosph`ere varient sous laction de la
lumi`ere solaire.
Il y a un grand nombre de rayons subissant des reflexions sur lionosph`ere et la terre qui vont du point
E au point F . Les ondes correspondantes interf`erent en F , mais par suite des variations temporelles

de lOionosph`ere, ces interferences sont tant
ot constructives, tant
ot destructives et le signal recu fluctue
aleatoirement dans le temps (phenom`ene du fading [evanouissement]).

II. Electromagn
etisme

2 2
1.a) = 2, k = , ~u1 = (sin )~ux + (cos )~uz , ~u2 = (sin )~ux (cos )~uz , = sin , =
c c
2 2 c
cos = cos , = .
c

b) Dans une onde plane progressive harmonique dans levide ~ ~ ~


letri`edre
E, B, k est orthogonal direct
0 sin
et E = cB. On a donc E ~1 = cB
~1 ~u1 = cB0 ei(txz) 1 0 soit, en effectuant le produit
0 cos
vectoriel :

cos z z
E~1 = cB0 ei(txz) 0 .
sin ~
k1

c) De meme x x
~2
B
~1
B
cos
~2 = cB0 ei(tx+z)
E 0 .
~
k2
sin ~1 ~2
E E
36 3. EXAMEN 2002-2003

~ = B
d) La superposition des champs donne B ~1 + B
~2 = B0 ~uy ei(tx) (eiz + eiz ) soit


0

~ = 2B0 ei(tx) cos(z)
B
0


(eiz eiz ) cos
~ = cB0 ei(tx))
et E 0 soit
(eiz + eiz ) sin


i sin(z) cos
~ = 2cB0 ei(tx))
E 0 .
cos(z) sin

2.a) Les relations de passage entre deux milieux imposent que E ~ k (composante parall`ele `a linterface de
~ et B
E) ~ (composante perpendiculaire ` ~ sont continues. Comme dans un conducteur
a linterface de B)
~ ~ ~ ~
parfait E = B = 0, Ek = B = 0 au voisinage du conducteur. Les conditions aux limites expriment ces
relations en tout point (x, y), `
a tout instant (t) au voisinage des deux conducteurs (pour z = 0, h).

b) Les conditions aux limites se reduisent `


a la condition

sin(h) cos = 0.

Elle peut etre realisee soit si (i) cos = 0, soit si (ii) sin(h) = 0. Le cas (ii) donne h = n, avec n Z.
2 2h n
Comme = cos , le cas (ii) secrit cos = n ou cos = . Le cas (i) est un cas particulier
2h

(n = 0) du cas (ii). Comme 0 < (0 cos < 1) lequation donnant est
2
 
2h
cos = nU n = 0, 1, 2, . . . , N 1 avec U= et N=
2h

o`
u X designe le plus petit entier plus grand ou egal `a X.

c)
= c/ N = 2h/ valeurs de 0 et 1
40 Hz 7500 km 1 0 = 90
4 kHz 75 km 2 0 = 90 , 1 = 60
400 kHz 750 m 200 0 = 90 , 1 = 89,43


d) Lentier n numerote (certains) modes du guide donde. Le mode n = 0 correspond `a = ,
2
~k1 = ~k2 = k~ux et E
~ = 2E
~ 1, B
~ = 2B
~ 1 . Ce mode est un mode TEM qui se propage `a toutes les frequences.
Si 2h, il ny a pas dautre mode qui se propage (n ne peut prendre que la valeur 0). La frequence
c
correspondant `a = 2h est = 2 kHz. Le guide est donc monomode pour les frequences
2h

c
0< ou, numeriquement, 0 < 2 kHz.
2h
37

III. Propagation dans loc


ean

1) Les equations de Maxwell macroscopiques dans un milieu sont

B~
~ E
~ = ,
t
D~
~ H
~ = ~ + ,
t
~ D
~ = ,
~ B
~ = 0.

2) Pour une onde de frequence les champs (complexes) dependent du temps par le facteur eit .

Portons (6) et i dans les equations de Maxwell. On obtient
t

~ E
~ = ~
i0 H,
~ H
~ = ~
( + ir 0 )E,
~ E~
= ,
0 r
~
~ H = 0.
r 0
Numeriquement, = 1,1 109 < 5 104 est tr`es petit par rapport `a 1 pour les frequences < 3 .

Cela justifie de negliger le terme ir 0 par rapport `a . On obtient alors les equations de lenonce.

3.a) La phase de londe est t + z. Les surfaces dondes (surfaces equiphases) sont les plans z =Cte :

londe S est une onde plane.

~ et B
Les champs E ~ ont les memes valeurs en tous les points dun plan donde `a un instant donne :

londe S est homog`ene.

Londe se propage suivant Oz, dans le sens de z decroissant si > 0 ou de z croissant si < 0.
~ E0 eqz , depend de z. Londe se propage en se deformant :
Le module de lamplitude de E,

londe S nest pas une onde progressive.

~ et B
b) Si la constante q est negative, E ~ tendent vers linfini quand z , ce qui est impossible.
On a donc
q > 0.

Cela signifie que lamplitude de londe crot avec z (decrot avec la profondeur). Une onde dans le milieu
perd de lenergie en se propageant (effet Joule). Son amplitude doit decrotre dans le sens de propagation.
On doit donc avoir
> 0.

Remarque. Dans locean reel, la profondeur est limitee et q < 0, < 0 est egalement possible (onde
emise par le sous-marin vers la surface).
38 3. EXAMEN 2002-2003

c) Reponse : ohm. Justification : E est mesure en V m1 , H en A m1 et , qui est homog`ene `a leur


rapport, est mesure en V A1 , cest-`
a-dire en ohms.
Remarque. est appele limp
edance de londe.

i0
4) Portons londe (11) dans les equations de Maxwell (710). On obtient q +i = et q +i = .

2
Eliminons entre ces deux equations
r(former leur produit). On obtient (q + i) = i0 . On en deduit
1+i 0
( i = ) que q + i = (1 + i) . Dapr`es la question 3.b), on doit rejeter le cas q < 0, < 0.
2 2
On a donc
r r
0 q + i 0
q== et = = (1 + i).
2 2

r
2 4
5.a) e = = . Numeriquement e = 250 m.
0
Dans le vide la longueur donde = c/ = 7500 km est 30 000 fois plus grande.
~ `a larri`ere
On peut envisager de deployer une longue antenne filaire e /2 remorquee parall`element `a E
du sous-marin.
Pour lemetteur terrestre, on devra se contenter dune antenne tr`es petite devant .

6 ln 10
b) eqd = 106 donne d = e et d = 550 m.
2

6) Lorsque est multiplie par 100, e et d sont divises par 10.


d
40 Hz 550 m
4 kHz 55 m
400 kHz 5,5 m
A tr`es basse frequence le taux de transmission est tr`es faible. La frequence 1 peut etre utilisee pour
envoyer une alerte au sous-marin en plongee et lui demander de remonter pour pouvoir communiquer
avec des frequences plus hautes.

B. Pouvoir s
eparateur dune lunette astronomique

B1
1) La figure sera consideree correcte si : I2
R1
limage I1 est au foyer image et I2 est dans le A1

plan focal image ; B2


C I1
les rayons Ri sont parall`eles en entree, conver- B3

gents en I2 en sortie ; R2
A2
le rayon qui passe par C fait le meme angle
avec laxe optique tant avant quapr`es la R3
A3
lentille ;
f
est un plan perpendiculaire aux rayons
dentree et une sph`ere centree en I2 .
39

2.a) Une lentille est parfaitement stigmatique lorsque les rayons issus dun point objet S convergent vers
un point I (ou semblent provenir dun point I) apr`es traversee de la lentille.
Cette propriete explique lexistence des images ponctuelles I1 et I2 . Elle implique la convergence des
rayons Ri au point I2 .

b) Theor`
eme de Malus. Dans un milieu isotrope les rayons lumineux sont orthogonaux aux surfaces
donde. est un plan perpendiculaire `a la droite CS2 et est une sph`ere centree en I2 .

c) Chemin optique. Le chemin optique [AB] le long dun chemin allant dun point A `a un point
B est lintegrale Z
[AB] = ndl (12)

o`
u n est lindice du milieu. Lindice n nest pas suppose constant. Une particule qui se meut `a la vitesse
c dl ndl
(de phase) de la lumi`ere v = parcourt lelement dl dans le temps dt = = . Une autre definition
n v c
du chemin optique [AB] est
[AB] = ctAB (13)
Z
o`
u tAB = dt est le temps que mettrait une telle particule pour parcourir le chemin (ce chemin nest

pas necessairement un rayon lumineux).

d) Lamplitude complexe a la meme phase (respectivement ) en tout point de la surface donde


(respectivement ). La difference de phase est
2c
= = (14)

o`u est le temps que met la lumi`ere pour parcourir un quelconque des rayons Ri de Ai `a Bi . Dapr`es
lequation (13) les trois chemins optiques [Ai Bi ] sont egaux :

[A1 B1 ] = [A2 B2 ] = [A3 B3 ] = c. (15)

e) Comme 1, d = f = 108 m.

f
3.a) La diffraction. Le rayon du disque dAiry est = 1,22 = 12 m.
D

b) E(x) E(x) E(x)

) d ) d = ) d

x x x
0 d 0 d 0d


c) Le pouvoir separateur de lobjectif est = = 1,22 = 1,32 105 rad.
f D
Les deux composantes peuvent donc etre separees lorsque = 1,2 104 rad.
40 4. EXAMEN (2E SESSION) 2002-2003

4. Examen (2e session) 2002-2003


Enonc
e
2 septembre 2003
duree : 3 heures
sans documents, calculatrices autorisees
Bar`eme indicatif : A = 3, B = 11, C = 6.

A. Ondes

1) Definir bri`evement ce quest une onde.

2) Definir bri`evement ce quest une onde acoustique.


A quelle(s) condition(s) est-elle audible?
Quelle est la grandeur physique qui caracterise la hauteur dun son pur?

3) Quappelle-t-on onde transversale et onde longitudinale? Donner dans chaque cas un exemple.

4) Quappelle-t-on surface donde?

B. Propagation dondes acoustiques


On supposera que les parois des differents tuyaux, qui interviennent tout au long de cette partie,
nexercent aucun frottement sur le (ou les) fluide(s). On neglige, de plus, laction de la pesanteur.

I. Propagation dune onde acoustique dans un tuyau de section constante


contenant un fluide unique
Un tuyau cylindrique de section constante S, daxe x x, contient un fluide qui, au repos, est `a la pression
P0 , `a la temperature T0 ; sa masse volumique est 0 (cf. figure 4).
On consid`ere une tranche de fluide qui, au repos, est situee entre les abscisses x et x + dx. Le passage
de londe acoustique saccompagne dun deplacement densemble des molecules contenues dans le plan
dabscisse x : soit (x, t) ce deplacement `a linstant t ; ainsi la tranche de fluide consideree se trouve `a
linstant t entre les plans x + (x, t) et x + dx + (x + dx, t). On notera de facon comparable :
v(x, t), la vitesse de deplacement de la section dabscisse x `a linstant t ;
p(x,t), la surpression liee au passage de londe en x `a t ; ainsi la pression secrira P (x, t) = P0 +
p(x, t) ;
41

(x + dx, t)
dx

x O x

x x + dx dx

(x, t)

Fig. 4: Le tuyau sonore.

(x, t), la masse volumique du fluide `a labscisse x `a linstant t ;


On se limitera aux mouvements de faibles amplitudes ; ainsi le deplacement (x, t), la surpression p(x, t),
la variation de la masse volumique (x, t) 0 et leurs derivees peuvent etre consideres comme des
infiniment petits du premier ordre. On negligera dans la suite tous les infiniment petits dordre superieur
ou egal `a deux.

1) Quelle est, `a linstant t, la nouvelle epaisseur dx de la tranche consideree ? Exprimer, au premier


ordre, cette epaisseur en fonction de dx et de lune des derivees de (x, t).

2) Montrer que la variation relative de volume de la tranche est


V
= . (1)
V x

3) Levolution de la tranche de fluide consideree est supposee adiabatique.

a) Justifier bri`evement cette hypoth`ese.

b) Le coefficient de compressibilite adiabatique dun fluide est defini par


 
1 V
S = (2)
V P S
o`
u V est le volume du fluide et P sa pression ; lindice S indique que la derivee est calculee `a entro-
pie constante (cest-`
a-dire pour une transformation adiabatique) ; pour le fluide contenu dans le tuyau
cylindrique, on supposera que S est une constante.
Montrer que lon peut ecrire
1
p= . (3)
S x


4) Ecrire les forces de pression qui agissent sur la tranche consideree, `a linstant t,
sur le plan x + (x, t)
sur le plan x + dx + (x + dx, t)
et en deduire que la resultante dF des forces qui agissent sur la tranche consideree, mesuree suivant Ox,
a` linstant t, est donnee par
p
dF = Sdx. (4)
x

5) En raisonnant sur la tranche de fluide consideree, etablir, en precisant la loi utilisee, que
2 p
0 2
= . (5)
t x
42 4. EXAMEN (2E SESSION) 2002-2003

6) a) En utilisant les resultats des questions 3) et 5) etablir lequation `a laquelle satisfait la grandeur
(x,t).

b) Quelle est la solution generale de cette equation?

c) Montrer que lon peut interpreter chacun des deux termes intervenant dans la reponse b) comme
des ondes progressives, se propageant `a la vitesse du son c.
Donner lexpression de c.

d) Ces ondes sont-elles des ondes planes?

7) Montrer que les grandeurs p(x,t) et v(x,t) satisfont `a la meme equation de propagation que (x,t).

8) Le fluide est de lair considere comme un gaz parfait :


CP
de = = 1,40 (rapport des capacites calorifiques molaires `a pression et `a volume constant) ;
CV
de masse molaire M = 29 g mol1 ;
de temperature T0 = 293 K.
R = 8,32 J mol1 K1 (constante molaire des gaz parfaits).

Etablir lexpression de c en fonction de , R, T0 et M .


Application numerique : calculer c.

9) On consid`ere la propagation dans le fluide dune onde plane progressive sinusodale de pulsation
quon represente en notation complexe par

1 (x,t) = A1 ei(tkx) (6)

o`
u A1 est une constante et k un reel positif (module du vecteur donde).
On supposera dans cette question que le tuyau est infini et donc quil ny a aucune onde reflechie se
superposant `a 1 .

a) Dans quel sens se propage cette onde?


Determiner lexpression de k en fonction de et c, et calculer sa valeur pour lair avec une frequence de
londe de 1 kHz.

b) Exprimer alors p1 (x, t) et v1 (x, t), representations complexes de la surpression et de la vitesse.

c) On appelle impedance acoustique Z, la grandeur caracteristique du milieu definie par

Z = 0 c. (7)

Montrer que le rapport p1 /


v1 sexprime simplement en fonction de Z.

10) Soit londe 1 = A1 ei(t+kx) , p1 et v1 etant les ondes de surpression et de vitesse associees.
v1 en fonction de limpedance acoustique Z du milieu.
Exprimer le rapport p1 /

II. R
eflexion et transmission dans un tuyau de section constante contenant
deux fluides
Le tuyau est maintenant separe en deux regions (fig. 5).
La region (1) (x < 0) contient un fluide (1) dimpedance acoustique Z1 = 1 c1 . La vitesse du son
dans ce fluide est c1 .
43

fluide (1) fluide (2)

x O x

Fig. 5: Le tuyau contenant deux fluides.

La region (2) (x > 0) contient un fluide (2) dimpedance acoustique Z2 = 2 c2 . La vitesse du son
dans ce fluide est c2 .
La surface de contact entre les deux fluides est donc le plan perpendiculaire en O `a laxe x x.
Une onde acoustique plane sinusodale se propage du milieu (1) vers le milieu (2) et est decrite en notation
complexe par
p1 = p01 ei(tk1 x) (onde de surpression damplitude p01 ). (8)

A linterface entre les deux milieux, cette onde incidente donne naissance `a une onde reflechie dans le
milieu (1), notee p1 , et `
a une onde transmise dans le milieu (2), notee p2 .
On admettra que les ondes reflechie et transmise sont des ondes planes sinusodales damplitudes respec-
tives p01 et p02 .
On precise que dans le plan x = 0 il y a continuite de la pression (donc de la surpression p) et du debit
volumique Sv (cest-`a-dire ici continuite de la grandeur v car la section S du tuyau est constante).

1) Montrer que les ondes reflechie et transmise sont de meme pulsation que londe incidente.

2) Exprimer p1 et p2 en termes de p01 , p02 , , k1 , k2 (que lon definira avec et c2 ), t et x.

3) En exploitant les conditions de continuite, exprimer en fonction de Z1 et Z2 les coefficients de


reflexion r12 et de transmission t12 relatifs aux amplitudes des surpressions. Ces coefficients sont definis
par
p p02
r12 = 01 et t12 = . (9)
p01 p01

4) Coefficients de reflexion et de transmission en puissance.


On peut calculer pour chacune des trois ondes une puissance acoustique transportee, definie par P =
|pv|S, qui est flux de lintensite acoustique I~ = p~v `a travers la section S du tuyau.

a) Determiner les coefficients de reflexion R et de transmission T , relatifs aux puissances acoustiques.

b) Quelle remarque peut-on faire au sujet de R et T ?

5) Application numerique.
Le milieu (1) est lair, le milieu (2) est leau. On prendra respectivement

Z1 = 4,50 102 uSI et Z2 = 1,40 106 uSI.

Determiner numeriquement R et T . Commenter.

III. R
eflexion et transmission dues `
a un changement de section dans un tuyau
sonore (Partie non pos ee `
a lexamen)
Soit un tuyau sonore compose de deux parties cylindriques de meme axe x x, de sections respectives S1
et S2 , raccordees par la surface perpendiculaire en O `a laxe x x.
44 4. EXAMEN (2E SESSION) 2002-2003

fluide (1) fluide (2)

x O x

Fig. 6: Le tuyau avec changement de section.

Les deux parties cylindriques sont remplies respectivement des fluides (1) et (2) (cf. figure 6).
Par definition, on appellera impedance caracteristique du milieu i, la grandeur i = Zi /Si , rapport de
limpedance acoustique du milieu par la section correspondante.
On consid`ere une onde plane acoustique incidente p1 (representation complexe) se propageant dans le
milieu (1) dans le sens des x positifs. Elle donne naissance `a une onde reflechie p1 et `a une onde transmise
p2 , `a linterface entre les deux milieux.

1) En ecrivant les conditions de continuite, determiner, en fonction de 1 et 2 , les coefficients de


reflexion r12 et de transmission t12 , relatifs aux amplitudes des surpressions.

2) Determiner les coefficients de reflexion R et de transmission T , relatifs aux puissances acoustiques.

3) En supposant que les fluides (1) et (2) sont identiques, y a-t-il reflexion `a linterface entre les deux
milieux?
Si oui, calculer r12 et t12 et preciser sil y a changement de phase.

4) Dans le cas o`
u les deux fluides sont de nature differente, retrouve-t-on les resultats du II si S1 = S2 ?
45

C. Induction
Un solenode de longueur b tr`es grande devant son rayon a comporte n spires circulaires jointives par
unite de longueur. Il est parcouru par un courant continu dintensite I. On designe par ~u un vecteur
unitaire parall`ele `
a laxe du solenode.

1) On supposera que le solenode est de longueur infinie (b = ) pour repondre `a cette question. On
designe par P un point quelconque de lespace.
~ ) au point P est parall`ele `a ~u.
a) Montrer que le champ magnetique B(P

~ ) ne depend que de la distance de P `a laxe du solenode.


b) Montrer que B(P

c) Montrer que le champ magnetique est uniforme `a linterieur du solenode.

d) Justifier que le champ magnetique est nul `a lexterieur du solenode.

e) Determiner le champ magnetique `a linterieur du solenode en fonction des donnees de lenonce.

f ) Quel est le coefficient dauto-induction par unite de longueur du solenode?

2) Une bobine de section circulaire de rayon r (r < a), comportant N tours de fil, est placee `a linterieur
du solenode. De plus, les axes de la bobine et du solenode sont confondus.
Quel est le flux de B ~ `a travers la bobine?

En deduire la valeur du rapport M = . Quelle est le nom de la grandeur M ?
I
3) Le solenode est `
a present parcouru par un courant I(t) sinusodal, de frequence = /2 et
damplitude I0 :
I(t) = I0 sin t. (10)

~ a` travers la bobine en fonction du temps?


a) Quelle est lexpression du flux de B

b) En deduire la f.e.m. dinduction e dans la bobine.

c) On relie les extremites de la bobine. On forme ainsi un circuit de resistance electrique R et dauto-
induction negligeable. Determiner le courant Ib qui circule dans la bobine.

d) Quel est le champ electrique Ei induit `a linterieur du fil de la bobine?

4) Proposer une methode pour mesurer la grandeur M .


46 4. EXAMEN (2E SESSION) 2002-2003

Corrig
e

A. Ondes

1) Une onde est une perturbation (oscillation, deformation) qui se propage dans le milieu ou bien une
perturbation qui se maintient dans le milieu (onde stationnaire).

2) Une onde acoustique (ou sonore) est un type dondes qui se propage dans un milieu materiel
elastique (lair, mais aussi leau, les murs, la terre, . . . ). La perturbation du milieu est formee dun petit
deplacement ou dune petite deformation de la mati`ere.
Londe cree une sensation auditive si sa frequence est comprise entre 20 Hz (grave) et 20 000 Hz (aigu)
et si son intensite est suffisante (environ 0 dB `a 1000 Hz).
La frequence caracterise la hauteur dun son pur : plus la frequence est grande et plus le son est haut
(aigu).

3) On suppose que le signal ~s (la perturbation) a un caract`ere vectoriel (deplacement, champ electri-
que). Soit ~u la direction de propagation.
On dit que londe est transversale si ~s ~u. Exemples : ondes dune corde vibrante, ondes electromagne-
tiques.
On dit que londe est longitudinale si ~s k ~u. Exemple : ondes sonores dans lair.

4) Definition 1. On appelle surface donde ` a un instant donne t une surface o`u le signal prend la meme
valeur. Cette definition est acceptable si le signal est scalaire (surpression dune onde sonore).
Definition 2. Pour une onde monochromatique et un signal scalaire ou vectoriel (representation complexe)
r )ei(t+(~r)) ,
s(~r, t) = A(~

avec A(~r) variant lentement en ~r par comparaison avec (~r), on appelle surface donde une surface
equiphase (~r) = Cte. Cette definition est acceptable pour les ondes electromagnetiques (approximation
de loptique geometrique).

B. Propagation dondes acoustiques

I. Propagation dune onde acoustique dans un tuyau de section constante


contenant un fluide unique

1)  
   
dx = x + dx + (x + dx, t) x + (x, t) = 1+ dx. (11)
x

2) V = S dx, V = S (dx dx) donnent avec (11)

V
= . (12)
V x

3.a) Les compressions et dilatations du fluide sont si rapides (au moins 20 Hz pour les ondes audibles)
quil ny a pas dechange de chaleur avec le reste du fluide. Levolution de la tranche de fluide est alors
adiabatique.
47

Remarque. Supposer une evolution isotherme conduirait `a une valeur de la vitesse du son en desaccord
avec lexperience (cf. corrige de la question 8).

3.b) Le volume de la tranche varie de V et sa pression de p. Ces grandeurs etant du premier ordre et
la transformation adiabatique, on a  
V V
= . (13)
P S p
Lequation (2) donne alors
V 1 V
S = soit p= . (14)
pV S V
Do`
u, avec lequation (12),
1
p= . (15)
S x
Noter le signe negatif dans (15) qui correspond au fait que le volume diminue quand la pression augmente
(S > 0).

4) Les forces de pression, mesurees suivant Ox, qui agissent sur la tranche consideree, `a linstant t,
sont :
sur la partie gauche  
Fg = P0 + p x + (x, t) S ; (16)
sur la partie droite  
Fd = P0 + p x + dx + (x + dx, t) S ; (17)
en tout
h  i
dF = Fd + Fg = p x + dx + (x + dx, t) p x + (x, t) S
 
p h i p p
= dx + (x + dx, t) (x, t) S = 1+ Sdx = Sdx (18)
x x x x

en negligeant le terme du 1er ordre par rapport `a 1.
x
Noter le signe negatif dans (18) qui correspond au fait que la force est negative quand la pression
augmente avec x.

5) On applique le principe fondamental de la dynamique (la deuxi`eme loi de Newton) `a la tranche de


fluide consideree. La masse de la tranche est dm = 0 Sdx. Son acceleration suivant Ox est `a linstant t
2 2
= 2 (x + (x, t), t) = 2 (x, t), en negligeant des termes dordres superieurs. Lequation dm = dF
t t
secrit
2 p
0 2 Sdx = Sdx (19)
t x
do`u la relation demandee en otant le facteur Sdx.

6) a) On porte p donne par lequation (3) dans lequation (5)

2 p 1 2
0 = = . (20)
t2 x S x2
La grandeur (x,t) satisfait `
a lequation de dAlembert

1 2 2
=0 (21)
c2 t2 x2
o`
u
1
c= . (22)
0 S
48 4. EXAMEN (2E SESSION) 2002-2003

b) La solution generale de cette equation est (theor`eme de dAlembert, 1747)


 x  x
(x, t) = F t +G t+ . (23)
c c

c) La solution F (t x/c) est une onde progressive se propageant `a la vitesse c dans le sens de x
croissant. La solution G (t + x/c) est une onde progressive se propageant `a la vitesse c dans le sens
oppose. La vitesse du son c est donnee par lequation (22).

d) Ce sont des ondes planes : le deplacement (x, t) est le meme en tout point dun plan x = Cte.

7) Derivons lequation (21) par rapport ` a x et divisons par S :


   
1 2 2 1 1 2p 2p
2 2
2
=0 soit 2 =0 (24)
c t x S x c2 t2 x
1
puisque p = dapr`es lequation (3).
S x
Derivons lequation (21) par rapport ` at
 
1 2 2 1 2v 2v
=0 soit =0 (25)
c2 t2 x2 t c2 t2 x2

puisque v = .
t
Les grandeurs p(x,t) et v(x,t) satisfont donc `
a la meme equation de propagation que (x,t).

8) Pour un gaz parfait, lequation detat est lequation de Boyle-Mariotte


P RT
= . (26)
M
Lequation qui caracterise la transformation adiabatique peut secrire

P V = Cte. (27)
dP dV
En la differentiant logarithmiquement on obtient + = 0. En posant P P0 , dP p et
P V
p V
dV V , il vient = . Pour un gaz parfait on a donc, en comparant avec lequation (14),
P0 V
1
S = . (28)
P0
En utilisant le coefficient de compressibilite adiabatique (28) et lequation detat (26) on obtient pour la
vitesse du son dun gaz parfait
r s r
1 P0 RT0
c= = = . (29)
0 S 0 M

Pour lair `a la temperature T0 = 293 K on a c = 343 m s1 en tr`es bon accord avec la valeur experimentale.
Si on avait suppose une evolution isotherme au lieu dadiabatique (cf. remarque du corrige de la question
3.a)), cela donnerait c = 290 m s1 (cette valeur sobtient en faisant = 1 dans (29)).

9.a) Le deplacement 1 (x,t) est une fonction de la forme f ([t kx/]) = F (t x/c), pour = kc.
On en deduit que londe se propage vers x croissant et que

2 103
k= = = 18,3 m1 . (30)
c 343
49

9.b)
1 1 ik A1 i(tkx)
p1 (x, t) = = e . (31)
S x S
1
v1 (x, t) = = i A1 ei(tkx) . (32)
t

9.c) Le rapport des deux derni`eres equations donne

p1 k 1
= = = 0 c = Z (33)
v1 S cS

en utilisant S = 1/0 c2 (cf. lequation (22)).

10) Au lieu des equations (31) et (32), on obtient, pour cette onde qui se propage vers x decroissant,

ik A1 i(t+kx)
p1 (x, t) = e et v1 (x, t) = i A1 ei(t+kx) . (34)
S
Le rapport demande est loppose de limpedance acoustique :

p1 k
= = 0 c = Z. (35)
v1 S

II. R
eflexion et transmission dans un tuyau de section constante contenant
deux fluides

1) Lenonce nous demande dadmettre que les ondes reflechie et transmise sont des ondes planes
sinusodales damplitudes respectives p01 et p02 . Nous notons et leurs pulsations respectives :


p1 = p01 ei( t+k1 x) onde reflechie avec k1 = ; (36)
c1

p2 = p02 ei( tk2 x)
onde transmise avec k2 = ; (37)
c2
p1 + p1 )x=0 dans le milieu 1 et (
La surpression en x = 0 est ( p2 )x=0 dans le milieu 2. Sa continuite en
x = 0 secrit
p01 eit + p01 ei t = p02 ei t . (38)
Cette relation doit etre vraie t ce qui implique que = = et p01 + p01 = p02 . Les ondes reflechie
et transmise sont donc de la meme pulsation que londe incidente.

2) On fait = = et k1 = k1 dans les equations (36) et (37) :



p1 = p01 ei(t+k1 x) onde reflechie avec k1 = ; (39)
c1

p2 = p02 ei(tk2 x) onde transmise avec k2 = ; (40)
c2
 
p1 p
3) Dapr`es les equations (33) et (35), la vitesse v en x = 0 est 1 dans le milieu 1 et
  Z1 Z1 x=0
p2
dans le milieu 2. Sa continuite en x = 0 secrit
Z2 x=0

p01 p p02
01 = . (41)
Z1 Z1 Z2
50 4. EXAMEN (2E SESSION) 2002-2003

Avec la continuite de la pression, on a le syst`eme lineaire de 2 equations `a 2 inconnues (p01 et p02 ) :



p01 + p02 = p01
(42)
Z2 p01 + Z1 p02 = Z2 p01 .
En divisant par p01 : 
r12 + t12 = 1
(43)
Z2 r12 + Z1 t12 = Z2
de solution
Z2 Z1 2Z2
r12 = , t12 = . (44)
Z2 + Z1 Z2 + Z1

4. a) Les puissances acoustiques transportees des ondes incidente, reflechie et transmise sont respecti-
vement
p2 S p2 S p2 S
P1 = p1 v1 S = 1 , P1 = 1 et P2 = 2 . (45)
Z1 Z1 Z2
Les coefficients de reflexion et de transmission en puissance sont respectivement
 2
P p2 Z2 Z1
R = 1 = 12 = r12 2
= (46)
P1 p1 Z2 + Z1
et
P2 p2 Z1 Z1 4Z1 Z2
T = = 22 = t212 = . (47)
P1 p1 Z2 Z2 (Z2 + Z1 )2

4. b) La somme des equations (46) et (47) donne R + T = 1. Il y a conservation de lenergie acoustique


dans un syst`eme o`
u on neglige les pertes (par viscosite, son qui traverse le tuyau, . . . ).

5) Comme Z1 Z2 , on a
4Z1 Z2 4Z1 Z2 4Z1
T = 2 = = 1,3 103 (48)
(Z2 + Z1 ) Z22 Z2
et R = 0,999. La reflexion est pratiquement totale, comme chaque fois que les impedances des deux
milieux sont dordres de grandeurs differents. Il ny a pas adaptation dimpedance et seule une fraction
tr`es petite de lenergie est transmise.

III. R
eflexion et transmission dues `
a un changement de section dans un tuyau
sonore

1) On doit ecrire la continuite du debit Sv. Au lieu de lequation (41) on a


S1 p01 S1 p01 S2 p02
= (49)
Z1 Z1 Z2
qui se recrit en fonction de 1 et 2 :
p01 p p02
01 = . (50)
1 1 2
Le syst`eme `a resoudre est le meme que le syst`eme (42) apr`es les changements Z1 1 et Z2 2 . On
a donc, sans nouveaux calculs :
2 1 22
r12 = , t12 = . (51)
2 + 1 2 + 1

2) Au lieu de (45) on a pour les puissances acoustiques transportees des ondes incidente, reflechie et
transmise :
p 2 S1 p2 p2 p 2 S2 p2
P1 = p1 v1 S1 = 1 = 1, P1 = 1 et P2 = 2 = 2. (52)
Z1 1 1 Z2 2
51

et les coefficients de reflexion et de transmission en puissance sont respectivement


 2
2 1 41 2
R= et T = 2. (53)
2 + 1 (2 + 1 )

3) Pour des fluides (1) et (2) identiques on a


S1 S2 2S1
r12 = , t12 = . (54)
S1 + S2 S1 + S2
Il y a donc reflexion `
a linterface entre les deux milieux (r12 6= 0).
Il y a changement de phase de pour S1 < S2 (r12 < 0).

4) Si S1 = S2 , on retrouve les resultats du II : Lequation (51) redonne bien (44) et lequation (53)
redonne bien (46) et (47).

C. Induction

1.a) Tout plan perpendiculaire ` ~ k ~u.


a ~u est plan de symetrie (pour b = ). Donc B

b) Le syst`eme est invariant par translation parall`ele `a ~u et par rotation autour de laxe du solenode.
~ ) = B(P )~u ne depend que de la distance de P `a laxe du solenode.
Donc B(P

c) Appliquons le theor`eme dAmp`ere au pourtour du rectangle P QRS. On prend P Q k ~u et de


 
longueur d. La circulation de B ~ le long de ce pourtour est d B(P ) B(R) . Pour P et R `a linterieur
du solenode, le courant `a travers le rectangle est nul et B(P ) = B(R). Le champ magnetique est donc
uniforme ` a linterieur du solenode.

d) On montre de meme que le champ magnetique est uniforme `a lexterieur du solenode. Si on


seloigne du solenode jusqu` ~ tend vers 0. On suppose que
a linfini, pour un solenode fini, le champ B
~
cela reste vrai pour le solenode infini. Comme B = Cte `a lexterieur du solenode cette constante est
nulle et B~ =0` a lexterieur du solenode.

e) Orientons ~u et I selon la r`egle du tire-bouchon et soit B~u le champ magnetique `a linterieur du


solenode. Le rectangle P QRS traversant le solenode, P `a linterieur du solenode, S `a lexterieur du
solenode, le theor`eme dAmp`ere donne Bd = 0 nId, do`u B = 0 nI et B ~ = 0 nI~u.

f ) Le flux du champ magnetique `a travers le solenode est s = Ba2 nb = 0 n2 a2 bI. Le coefficient


dauto-induction du solenode est L = s /I = 0 n2 a2 b. Le coefficient dauto-induction par unite de
longueur du solenode est = L/b = 0 n2 a2 .
~ a
2) Le flux de B ` travers la bobine est
= Br2 N = 0 nr2 N I. (55)
Le signe est si les sens denroulements du solenode et de la bobine sont opposes.

Le rapport M = = 0 nN r2 est le coefficient de mutuelle-induction solenode-bobine.
I
~ a
3.a) Le flux de B ` travers la bobine est
= M I0 sin t = 0 nN r2 I0 sin t. (56)

b) La f.e.m. dinduction dans la bobine est


d
e= = M I0 cos t = 0 nN r2 I0 cos t. (57)
dt
52 4. EXAMEN (2E SESSION) 2002-2003

c) Le courant qui circule dans la bobine est

e M I0 0 nN r2 I0
Ib = = cos t = cos t. (58)
R R R
Cest un courant sinusodal damplitude
|M |I0
Ib0 = (59)
R

d) La circulation du champ electrique induit Ei le long du fil de la bobine est la f.e.m. e. Linvariance
du syst`eme par rotation autour de son axe implique que Ei est le meme en tout point du fil. On a donc
e = 2rN Ei et
e 0 nrI0
Ei = = cos t. (60)
2rN 2

4) Envoyer un courant I(t) de la forme (10) dans le solenode et mesurer son amplitude I0 et sa pulsation
. Mesurer lamplitude Ib0 du courant Ib (t) qui circule dans la bobine. Dapr`es (59), le coefficient de
mutuelle-induction solenode-bobine sobtient par
RIb0
|M | = . (61)
I0
On a M > 0 (resp. M < 0) si I(t) et Ib (t) sont en opposition de phase (resp. en phase).
53

5. Premier partiel 2003-2004


Enonc
e
13 octobre 2003
duree : 2 heures
sans documents, calculatrices autorisees

Le bar`eme (sur 60 points) indique le poids respectif de chaque partie.


Les parties A, B, C, D, E et F sont independantes.

A. Onde progressive (4 points)


Une corde infiniment longue est tendue le long de laxe
x Ox. Une onde transverse progressive se propage sur cette y
corde sans se deformer vers les x croissants. La forme de S
la corde `a linstant t = 0 est donnee par la fonction
2
y = aex (1)

o`u a et sont des constantes (cf. figure 7).


On appelle sommet de londe le point de la corde dont x 0 x0 x
le deplacement par rapport ` a sa position dequilibre est
maximal. Le sommet de londe ` a linstant t = 0 est en S Fig. 7 Corde a
` linstant t = 0.
dabscisse x = 0.

1) Representer lallure de la corde `


a linstant t0 o`
u le sommet de londe atteint le point dabscisse x = x0 .
On reproduira sur la meme figure lallure de la corde `a linstant t = 0.

2) Quelle est `
a linstant t0 , en termes de x, a, et x0 , la fonction y = f (x) qui decrit la forme de la
corde ?

B. Ondes progressives en sens oppos


es (4 points)
25 cm 25 cm
y

2 cm

x A 0 B x
10 cm 10 cm 15 cm 15 cm 10 cm 10 cm
54 5. PREMIER PARTIEL 2003-2004

Fig. 8 Corde a
` linstant t = 0.

Une corde infiniment longue est tendue le long de laxe x Ox. Deux ondes transverses progressives se
propagent en sens opposes sur la corde.
` linstant t = 0, les deux ondes ont une forme triangulaire et se dirigent lune vers lautre (cf. figure 8).
A
La vitesse de propagation de ces ondes est c = 10 m s1 .
Representer lallure de la corde `
a linstant t = 0,02 s. On marquera sur la figure les points A et B de la
corde (dabscisses 15 cm) ainsi que suffisamment dindications (cotes) pour determiner compl`etement
la forme de la corde.

C. Reconnatre les ondes (4 points)


Les expressions suivantes represente-t-elles des ondes solutions de lequation de dAlembert avec une
vitesse de propagation des ondes egale `
a c ? On justifiera les reponses.
Nota : 1 point par reponse juste et justifiee ; 0 point pour une reponse juste sans justification.

p
(a) s1 (x,t) = 3 sin (3t 3x/c),

(b) s2 (x,t) = 5 x2 2cxt + c2 t2 ,
(c) s3 (x,t) = x2 c2 t2 ,
(d) s4 (x,t) = sin(2t) 2 cos2 (x/c) sin(2t).

D. Battements (4 points)
On rappelle que lintervalle entre les frequences 1 et 2 sexprime en savarts par
2
isav = 1000 log10 . (2)
1
Deux frequences 1 et 2 voisines de 200 Hz produisent un battement de frequence 2 Hz. Determiner
lintervalle en savarts entre ces deux frequences.

E. Corde sol dun violon (20 points)

La corde sol dun violon est assimilee `


a une corde vibrante homog`ene de
masse lineique = 2,5 g m1 fixee aux points A (sur le chevalet) et D cheville
(sur le sillet). On schematise la corde au repos par le segment de droite D
touche
AD de longueur L = AD = DA = 30 cm. La tension T0 de la corde M
est ajustable `a laide dune cheville. Le violon est suppose accorde : la
frequence fondamentale de la corde AD est

1 = 196 Hz (note sol2 ).

On rappelle que la vitesse des ondes transversales de la corde est


s A
T0
c= .

1) Donner lexpression theorique de 1 en fonction de c et L.


55

Quelle est la frequence n du mode propre n (n Z+ ) de vibration de la corde AD ?


Dessiner laspect de la corde AD lorsquelle vibre dans le mode propre n = 3.

2) Calculer la tension T0 de la corde.

3) Le violoniste desire que la note sol2 jouee par la corde AD soit juste `a 5 savarts pr`es. Avec quelle
precision (absolue et relative) doit-il regler la tension T0 ?

4) Par la pression du doigt au point M du segment AD, le violoniste appuie en M la corde contre la
touche (pi`ece en eb`ene du violon). Il obtient ainsi une corde vibrante plus courte, la corde AM , qui peut
emettre une nouvelle note. On supposera que cette corde AM est fixee en A et M et que sa tension reste
egale `a T0 .
Determiner le point B du segment de droite AD tel que la note emise par la corde AB (quand le
51
violoniste appuie son doigt en B) soit la note si2 (de frequence ). On calculera la valeur numerique
4
de la longueur AB.

5) Determiner de meme le point C du segment de droite AD tel que la note emise par la corde AC soit
31
la note re3 (de frequence ).
2
Tracer une figure representant les quatre points A, B, C et D.
Nota. Les trois notes des cordes AD, AB et AC, dont les frequences fondamentales sont proportionnelles
5 3
`a 1, et respectivement, forment lorsquelles sont jouees simultanement un accord parfait majeur.
4 2

6) Pour que la note re3 soit jouee juste `a 5 savarts pr`es, le violoniste doit poser son doigt sur la corde
`a une distance inferieure `
a x du point C. Determiner la valeur numerique de la longueur x.

7) Lorsque le violoniste appuie son doigt en B, il ne fait pas vibrer habituellement la portion BD de la
corde (il donne un coup darchet sur la portion AB de la corde). Quelle est la frequence de la note quil
obtiendrait sil faisait vibrer cette corde BD ? Pour ce calcul on supposera que la corde BD est fixee en
B et D et que sa tension est T0 . On donnera la frequence en fonction de 1 .
Reprendre la meme question pour le point C et la portion CD.

8) On dit que des nombres forment une progression harmonique si leurs inverses forment une progression
1 1 1 1
arithmetique. Exemple : , , , .
6 11 16 21
Les longueurs AD, AB et AC forment-elles une progression harmonique?
Les longueurs AD, CD et BD forment-elles une progression harmonique?
CA DA
Calculer le birapport H(AB, CD) = : et commenter sa valeur.
CB DB

F. Vibration forc
ee dune corde (24 points)
On etudie les vibrations transverses dune corde vibrante homog`ene de masse lineique = 1 g m1 et de
longueur L = 30 cm. La corde est tendue avec la tension constante T0 = 14,4 N entre les points fixes A
(L/2, 0, 0) et B (L/2, 0, 0) o`u la notation (x, y, z) designe les coordonnees dans un syst`eme cartesien
Oxyz.
56 5. PREMIER PARTIEL 2003-2004

En plus des forces de tension, la corde est soumise `a des forces exterieures reparties sur toute la longueur
de la corde. Le mouvement de la corde seffectue dans le plan Oxy et est decrit par le deplacement y(x, t) :
le point (x, 0, 0) de la corde au repos se deplace en (x, y(x, t), 0) `a linstant t. La fonction y(x, t) verifie
lequation differentielle
2y 2y
2 = T0 2 + F (x, t) (3)
t x
s
T0
o`u la fonction F (x, t) de x et de t est une donnee du probl`eme. On posera c = .

1) Quelle est la signification physique de lequation (3) ? Quelle est la signification physique des trois
2y 2y
termes 2 , T0 2 et F (x, t) de cette equation ? En quelle unite la fonction F (x, t) est-elle mesuree?
t x
Lorsque F (x, t) = 0 pour tout x et tout t on dira que la corde est libre. Quelle est la valeur numerique
de la frequence fondamentale 1 de la corde libre ?

2) On impose les conditions aux limites

y(L/2, t) = y(L/2, t) = 0 t. (4)

Comment procedez-vous experimentalement pour imposer ces conditions?

3) On suppose dans la suite que la fonction F (x, t) est independante de x et donnee par

F (x, t) = A0 cos(t). (5)


c
On posera = et = .
2
Decrire un dispositif experimental dans lequel le mouvement de la corde verifie lequation (3), la fonction
F (x, t) etant donnee par lexpression (5).
Comment lexperimentateur peut-il faire varier la frequence dans ce dispositif ?

4) On cherche une solution particuli`ere des equations (3) et (4) sous la forme

y(x, t) = b(x) cos(t). (6)

Ecrire lequation differentielle et les conditions aux limites verifiees par b(x). On ecrira ces equations en
A0
termes de L, k = et B0 = 2 .
c k T0

5) Montrer que, sauf pour certaines valeurs de ( = 1 , = 2 , . . . ), il existe une ou plusieurs solutions
b(x) de ces equations.
Lorsque 6= 1 et 6= 2 et . . . determiner cette solution b(x) (ou une de ces solutions b(x) sil y en a
plusieurs) et lecrire en termes de L, k et B0 .
Exprimer les valeurs 1 , 2 , . . . de pour lesquelles il ny a pas de solution b(x) en fonction de la
frequence fondamentale 1 de la corde libre.


6) Etude de la fonction b(x).
On repondra aux questions ci-dessous en termes de L, et B0 . On supposera B0 > 0 et on se placera `a
une frequence o`
u la solution b(x) est unique.
57

Etudier les maximums et minimums de la fonction b(x) (valeurs et positions).


Pour quelles valeurs de x la fonction b(x) sannule-t-elle ?
Tracer la courbe de b(x) en fonction de x pour L/2 x L/2 (L = 30 cm) lorsque = 16 cm.

7) Representer laspect visuel de la corde lorsque la frequence correspond `a la longueur donde =


16 cm. On supposera que le deplacement est donne par la solution particuli`ere determinee ci-dessus.

8) Que se passe-t-il si la frequence est = 200 Hz ?

9) Que se passe-t-il si la frequence est = 400 Hz ?

10) Quelle est la solution generale de lequation (3) ? On se limitera aux cas o`
u b(x) existe (cf. question
5).
Laspect visuel de la corde, pour = 16 cm, est-il celui determine `a la question 7 ?
58 5. PREMIER PARTIEL 2003-2004

Corrig
e

A. Onde progressive

1) y

Fig. 9 Allure de la corde lorsque le sommet atteint labs-


cisse x = x0 .

2) La courbe est la translatee de x0 de la courbe `a t = 0 :


x 0 x0 x
(xx0 )2
y = ae .

B. Ondes progressives en sens oppos


es
` linstant t = 0,02 s, la forme de la corde resulte de la superposition de deux ondes
A
y

2 cm

x A 0 B x
10 cm 10 cm

et
y

2 cm

x A 0 B x
10 cm 10 cm

Cest
y

2 cm

x A 0 B x
10 cm 10 cm 10 cm

C. Reconnatre les ondes


Pour montrer que s(x, t) est solution de de lequation dondes de dAlembert avec une vitesse de propa-
gation des ondes egale `
a c, on peut
soit verifier directement lequation
2s 1 2s
= 0; (7)
x2 c2 t2
59

soit utiliser le theor`eme de dAlembert et verifier que s(x, t) est de la forme

s(x, t) = F (t x/c) + G(t + x/c) (8)

o`
u F (u) et G(v) sont des fonctions arbitraires (on peut avoir G 0).

p
a) Oui. s1 (x,t) = F (t x/c) avec F (u) = 3
sin (3u).

b) Oui. s2 (x,t) = 5(x ct)2 = F (t x/c) avec F (u) = 5c2 u2 .

2 s3 2 s3 2 2 s3 1 2 s3
c) Non. On calcule = 2, = 2c et = 4. Cela montre que s3 nest pas une
x2 t2 x2 c2 t2
solution de lequation (7).

 
d) Oui. s4 (x,t) = 1 2 cos2 (x/c) sin(2t) = cos(2x/c) sin(2t) qui est une onde stationnaire de
pulsation 2. (Il a ete montre en cours et TD que londe stationnaire A sin(t+) sin(kx+) est solution
de lequation (7) pour = kc.)

D. Battements
|1 2 | = 2 Hz ; 1 /2 = 1 + (1 2 )/2 1 2/200 ; isav = 4,3 savarts 1 .

E. Corde sol dun violon

c
1) La frequence fondamentale est 1 = .
2L
La frequence du mode propre n de vibration de la corde
nc
est n = n1 = . A D
2L
s
c 1 T0
2) La frequence fondamentale est 1 = = . Fig. 10 Aspect de la corde dans le
2L 2L
Do`
u mode propre n = 3.
T0 = (2L1 )2 = 34,6 N. (9)

3) Un intervalle de 5 savarts correspond `a un intervalle i = /1 = 100,005 . Pour le signe +, =


1,0115795 1 et 1 = | 1 | = 2,27 Hz. Pour le signe , = (1 0,0114469)1 et 1 = | 1 | =
2,24 Hz. On ne distinguera pas les deux cas et on prendra 1 /1 = 1,15%. Dapr`es lequation (9)
T0 1
=2 = 2,3%. Pour lincertitude absolue on a T0 = 0,8 N.
T0 1

4) Designons par AM la frequence fondamentale de la note emise par la corde AM .


Dapr`es lequation (9),
Cte
AM = . (10)
AM
AB AD 4
On en deduit = = et AB = 24 cm.
AD AB 5
1. Felix Savart (1791-1841)
60 5. PREMIER PARTIEL 2003-2004

AC AD 2
5) De meme = = et AC = 20 cm.
AD AC 3
A C B D

x
6) Dapr`es lequation (10), la variation x produit une variation de frequence AC telle que =
AC
AC
= 1,15%. On en tire x = 2,3 mm.
AC

7) Les longueurs des cordes BD et CD sont respectivement 6 et 10 cm.


Dapr`es lequation (10), les frequences fondamentales de ces cordes sont respectivement 51 = 980 Hz et
31 = 588 Hz.
Remarque. Il y a un intervalle dune tierce (i = 5/4) entre les frequences 51 et 41 (sol4 ). La note emise
par la corde BD est donc si4 .
Il y a un intervalle dune quinte (i = 3/2) entre les frequences 31 et 21 (sol3 ). La note emise par la
corde CD est donc re4.

8) Les longueurs de trois cordes forment une progression harmonique si leurs frequences fondamen-
tales (10) forment une progression arithmetique. Cest le cas des trois cordes AD, AB et AC (frequences
1 1 1
4 ,5 et 6 ). Cest aussi le cas des trois cordes AD, CD et BD (frequences 1 , 31 et 51 ).
4 4 4
Le birapport vaut H(AB, CD) = 1. Geometriquement les points A, B, C et D forment une division
harmonique, les points C et D etant conjugues harmoniques par rapport `a A et B et inversement. Lorsque
les points A, B, C et D forment une division harmonique, les longueurs AD, AB et AC forment une
1 1 1 1
progression harmonique. Cela revient ` a dire que = ou encore que AB est la
AC AB AB AD
2 1 1
moyenne harmonique de AD et AC ( = + ).
AB AC AD

F. Vibration forc
ee dune corde

1) Lequation (3) est lequation du mouvement de la corde. Apr`es multiplication par dx~ey lequation (3)
secrit
2y 2y
(dx) 2 ~ey = T0 2 ~ey dx + F (x, t) ~ey dx. (11)
t x
2y
Le premier terme est lacceleration 2 ~ey multipliee par la masse dx de lelement de corde entre x et
t
x + dx. Dapr`es la 2e loi de Newton, il est egal `a la somme des forces qui agissent sur lelement de corde.
2y
Le terme T0 2 ~ey dx est la resultante des forces de tension qui agissent sur lelement de corde (resultat
x
montre en cours) et le terme F (x, t)~ey dx est la resultante des forces exterieures qui agissent sur lelement
de corde.
La fonction F (x, t) est donc la force lineique agissant sur la corde au point x `a linstant t.
Elle est mesuree en N m1 (ou kg s2 ).
Pour la corde libre (sans force exterieure), la vitesse des ondes est c = 120 m s1 et la frequence fonda-
c
mentale est 1 = = 200 Hz.
2L

2) On impose ces conditions en fixant les extremites de la corde aux points A et B.


61

3) Voici deux dispositifs envisageables :

On envoie une onde plane progressive harmonique sonore de surpression p(~r,t) = p0 cos(t + ky)
vers la corde. Les mouvements de la corde etant tr`es petits (|ky| 1), la surpression sur la corde,
du cote y > 0 do` u provient londe sonore, est p0 cos(t) qui est independant de x. Supposant
une surpression nulle sur la corde, du c ote y < 0, la force lineique est presque parall`ele `a ~ey ,
proportionnelle ` a p0 cos(t) et donc de la forme (5). Lexperimentateur fait varier la frequence
dans ce dispositif en modifiant la hauteur (la frequence) de londe sonore.
On peut utiliser une corde conductrice parcourue par un courant alternatif I(t) = I0 cos(t). Si
la corde est placee dans un champ magnetique B ~ 0 uniforme et constant parall`ele `a Oz, la force
 
y
magnetique (force de Laplace) dF sur lelement de corde ~u dl = ~ex + ~ey dx (~u vecteur unitaire
x
y
tangent ` a la corde) est, en utilisant 1,
x
~ 0 B0 I0 cos(t)dx ~ey .
dF = Idl ~u B

La force lineique est de la forme (5) avec F (x, t) = B0 I0 cos(t). Lexperimentateur fait varier la
frequence dans ce dispositif en modifiant la frequence du courant.

4) En portant y(x, t) = b(x) cos(t) dans lequation (3) on obtient

2 b(x) cos(t) = T0 b (x) cos(t) + A0 cos(t). (12)

En simplifiant
2 A0
b (x) + b(x) = (13)
T0 T0
soit
b (x) + k 2 b(x) = k 2 B0 . (14)
En portant y(x, t) = b(x) cos(t) dans lequation (4) on obtient pour les conditions aux limites

b(L/2) cos(t) = b(L/2) cos(t) = 0 t (15)

soit
b(L/2) = b(L/2) = 0. (16)

5) La solution generale de lequation (14) est

b(x) = B0 + sin(kx) + cos(kx) (17)

o`
u et sont des constantes reelles arbitraires. Les conditions aux limites (16) donnent

B0 sin(kL/2) + cos(kL/2) = 0 (18)

soit

sin(kL/2) = 0 (19)
cos(kL/2) = B0 . (20)

Ces equations pour et ont une solution si

cos(kL/2) 6= 0. (21)

Cette solution est unique si sin(kL/2) 6= 0.


62 5. PREMIER PARTIEL 2003-2004

Si sin(kL/2) = 0, est arbitraire et il y a une infinite de solutions.


kL
La condition (21) exprime que = est different de + (n 1) (n Z+ ) soit
2 2 1 2

6= n avec n = (2n 1)1 (n Z+ ) (22)

ce qui signifie que la frequence nest pas un harmonique impair de la frequence fondamentale de la
corde libre. Lorsque la condition (22) est satisfaite, les equations (19) et (20) ont une solution

B0
= 0, = . (23)
cos(kL/2)

On a alors pour b(x) :


 
cos(kx)
b(x) = B0 1 . (24)
cos(kL/2)
Il y a une infinite de solutions lorsque sin(kL/2) = 0 cest-`a-dire lorsque la frequence est un harmonique
pair de la frequence fondamentale de la corde libre :

L
= 2n1 , = (n Z+ ) (25)
n
et toutes les solutions pour b(x) sont donnees par ( arbitraire)

b(x) = B0 ((1)n cos(2nx/L) 1) + sin(2nx/L). (26)

6) 
Soit = 1 le signe de cos(kL/2). La valeur maximum de b(x) correspond `a cos(kx) = ; cest

B0 1 qui est atteint pour kx = 2n (n Z) soit x = n (n Z) si = 1 et pour
cos(kL/2)
kx = (2n + 1) (n Z) soit x = (2n + 1)/2 (n Z) si = 1.
 

La valeur minimum de b(x) correspond ` a cos(kx) = ; cest B0 + 1 qui est atteint
cos(kL/2)
pour x = (2n + 1)/2 (n Z) si = 1 et pour x = n (n Z) si = 1.
La fonction b(x) sannule pour cos(kx) = cos(kL/2) soit pour kx = kL/2 + 2n (n Z). En termes de
cela donne x = L/2 + n.

= 16 cm, = 750 Hz
y 7)
15 10 5 5 10 15

A B

8) La frequence est la frequence fondamentale de la corde libre = 1 . Il ny a pas de solution forcee


de la forme (6). Quand 1 , cos(kL/2) 0 et b(x) . Lamplitude de vibration de londe crot
avec le temps pour cette frequence. Cest le phenom`ene de resonance. Bien s ur, lamplitude maximale de
londe ne devient pas infinie par suite de phenom`enes negliges (non linearites, amortissements).
63

9) Pour = 21 = 400 Hz, il ny a pas de phenom`ene de resonance. Mais il y a une infinite de solutions
forcees b(x). On a = 30 cm et la fonction b(x) est donnee par (26) avec n = 1 :

b(x) = B0 (cos(2x/L) + 1) + sin(2x/L). (27)

Pour = 0, la fonction b(x) et laspect de la corde correspondant sont representes ci-dessous.

15 10 5 5 10 15

A B

Pour 6= 0, la courbe b(x) nest plus paire. La fonction b(x) et laspect de la corde correspondant sont
representes ci-dessous (on a pris = 2B0 ).

15 10 5 5 10 15

A B

10) La solution generale de lequation (3) est la solution particuli`ere (6) plus la solution generale de
lequation homog`ene associee
2y 2y
2 = T0 2 (28)
t x
qui est lequation de dAlembert pour la vitesse c des ondes.
La solution generale de lequation (3) est donc
 
cos(kx)
y(x, t) = B0 1 cos(t) + (t x/c) + (t + x/c) (29)
cos(kL/2)

o`
u (u) et (v) sont des fonctions arbitraires. Pour une corde reelle, par suite des frottements, les
solutions libres et samortissent rapidement. Apr`es une periode transitoire, londe est donnee par la
solution particuli`ere (6) et laspect visuel est bien celui obtenu `a la question 7.
64 `
6. DEUXIEME PARTIEL 2003-2004

6. Deuxi`
eme partiel 2003-2004


Enonc
e
13 novembre 2003
duree : 3 heures
sans documents, calculatrices autorisees

Les parties A, B, C et D sont independantes.


Bar`eme indicatif : presentation et concision = 1,5 ; A = 5 ; B = 3,5 ; C = 5 ; D = 5.

A. Effet Doppler dun sonar


On consid`ere un sonar S constitue dun emetteur et dun recepteur dondes acoustiques. Il se trouve `a
bord dun bateau de peche et est utilise pour detecter les bancs de poissons. Il emet des ondes qui se
propagent dans la mer avec la celerite
c = 1500 m s1 . (1)
Le sonar est immobile et situe en O sous la surface de la mer. Les ondes emises par le sonar S sont reflechies
par un poisson P . Le poisson P se deplace sur le demi-axe Ox `a la vitesse algebrique v constante (v > 0
si P seloigne de O). A ` linstant t = 0 la distance OP est L. Les ondes reflechies par le poisson reviennent
en O o` u elles sont detectees par le sonar.
O P v x
S

1) Le sonar S emet une onde de frequence 0 .

a) Quelle est la frequence 1 du signal recu par le poisson P ?

b) Quelle est la frequence du signal recu par le sonar apr`es reflexion sur le poisson ?

c) On pose = 0 . Montrer que la vitesse v du poisson sexprime en fonction de c, 0 et .


Application numerique. Calculer v pour = 1225 Hz et 0 = 75 kHz.

d) On peut obtenir experimentalement en mesurant et 0 et en calculant leur difference. Decrire


une autre methode qui permet de mesurer avec plus de precision.
65

e) Citer dautres exemples dapplications de leffet Doppler.

2) Le sonar emet deux impulsions sonores tr`es br`eves aux instants t = 0 et t = T . Soient t1 et t2 les
u ces impulsions reviennent en O apr`es reflexion sur le poisson. On pose T = t2 t1 .
temps o`

a) Determiner t1 et t2 en fonction de c, T , v et L.
En deduire une expression de T en fonction de c, v et T et retrouver la reponse de la question 1.b.

b) Determiner v et L en fonction de c, T , t1 et T .
Application numerique. Calculer v et L pour T = 13,3 s, T = 13,1 s et t1 = 0,172 s.

Rappel

Effet Doppler pour un milieu immobile, une source S de vitesse ~v et un recepteur M de vitesse w
~:

1w ~ ~u/c SM
= 0 avec ~u = . (2)
1 ~v ~u/c SM

B. R
eflexion et transmission
Une corde tendue ` a la tension T0 = 10 N est
constituee de deux moities 1 et 2 de masses lineiques
y
respectives 1 = 4 g m1 et 2 = 6,25 g m1 .
La corde au repos concide avec laxe x Ox, la
2
moitie 1 avec x 0 et la moitie 2 avec x 0. x 1 O x
On etudie les petits deplacements transverses sui- T0
vant Oy de la corde. On ecrit le deplacement y(x, t) T0
dans chaque moitie sous la forme
(
y1 (x, t) si x 0 ;
y(x, t) = (3)
y2 (x, t) si x 0.
Fig. 11 La corde vibrante.


1.a) Ecrire les equations de dAlembert verifiees par y1 (x, t) et y2 (x, t).
s s
T0 T0
On posera c1 = et c2 = .
1 2

b) Justifier que les conditions aux limites en x = 0 secrivent

y1 (0, t) = y2 (0, t) (4)

et
y1 y2
(0, t) = (0, t). (5)
x x

2) Une onde incidente s1 , sinusodale, de frequence = 100 Hz, se propage de la moitie 1 vers la moitie
` la jonction en x = 0 cette onde incidente donne naissance `a une onde reflechie s1 dans la moitie 1
2. A
66 `
6. DEUXIEME PARTIEL 2003-2004

et `a une onde transmise s2 dans la moitie 2. On ecrit en representation complexe

s1 (x, t) = A1 ei(tk1 x) x0 (6)


s1 (x, t) = A1 ei(t+k1 x) x0 (7)
s2 (x, t) = A2 ei(tk2 x) x 0. (8)

a) Justifier pourquoi les ondes reflechie et transmise ont la meme frequence que londe incidente.

b) Calculer numeriquement k1 et k2 .

c) Determiner A1 et A2 en fonction de A1 , c1 et c2 .

d) Application numerique. Calculer A1 et A2 pour A1 = 9 mm. Quel est le dephasage entre londe
reflechie et londe incidente en x = 0 ?

e) Calculer la valeur numerique du coefficient de reflexion en puissance


2
A
R = 1 .

(9)
A1

Quelle est la valeur numerique du coefficient de transmission en puissance T ?

C. Condensateur
elastique

Le condensateur elastique est un syst`eme compose dun


ressort R de raideur k et dun condensateur plan forme de P
deux armatures circulaires A et B de rayon r et de meme
axe vertical (cf. figure 12). On appelle disques A et B les

AAAAAAA
surfaces de ces armatures internes au condensateur. R k

AAAAAAAA
Larmature B est fixe. Larmature A est mobile de sorte Q

BAAAAAAA
que la distance x entre les disques A et B du condensateur x
est variable. On suppose que r x 0 (x = 0 lorsque les
disques A et B sont en contact). AAAAAAA
Q r
O

Le ressort R est tendu entre un point fixe P situe dans


laxe du condensateur et le centre de larmature A du
Fig. 12 Le condensateur elastique.
condensateur.

On neglige le poids du ressort et de larmature A. La repartition des charges est en equilibre electrosta-
tique. Larmature A, au potentiel electrostatique UA , porte la charge Q 0 et larmature B, au potentiel
electrostatique UB , la charge Q. Les armatures sont isolees (Q ne varie pas).
Lorsque le condensateur est decharge, la position dequilibre de larmature A est x = a.
Donnees numeriques. Les constantes sont 0 = 8,85 1012 F m1 , k = 102 N m1 , r = 10 cm et a = 1 cm.

1) Pourquoi peut-on considerer que la repartition des charges est uniforme sur les disques A et B ?

2) Calculer, en fonction de 0 , k, Q, r, a et x, la force electrique F~ et la force elastique F~ due au ressort


qui agissent sur larmature A.
67

3) Montrer que la position dequilibre correspond `a la distance x egale `a

Q2
d=a . (10)
20 kr2

4) Cette position dequilibre est-elle stable ?

5) Pour quelle valeur Q0 de Q les deux armatures sont-elles en contact `a lequilibre ?


Que se passe-t-il si Q Q0 ?

6) Tracer la courbe de d en fonction de Q.

7) Determiner, en fonction de 0 , k, Q, r et a, la difference de potentiel U = UA UB `a lequilibre


(x = d).

8) Application numerique. Calculer la valeur numerique de Q0 .


Calculer la distance d et la difference de potentiel U `a lequilibre pour Q = Q0 /5.

9) Determiner, en fonction de 0 , k, Q, r, a et x, lenergie electrostatique We (x) et lenergie elastique


Wm (x) du syst`eme (on ne suppose pas quil y a equilibre mecanique).

10) Retrouver les reponses aux questions 3 et 4 en utilisant des considerations energetiques.

D. Induction magn
etique dans un cadre en mouvement
Soit (~ex , ~ey , ~ez ) la base orthonormee dun referentiel galileen Oxyz. On consid`ere le champ electroma-
gnetique statique (dans ce referentiel) donne au point ~r = (x, y, z) par

~ r ) = (ky, kx, 0)
B(~ et ~ r ) = (0, 0, 0)
E(~ (11)

u k = 0,1 T m1 est une constante.


o`
On donne la permeabilite magnetique du vide 0 = 4 107 SI.


1.a) Ecrire les equations de Maxwell.

b) Montrer que le champ (11) est une solution de ces equations.


Determiner le courant volumique ~(~r).

c) En quelle unite ~(~r) est-il mesure ? Calculer sa valeur numerique.


68 `
6. DEUXIEME PARTIEL 2003-2004

z
2) Un cadre carre P QRS, de c ote 2a, se deplace par un mouvement de a a
translation `a vitesse constante
~ = (u, v, w). S P
V
a
` linstant t = 0, les coordonnees des sommets du carre sont
A
O x
P (+a, 0, + a) a
Q (+a, 0, a) R Q
R (a, 0, a)
S (a, 0, + a) Fig. 13 Le cadre carre a
`
t = 0.
Le cadre est oriente dans le sens P QRS.

a) Calculer le flux (t) du champ magnetique `a travers le carre `a linstant t.

b) En deduire la force electromotrice dinduction qui apparat dans le cadre.


Application numerique. Calculer e au temps t = 0 pour a = 1 cm et u = v = w = 10 m s1 .


c) Enoncer la loi de Lenz. Verifier quelle sapplique ici.

3) Un theor`eme affirme que la force electromotrice dinduction qui apparat dans un circuit ferme C en
mouvement dans un champ magnetostatique est donnee par la circulation
I
~

= ~ B
V dl. (12)
C



Dans cette expression, V~ est la vitesse de lelement de circuit dl et B
~ est le champ magnetique au point


o`
u se trouve lelement de circuit dl.
Calculer `a linstant t = 0 la circulation (12) le long du cadre carre P QRS et verifier que le resultat est
coherent avec lexpression de la force electromotrice dinduction e obtenue `a la question 2.b.
69

Corrig
e
A. Effet Doppler dun sonar

 v
1.a) La frequence du signal recu par le poisson est 1 = 1 0 .
c

b) La frequence du signal recu par le sonar apr`es reflexion sur le poisson est
1 cv
= = 0 . (13)
1 + v/c c+v

0
c) On en tire v = c= c = 12,15 m s1 .
0 + 20 +

d) Les ondes emises et recues sont converties en signaux electriques U (t) = A cos(20 t) et U (t) =
A cos(2 t) respectivement. Ces deux signaux sont superposes pour former Ur (t) = U (t) + U (t) qui
presente le phenom`ene des battements. Une diode detectrice permet de ne conserver du signal precedent
que la partie positive de lenveloppe. Cest un signal de frequence . La mesure de sa frequence donne
avec une meilleure precision que par mesures separees de et 0 (si la precision relative des mesures
de frequences est la meme).

e) Mesure de la vitesse des galaxies lointaines `a partir du decalage vers le rouge de leur lumi`ere.

Elargissement des raies des lampes spectrales.


Mesure de la vitesse du sang dans les vaisseaux sanguins.
Controle de la vitesse des automobiles par la police.

2.a) Les impulsions 1 et 2 sont reflechies sur le poisson aux instants t1 /2 et (t2 + T )/2 respectivement.
Les distances parcourues par les impulsions sont 2L+vt1 et 2L+v(t2 +T ) respectivement. Les impulsions
se propagent `a la vitesse c pendant les temps t1 et t2 T respectivement. On a donc

ct1 = 2L + vt1 et c(t2 T ) = 2L + v(t2 + T ). (14)

On en tire
2L T (c + v) + 2L
t1 = et t2 = , (15)
cv cv
c+v
puis T = T . On retrouve lequation (13) en posant T = 1/ et T = 1/0 .
cv

b) On obtient, en resolvant en v et L le syst`eme dequations (14),

T T ct1 T
v= c et L= . (16)
T + T T + T

v = 10 m s1 et L = 130 m.
70 `
6. DEUXIEME PARTIEL 2003-2004

B. R
eflexion et transmission

1.a) Les equations de dAlembert sont

2 y1 1 2 y1 2 y2 1 2 y2
=0 et = 0. (17)
x2 c21 t2 x2 c22 t2

b) Se reporter `a la section 4.16 du cours Corde vibrante & acoustique.

2.a) Se reporter `
a la section 4.16 du cours Corde vibrante & acoustique.

b) Application numerique : c1 = 50 m s1 , c2 = 40 m s1 , k1 = 2/c1 = 12,6 m1 et k2 = 2/c2 =


15,7 m1 .

c) Les conditions aux limites (4) et (5) donnent

A1 + A1 = A2
k1 A1 k1 A1 = k2 A2

On en tire
k1 k2 c2 c1
A1 = A1 = A1 (18)
k1 + k2 c1 + c2
2k1 2c2
A2 = A1 = A1 . (19)
k1 + k2 c1 + c2

d) Application numerique. A1 = 1 mm et A2 = 8 mm. Le signe negatif de A1 signifie que le dephasage


entre londe reflechie et londe incidente en x = 0 est . Les vibrations sont opposition de phase. Il y a
reflexion avec changement de signe.

e) R = 0,0123456 et T = 0,9876543.

C. Condensateur
elastique

1) Comme x r, le condensateur est beaucoup plus etendu horizontalement que verticalement. Loin
des bords, il peut etre considere comme invariant par translations horizontales. Il en resulte quon peut
considerer que la densite de charge est uniforme sur les disques A et B. Ce resultat nest quapproche et
nest plus valable `
a des distances de lordre de x du bord des disques A et B.

Q
2) La densite de charge surfacique sur larmature A vaut = . La pression electrostatique est
r2
2
p = . Soit ~ux un vecteur unitaire le long de laxe Ox (~ux est vertical et va de larmature B vers
20
larmature A). La force electrique qui agit sur larmature A est verticale et vaut

Q2
F~ = pr2 ~ux = ~ux . (20)
20 r2
71

Le signe negatif indique que cette force est dirigee vers larmature B.
La force elastique est une force verticale de rappel vers la position x = a

F~ = k(x a) ~ux . (21)

La force va vers larmature B si x > a et dans le sens oppose si x < a.

Q2
3) La condition dequilibre est F~ + F~ = 0, soit k(d a) = 0. On en tire la position dequilibre
20 r2

Q2
d=a . (22)
20 kr2

4) La force electrique est constante. La force elastique mesuree algebriquement suivant Ox augmente si
x diminue. La force F~ + F~ tend `
a ramener larmature vers x = d. La position dequilibre x = d est donc
stable.

5) On obtient d = 0 (les armatures sont alors en contact) 6) Courbe de d en fonction de Q.


pour p d
Q = Q0 = 20 kr2 a. (23) a
Lequation (22) secrit en fonction de a, Q et Q0 :
 
Q2
d=a 1 2 . (24)
Q0

Si Q Q0 , le disque A se colle au disque B et le conden-


sateur se decharge.

O Q0 Q

7)
 
Qd Q Q2
U= 2
= a (25)
0 r 0 r2 20 kr2

8) Q0 = 7,46 109 C ; d = 0,96 cm ; U = 51,5 V.

Q2 x
9) Lenergie electrostatique est We (x) = et lenergie mecanique Wm (x) = 21 k(x a)2 .
20 r2

10) A` lequilibre mecanique, lenergie totale W (x) = We + Wm est stationnaire. Lequation dW =


dx
Q2 d2 W
+ k(x a) = 0 redonne lequation (22). Comme = k > 0, lenergie W (x) est minimum pour
20 r2 dx2
x = d. On retrouve ainsi que lequilibre est stable.
72 `
6. DEUXIEME PARTIEL 2003-2004

D. Induction magn
etique dans un cadre en mouvement

1) Les equations de Maxwell secrivent


~
E
~ B
~ = 0~ + 0 0 (26)
t
~ E
~
= (27)
0
~
B
~ E
~ = (28)
t
~ B
~ = 0. (29)

b) Le rotationnel du champ magnetique (11) est ~ B ~ = 2k~ez . On en deduit que le champ (11) est une
2k
solution des equations de Maxwell avec (~r) = 0 et ~(~r) = ~ez .
0

c) Le courant volumique est uniforme et vaut numeriquement j = 1,6 105 A m2 .

` linstant t, le carre est lensemble des points de coordonnees (x, y, z) telles que :
2.a) A

ut a x ut + a, y = vt, wt a z wt + a.

On calcule le flux `
a travers le carre oriente suivant la normale ~ey :
Z ut+a Z wt+a Z ut+a
 
(t) = dx ~
dz B ~ey = 2ka xdx = ka (ut + a)2 (ut a)2 = 4ka2 ut.
uta wta | {z } uta
kx

b) On en deduit
d
e= = 4ka2 u (30)
dt
qui est independant de v et w. Application numerique. e = 4 104 V.

c) Loi de Lenz. La force electromotrice dinduction qui apparat dans un circuit C tend `a produire un
courant de sens tel que le flux induit (cest-` ~ cree par ce courant) quil envoie
a-dire le flux du champ B
a travers C soppose `
` a la variation du flux inducteur qui lui donne naissance.
Ici, pour u > 0, le cadre se deplace vers des regions o`
u By crot. La force electromotrice induite tend `a
faire circuler un courant qui envoie un flux induit negatif a` travers le carre. Ce flux induit soppose bien
a la variation de comme indique par la loi de Lenz.
`

3) Comme la vitesse de tous les points du circuit est la meme, on a


I I
 ~
= ~ ~
V B dl = V B ~
dl . (31)
C C

~ est uniforme le long du segment P Q et


Le champ B
Z
B~
~
dl = B P Q = (ka ~ey ) (2a ~ez ) = 2ka2~ex . (32)
PQ
73

~ est uniforme le long du segment RS et


Le champ B
Z
~
B
~
dl = B RS = (ka ~ey ) (2a~ez ) = 2ka2~ex . (33)
RS

Le champ B~ est le meme en deux points des segments QR et SP de meme abscisse x. Il en resulte que
~
R ~
R
QR
B dl + SP B dl = 0. On a donc
I
~
B

dl = 4ka2~ex et = V~ (4ka2~ex ) = 4ka2 u. (34)
C

Cest bien coherent avec lexpression (30) de la force electromotrice dinduction e.


74 7. EXAMEN 2003-2004

7. Examen 2003-2004


Enonc
e
12 janvier 2004
duree : 3 heures
sans documents, calculatrices autorisees

Bar`eme indicatif (total 20) : A = 5 ; B = 3 ; C.I = 4,5 ; C.II = 2,5 ; C.III = 5.


Il sera tenu compte de la presentation et de la concision dans la notation.
De tr`es nombreuses questions sont independantes.

A. Questions de cours

1) Donner lequation locale exprimant la conservation de lenergie electromagnetique.


Commenter la signification physique des differents termes de cette equation.


2) Etablir lexpression de la capacite C dun condensateur plan, dans le vide, de surface S et depaisseur
d. Vous introduirez les approximations qui vous semblent pertinentes.
Que devient cette capacite lorsque le condensateur est rempli dun dielectrique lineaire, homog`ene et
isotrope de permittivite absolue ?

3) Un faisceau lumineux parall`ele et monochromatique de longueur donde 0 = 0,5 m passe `a travers


une ouverture circulaire de diam`etre D = 0,1 mm percee dans un ecran opaque place perpendiculairement
a la direction de propagation du faisceau.
`
On observe la figure de diffraction dans le plan focal dune lentille convergente de distance focale f = 3 m.
Decrire sommairement cette figure.
Quelles sont les dimensions de sa tache centrale ?
Donner le resultat numerique.

4) Les expressions suivantes representent le champ electrique au point M et `a linstant t de diverses


ondes.
Pour chacune de ces ondes, dire si londe est plane, spherique, homog`ene, inhomog`ene, progressive,
stationnaire, attenuee ; preciser sa polarisation et ses surfaces donde.
75

(a) A cos(t) cos(ky)~uz

(b) A cos(t kx)~uy + A sin(t kx)~uz



(c) Aek z
cos(t k z)~ux pour z > 0

A sin h  r i
(d) cos t ~u pour r > D
r c


Le vecteur ~r = OM est donne en coordonnees spheriques r, , ou cartesiennes x, y, z, les vecteurs
unitaires associes etant notes ~ur , ~u , . . . Les constantes A, D, , k, k , k sont des nombres reels
strictement positifs.

B. Fentes dYoung

L E L x

O1 E

M
S d
z F C C z
O

O2

f f

Fig. 14 Dispositif des fentes dYoung.

On place, dans le plan focal objet dune lentille mince convergente L , une fente F , fine, rectangulaire
dont la plus grande dimension est perpendiculaire au plan de section principale represente sur la figure 14.
Une lampe spectrale S emet une lumi`ere monochromatique de longueur donde dans lair (dont lindice
de refraction est n) et eclaire cette fente F .
Dans lecran opaque E , perpendiculaire `a laxe z z du dispositif, on decoupe deux fentes O1 et O2 ,
a F , distantes de d et symetriques par rapport `a z z.
parall`eles et identiques `
La lumi`ere issue des fentes O1 et O2 est recue par une deuxi`eme lentille mince convergente L, de distance
focale f , dont le plan focal image est occupe par un ecran dobservation E. Soit O le foyer image de L,
Oy un axe perpendiculaire au plan de la figure 14 et Ox un axe perpendiculaire au plan Oyz.
On observe les interferences au point M E de coordonnees (x, y). On suppose que |x| f et |y| f .

1) Reproduire la figure 14 et tracer les deux rayons issus de F qui arrivent au point M de laxe Ox. On
demande une construction geometrique de ces deux rayons.

2) Calculer la difference de marche des deux rayons qui arrivent au point M (x, y = 0) de laxe Ox en
fonction des donnees de lenonce.
76 7. EXAMEN 2003-2004

3) Decrire les franges dinterferences observees sur lecran E


(question de cours ` a laquelle on demande de repondre sans
calculs). Determiner linterfrange i.
z S F z

4) Comment peut-on mesurer la longueur donde avec ce


dispositif ?

Lc
5) On place entre S et F une lentille convergente Lc qui forme
limage de la lampe S sur la fente F (cf. figure 15).
Quel est linteret de ce nouveau montage par rapport au mon-
Fig. 15 Nouveau montage.
tage de la figure 14 ?

C. Guide dondes hyperfr


equences

On consid`ere un guide dondes rectangulaire daxe parall`ele AAAAAAAAA


y

AAAAAAAAA
a
a Oz (cf. figure 16). Cest un tube metallique creux rempli
`
AAAAAAAAA
AAAAAAAAA
dair de section rectangulaire de dimensions a suivant x et b

AAAAAAAAA
suivant y (a > b). Le metal du guide donde est suppose par-
air b
faitement conducteur. Les ondes se propagent suivant laxe
Oz perpendiculaire `
on prendra
a la figure. On assimilera lair au vide et
AAAAAAAAA
O

AAAAAAAAA
x

c = 3 108 m s1
conducteur
pour la celerite de la lumi`ere dans lair ou le vide. La base Fig. 16 Section droite du guide
orthonormee cartesienne est notee (~ux , ~uy , ~uz ). dondes.

I. Mode TE10

1) Une onde TE10 (transverse electrique un-zero) de pulsation se propage dans le guide. Son champ
electrique en representation complexe secrit `
a linstant t et au point de coordonnees x, y, z (0 < x < a,
0 < y < b)
E
x = 0

 
Ey = A sin x eitikz (1)

a

Ez = 0
o`
u A et k sont des constantes reelles strictement positives.

a) Ecrire lequation donde verifiee par Ey .

b) En deduire que
2  2
= + k2 . (2)
c2 a


2) Ecrire les equations de Maxwell dans le guide (0 < x < a, 0 < y < b).

3) Determiner le champ magnetique de londe. On determinera ses composantes B x , B


y et B
z en repre-
sentation complexe `a linstant t et au point de coordonnees x, y, z en fonction de a, A, k et . On ecrira
ensuite ses composantes reelles Bx , By et Bz .
77

Justifier le nom transverse electrique donne `a londe etudiee.

4.a) Soit P un point sur la surface interne du conducteur. Soit ~n le vecteur unitaire normal `a cette surface

au point P et oriente vers linterieur du guide dondes. Ecrire les relations de passage metal-air du champ
electromagnetique au point P . On les ecrira en fonction des champs electrique E ~ P et magnetique B~P
dans lair au voisinage de P , des densites surfaciques de charge et de courant ~ au point P et du
vecteur unitaire ~n.
~ determine en 3) sont compatibles
b) Montrer que le champ electrique (1) et le champ magnetique B
avec ces relations.

c) Determiner les densites (reelles) (x, z, t) et ~ (x, z, t) = jx ~ux + jz ~uz `a linstant t et au point P
(x, 0, z) de la paroi y = 0.

jx jz
d) Quelle relation existe-t-il entre les derivees partielles du premier ordre , et des densites
t x z
determinees en c) ? Quelle est la signification physique de cette relation ?

x
5) On realise une ligne a` fente (cf. figure 17) en percant a
5a
dans la paroi du guide donde une fente etroite. Soit D a D
2 a/10
la droite du plan Oxz dequation x = a/2. La fente
O z
de longueur 5a et de largeur a/10 est placee le long
de D. Elle permet dintroduire dans le guide dondes
une tr`es fine et courte antenne, appelee sonde, qui peut Fig. 17 Ligne a
` fente.
etre deplacee parall`element `
a laxe Oz.

a) Verifier que, dans le guide dondes sans fente, la droite D est une ligne de courant. Faire un schema
qui indique le sens du courant sur cette ligne `a linstant t = 0.

b) Les autres droites parall`eles `


a Oz du plan Oxz sont-elles aussi des lignes de courant ?

c) Quel avantage voyez-vous `


a percer le guide dondes le long de D plut
ot quailleurs pour realiser la
ligne `a fente ?

II. Autres modes de propagation

1) Une onde de pulsation se propage dans le guide. Son champ electrique en representation complexe
secrit `a linstant t et au point de coordonnees x, y, z (0 < x < a, 0 < y < b)


E = 0
x


y
E = A sin(x)eitikz (3)

z
E = 0

o`
u A, et k sont des constantes reelles strictement positives.
Quelle condition doit satisfaire le champ electrique au voisinage des parois ?
Montrer que cette condition est satisfaite sur les parois du guide par le champ (3) pour des valeurs
discr`etes m de qui sexpriment en fonction de a et dun entier m = 1, 2, . . .

2) Determiner la relation entre , c, et k qui doit etre verifiee pour que londe puisse se propager dans
le guide.
78 7. EXAMEN 2003-2004

3) Quelle est la relation de dispersion du mode m (qui correspond `a m ) ?


Montrer que le mode m ne peut se propager que pour une pulsation superieure `a la pulsation de coupure
m que lon exprimera en fonction de a et c.

4) Depuis la Conference Generale des Poids et Mesures de 1967,


la seconde est la duree de 9 192 631 770 periodes de la radiation correspondant `a la transition
entre les deux niveaux hyperfins de letat fondamental de latome de cesium 133.
On desire faire propager dans le guide une onde electromagnetique ayant la frequence 0 de cette radiation
(pour construire une horloge atomique).

a) Quelle condition la largeur a du guide doit-elle verifier ?

b) On souhaite de plus que londe de frequence 0 ne se propage dans le guide que dans le mode
m = 1. Determiner, litteralement et numeriquement, lintervalle des largeurs a du guide qui satisfont `a
cette condition.

III. R
eflexion et interf
erences dans une ligne hyperfr
equence

sonde
1) Le guide donde rectangulaire est O z
ferme en z = 0 par une lamelle en plas- lamelle en
plastique a b
tique (appelee charge). Un emetteur
emetteur
x
produit une onde hyperfrequence de
frequence et de longueur donde dans
y
lair . Cette onde est envoyee dans le
guide et se reflechit sur la charge (cf.
Fig. 18 Reflexion sur une lamelle en plastique.
figure 18).

Londe dans le guide (z 0) est la superposition dune onde incidente se propageant vers les z decroissants
et dune onde reflechie en sens oppose. Ces deux ondes, incidente et reflechie, sont toutes deux des ondes
TE10 (le guide est suppose monomode ` a la frequence ) dont le champ electrique est parall`ele `a Oy. On
ecrit pour le champ electrique reel total
~ = Ey (x, z, t) ~uy ,
E Ey = Ey + Ey (4)

avec, en representation complexe,


   
= A sin x eit+ikz ,
E = rA sin x eitikz
E (5)
y y
a a
o`
u A, et k sont des constantes reelles positives et o`
u le nombre complexe

r = rei (6)

de module r et dargument est le coefficient complexe de reflexion sur la charge.


2
a) En utilisant les resultats de la partie C.I, determiner la frequence en fonction de g = , a et c.
k
b) Comparer g `
a la longueur donde dans lair .

c) La constante g sappelle la longueur donde dans le guide. Justifier ce nom.

2) Une sonde detectrice (antenne plongeant dans le guide et reliee `a un cristal detecteur) se deplace
suivant Oz en restant au centre du guide. Elle produit une tension V (z) proportionnelle au carre du
79

module du champ electrique complexe :


2

V (z) = K E y (x = a/2, z, t) (7)

o`
u K (K > 0) est une constante.
Calculer la tension V (z) en fonction de z et des constantes K, A, g , r et .

3.a) Montrer que V (z) est une fonction periodique de z. Quelle est sa periode ?

b) Determiner le maximum Vmax et le minimum Vmin de V (z) ainsi que leurs positions respectives zmax
et zmin .

c) Determiner zmax zmin pour deux extremums successifs.


Nota : Les diverses reponses `
a la question 3) seront donnees en fonction de K, A, g , r et .

4) Les dimensions du guide dondes sont a = 22,86 mm et b = 11,43 mm. Des mesures avec la sonde
donnent deux minimums successifs de V (z) aux distances z1 = 48,25 mm et z2 = 71,44 mm de la charge
Vmin
et un rapport = = 0,140.
Vmax

a) Exprimer litteralement r en fonction de . Calculer sa valeur numerique.

b) Calculer numeriquement g , et (donner en radians avec 0 < 2).

5) Tracer la courbe de V (z) en fonction de z pour z variant de 0 `a z2 = 71,44 mm.

6) Quelle est la signification physique du produit g ?


Calculer et commenter sa valeur numerique.
80 7. EXAMEN 2003-2004

Corrig
e
A. Questions de cours

1) Lequation locale exprimant la conservation de lenergie electromagnetique est

u ~ +
~ P~ = 0.
+ ~ E (8)
t
La grandeur
~2
0 E ~2
B
u(~r, t) = + (9)
2 20
est la densite denergie electromagnetique et
~ B
E ~
P~ (~r, t) = (10)
0
~ est la puissance par unite de
est le vecteur de Poynting (vecteur courant denergie). La grandeur ~ E
volume fournie par le champ electromagnetique aux charges.

2) On suppose que lepaisseur d est tr`es petite devant les di-


mensions lineaires des armatures. On peut alors negliger les
effets de bord. Le champ electrique est determine comme
etant approximativement celui du syst`eme forme de deux d u
~z ~
E
plans parall`eles distants de d, chaque plan portant une charge
surfacique constante et uniforme ( sur le plan z = 0, O x
sur le plan z = d). On prend = Q/S o` u Q > 0 est la
charge electrique portee par une armature du condensateur
reel (lautre armature porte la charge Q) (cf. figure 19). Fig. 19 Condensateur plan.

Le champ electrique du condensateur E ~ =E ~1 + E~ 2 est determine par superposition des champs crees
par chacun des plans.
Determinons le champ E ~ 1 cree par les charges du plan z = 0. Ce sous-syst`eme est invariant par transla-
tions parall`eles `a Ox et Oy, par rotations autour de tout axe parall`ele `a Oz et par symetrie par rapport
au plan z = 0. Il en resulte que le champ electrique est de la forme E ~ 1 = E1 (z)~uz avec E1 (z) = E1 (z)
(fonction impaire). Lequation de Gauss-Maxwell

~ 1 = dE1 = 0
~ E pour z 6= 0 (11)
dz
et la relation de passage

E1 (0+ ) E1 (0 ) = (12)
0
donnent
si z > 0
E1 (z) = 20 (13)
si z < 0.
20
~ 2 cree par les charges du plan z = d est E
On montre de meme que le champ E ~ 2 = E2 (z)~uz avec

si z > d
E2 (z) = 20 (14)

si z < d.
20
81

Le champ electrique du condensateur est donc



~uz si d > z > 0
~
E = 0 (15)
0 sinon.

Le potentiel electrostatique V (z) sobtient `a partir de lequation E ~ = V


~ et de la continuite de V (z) :
z
0 + K si d z 0



V (z) = K si z 0 (16)


d
+ K si z d
0
o`
u K est une constante. La difference de potentiel entre les armatures est
d Qd
U = V (0) V (d) = = . (17)
0 S0
On en deduit la capacite C = Q/U du condensateur :

S0
C= . (18)
d

Cette capacite devient, lorsque le condensateur est rempli dun dielectrique lineaire, homog`ene et isotrope
de permittivite absolue ,
S
C= . (19)
d
Cette formule est obtenue en remplacant 0 par dans (18).

3) La figure de diffraction (tache dAiry) a lallure de la photo ci-contre.


La plus grande partie (84 %) de la puissance lumineuse est concentree
dans la tache centrale qui est un disque (disque dAiry) de rayon

1,22 f 0
1 = = 18 mm. (20)
D

4) Cas a b c d3
plane oui oui oui
spherique oui
homog`ene oui oui oui
inhomog`ene oui4
progressive oui 1 1

stationnaire oui
attenuee oui2 5

rectiligne circulaire rectiligne rectiligne6


polarisation
suivant ~uz gauche suivant ~ux suivant ~u
surfaces donde plans y = Cte plans x = Cte plans z = Cte sph`eres r = Cte

Notes.
1. Le signal dune onde progressive se propage `a vitesse constante sans se deformer. Londe (c) nest pas
progressive (le signal sattenue). Londe (d) non plus (decroissance en 1/r du signal).

2. Londe (c) se propage avec attenuation dans la direction et le sens de ~uz car son amplitude Aek z

decrot exponentiellement avec la distance z parcourue par londe. (Le milieu est absorbant.)
82 7. EXAMEN 2003-2004

3. Londe (d) est londe cree par un dip ole electrique oscillant `a lorigine de moment dipolaire electrique
p~(t) = p0 cos(t)~uz , avec A = 0 p0 2 /4. Londe est spherique divergente.
A sin
4. Londe (d) est inhomog`ene car son amplitude ~u varie sur une surface donde.
r
5. Lamplitude de londe (d) decrot en 1/r, mais on ne dit pas quelle est attenuee, car son energie nest
pas absorbee (en se propageant, elle se repartit dans un plus grand volume).
6. La polarisation de londe (d) depend du point M .

B. Fentes dYoung

1) Les rayons O1 A1 et O2 A2 sont parall`eles `


a CM .
L
L E x
O1 A1
E

M
S d
z F C C z
O

A2
H
O2
f f

Fig. 20 Construction geometrique des rayons.

2) Calculons la difference de marche = [F O2 M ][F O1 M ]. Soit H la projection de O1 sur O2 A2 . Langle


= (Oz, CM ) = (O1 O2 , O1 H) vaut x/f . Il y a egalite des chemins optiques [HA2 M ] = [O1 A1 M ]
(stigmatisme). On en deduit = [O2 H] nd soit
nxd
= . (21)
f

3) Les franges sont rectilignes et parall`eles `


a Oy. Les franges brillantes correspondent `a = p0 = pn
(p entier) soit x = pf /d. Linterfrange est donc
f
i= . (22)
d

4) La mesure de i, d et f permet dobtenir la longueur donde


par
id
= . (23)
f z S F z

5) La lentille Lc concentre le flux lumineux emis dans langle


solide sur la fente. En cas declairage direct, la fente F Lc
ne recoit que le flux lumineux emis dans langle solide
( ) sous lequel elle est vue de S. Lutilisation de la
lentille Lc (ce dispositif sappelle un condenseur) augmente Fig. 21 Nouveau montage.
leclairement des franges dinterferences.
83

C. Guide dondes hyperfr


equences

I. Mode TE10

y du champ doit etre solution de lequation de dAlembert :


1.a) La composante E

2Ey 2Ey 2Ey 2Ey


2 2
2
2
= 0. (24)
c t x y z 2
b) On a donc

y  2
2E
+ y + k 2 E
E y = 0 (25)
c2 a
do`
u, en simplifiant,
2  2
= + k2 . (26)
c2 a

2) Dans le guide = 0 et ~ = 0. Les equations de Maxwell secrivent

1 E ~
~ B
~ = (27)
2
c t
~ E
~ = 0 (28)
B~
~ E
~ = (29)
t
~ B
~ = 0. (30)

3) Lequation (29) donne, pour B~ dependant du temps par le facteur eit ,


 x 
x
B /x 0 ikA sin eitikz
  a
~ = /y A sin x eitikz =

i y =
B ~ E 0

. (31)
a  
A x itikz
z
B /z 0 cos e
a a
On en tire
kA  x 
x
B = sin eitikz
a
y
B = 0 (32)
iA  x 
z
B = cos eitikz .
a a
On obtient en prenant les parties reelles de ces equations

kA  x 
Bx = sin cos(t kz)
a
By = 0 (33)
A  x 
Bz = cos sin(t kz).
a a

~ est perpendiculaire `
Le champ E ~ (puisquil a une
a la direction de propagation Oz, mais pas le champ B
composante Bz 6= 0) : Londe est transverse electrique (TE), mais pas TEM.
84 7. EXAMEN 2003-2004

4.a) Le metal du guide donde etant suppose parfaitement conducteur, les champs sont nuls dans le
conducteur. Les relations de passage en P secrivent :
~P
~n B = 0~ (34)
~P
~n E = (35)
0
~n E ~P = 0 (36)
~P
~n B = 0. (37)

On peut aussi les ecrire de facon equivalente

~P
E = ~n (38)
0
~P
B = 0~ ~n. (39)

b) Lequation (36) signifie que le champ electrique E ~ P doit etre perpendiculaire aux parois (ou nul). Le
champ (1) etant parall`ele ` a Oy, cette condition est verifiee sur les parois y = 0 et y = b. La condition
est verifiee sur les parois x = 0 et x = a o`
u le champ sannule.
Lequation (37) signifie que le champ magnetique B ~ P doit etre parall`ele aux parois (ou nul). Le champ (33)
etant perpendiculaire ` a Oy, cette condition est verifiee sur les parois y = 0 et y = b. La condition est
verifiee sur les parois x = 0 et x = a o`
u sa composante Bx sannule.
Cela termine la demonstration que le champ electromagnetique donne par (1) et (32) est compatible
avec les relations de passage (3437). En effet, il ny a rien a` verifier pour les equations (34) et (35). On
utilisera dans la question suivante ces deux equations pour determiner les densites et ~ .

c) Sur la paroi y = 0, on a ~n = ~uy . En portant le champ (33) dans (34) on obtient


 x  A  x 

0
kA cos sin(t kz)
sin cos(t kz) 0 a a
1 a
~ =

0 =
0 . (40)

0 A  x 
kA  x 
0 cos sin(t kz) sin cos(t kz)
a a 0 a
En portant le champ (1) dans (35) on obtient
 x 
= 0 Ey = 0 A sin cos(t kz). (41)
a

d) On a la relation
jx jz
+ + =0 (42)
t x z
qui exprime la conservation de la charge. La relation peut se verifier explicitement `a partir de (40),
(41) et (26).

5.a) Au point P D de coordonnees (x = a/2, y = 0, z) et `a linstant t, le courant surfacique


kA
~ = cos(t kz)~uz (43)
0
est parall`ele `a D. La droite D est donc une ligne de courant.
85

Le sens du courant `
a linstant t = 0 est donne par le signe de cos(kz).

D
z = 3
2k

2k
2k
3
2k
5
2k

Fig. 22 La ligne de courant D a


` t = 0.

b) Au point P / D de coordonnees (x 6= a/2, y = 0, z) et `a linstant t, le courant surfacique a une


composante jx 6= 0. Une autre droite D (D 6= D), parall`ele `a Oz et dans le plan Oxz nest donc pas
une ligne de courant.

c) Les courants traversent donc D . Si on perce une fente le long de D , les courants surfaciques sont tr`es
modifies. Par contre, en percant une fente le long de la droite D, les courants surfaciques sont assez peu
modifies.
Lavantage de lemplacement choisi est de minimiser la perturbation du champ electromagnetique du
mode m = 1.

II. Autres modes de propagation

1) Le champ electrique doit etre perpendiculaire aux parois (ou nul) au voisinage dune paroi parfaitement
conductrice.
Le champ (3) etant parall`ele `a Oy, cette condition est verifiee sur les parois y = 0 et y = b. La condition
est verifiee sur la paroi x = 0 o`
u le champ sannule. La condition est verifiee sur la paroi x = a si et
seulement si
sin(a) = 0 a 0 mod (44)
et donc si et seulement si ( > 0)
m
m = avec m Z+ . (45)
a
La valeur m = 1 correspond au cas etudie dans la partie I.

y du champ doit etre solution de lequation de dAlembert (24). On a donc


2) La composante E

2Ey
y + k 2 E
+ 2 E y = 0 (46)
c 2

et , c, et k doivent verifier
2
= 2 + k 2 (47)
c2
pour que londe puisse se propager dans le guide.

3) La relation de dispersion du mode m est

2  m 2
2
= + k2 . (48)
c a
Le mode m ne peut se propager que si cette relation correspond `a k reel non nul. On doit donc avoir
> m o`
u la pulsation de coupure est
mc
m = . (49)
a
86 7. EXAMEN 2003-2004

4.a) Pour que londe de pulsation 0 = 20 (avec 0 = 9 192 631 770 Hz) se propage, la pulsation 0 doit
au moins valoir la plus petite des pulsations de coupure. On doit donc avoir 0 > 1 , soit 20 > c/a
et la largeur du guide doit etre
c
a> = 1,63 cm. (50)
20
b) La condition necessaire et suffisante pour que londe de pulsation 0 ne se propage pas dans les modes

m = 2, 3, . . . est 0 2 (car 2 < 3 < . . . ) soit 20 2c/a ou encore a c/0 .


Lintervalle des largeurs tel que la frequence 0 ne se propage que dans le mode m = 1 est donc
c c
<a et, numeriquement, 1,6 cm < a 3,2 cm. (51)
20 0

Remarque. Le mode m est le mode TEm0 . Les modes du guide sont


TEmn (transverse electrique), o`
u m, n = 0, 1, 2, . . . sont des entiers positifs ou nuls, le cas m =
n = 0 etant exclu.
TMmn (transverse magnetique), o` u m, n = 1, 2, 3, . . . sont des entiers strictement positifs.
La frequence de coupure du mode TEmn ou TMmn est
r
c m2 n2
mn = 2
+ 2. (52)
2 a b
Le guide est monomode pour la frequence 0 si, en plus de la condition (51), cette onde ne se propage
pas dans le mode TE01 cest `
a dire si 0 c/2b soit b c/20 .

III. R
eflexion et interf
erences dans une ligne hyperfr
equence

1.a) Londe consideree est la superposition de deux ondes TE10 se propageant en sens opposes. La
relation entre = 2 et k = 2/g est la meme pour chacune de ces ondes et est donnee par (26). On
a donc
s   2
2
1 1
=c + . (53)
2a g

b) Dapr`es (53), > c/g . On a donc g > c/ = soit

g > . (54)

c) Le champ electrique E~ est periodique en z de periode spatiale g . Cela justifie le terme longueur
donde. On precise dans le guide car g est different de la longueur donde dans lair .

2) On a 
y (x = a/2, z, t) = Aeit eikz + reiikz .
E (55)
On calcule
2 2 
V (z) = KA2 eikz + reiikz = KA2 1 + rei2ikz = KA2 1 + 2r cos( 2kz) + r2 . (56)

La reponse est demandee en fonction de g :


   
4z
V (z) = KA2 1 + 2r cos + r2 . (57)
g
87

3.a) Lequation (57) montre que V (z) est periodique de periode

g
= . (58)
2

 
4z 4z
b) La tension V (z) est maximum pour cos = 1 soit = 0 mod 2. On a donc
g g

g g
Vmax = KA2 (1 + r)2 et zmax = mod . (59)
4 2
 
4z
La tension V (z) est minimum pour cos = 1 soit
g
4z
= mod 2 (60)
g
On a donc
g g g
Vmin = KA2 (1 r)2 et zmin = + mod . (61)
4 4 2

g
c) La difference demandee est zmax zmin = .
4

4.a) Dapr`es les equations (59) et (61)


 2
Vmin 1r
= = . (62)
Vmax 1+r
En prenant la racine carree, on a
1r
= . (63)
1+r
On doit prendre le signe + de car le coefficient de reflexion a un module inferieur ou egal `a 1 (0 r 1) :
1r
= . (64)
1+r
On en tire

1
r= = 0,4554. (65)
1+

b) La difference entre les distances de deux minimums successifs est = g /2. On a donc

g = 2(z2 z1 ) = 46,38 mm. (66)

Lequation (53) donne la frequence


= 9,214 GHz. (67)

4z1
On a un minimum en z1 . Lequation (60) donne = + = 5,161 mod 2 soit
g

= 3,648 rad = 209 . (68)


88 7. EXAMEN 2003-2004

5) Courbe de V (z) en fonction de z. 6) Le produit g = /k est la vitesse de phase v .


V Avec les donnees de la question precedente, la vitesse
Vmax
de phase v = 4,273 108 m s1 est plus grande que la
vitesse de la lumi`ere dans le vide c. Cest vrai de facon
generale, puisque lon a dapr`es (53)
s  2
g
v = g = c 1 + > c. (69)
2a

Vmin
La vitesse de groupe
z
O z1 z2

d c
vg = = s  2 , (70)
dk
g
1+
2a
qui est aussi la vitesse de propagation de lenergie dans le guide, est inferieure `a c.
89

8. Examen (2e session) 2003-2004


Enonc
e
1er septembre 2004
duree : 2 heures
sans documents, calculatrices autorisees

Les parties A, B et C sont independantes.


Bar`eme indicatif (total 20) : A = 6 ; B = 8 ; C = 6.

A. Onde harmonique stationnaire dune corde

Une corde de longueur l = 50 cm est tendue entre deux points


A (dabscisse xA = 0) et B (dabscisse xB = l). La masse
lineique de la corde est = 10 g m1 et la tension de la corde A B

est T . On observe dans la corde une onde transversale, harmo-


nique, stationnaire de frequence = 500 Hz. La corde prend
laspect de la figure ci-contre.

1) Expliquer le sens des mots transversale, harmonique et stationnaire.

2) Donner lexpression s(x,t) du deplacement du point dabscisse x `a linstant t pour xA x xB


sachant qu`a t = 0 la corde est dans sa position dequilibre et que la valeur maximum de s(x,t) est A.
On ecrira s(x,t) en fonction de x, t, , l et A. Sil y a plusieurs solutions s(x,t), on les donnera toutes.

3) Representer la forme de la corde `


a divers instants. Expliquer pourquoi la corde prend laspect de la
figure.

4) Determiner numeriquement la longueur donde , la vitesse v des ondes de la corde et la tension de


la corde T .
90 8. EXAMEN (2E SESSION) 2003-2004

B. Induction
Un solenode de longueur b tr`es grande devant son rayon a comporte n spires circulaires jointives par
unite de longueur. Il est parcouru par un courant continu dintensite I. On designe par ~u un vecteur
unitaire parall`ele `
a laxe Oz du solenode.

z
1) On supposera que le solenode est de longueur infinie (b = ) pour a

repondre `a cette question. On designe par P un point quelconque de lespace.


~ ) au point P est parall`ele `a ~u.
a) Montrer que le champ magnetique B(P
~ ) ne depend que de la distance de P `a laxe du
b) Montrer que B(P
solenode.
c) Montrer que le champ magnetique est uniforme `a linterieur du u
~
b
solenode. O
d) Justifier que le champ magnetique est nul `a lexterieur du solenode.
e) Determiner le champ magnetique `
a linterieur du solenode en fonction I
des donnees de lenonce.
f ) Quel est le coefficient dauto-induction par unite de longueur du
solenode?

2) Une bobine de section circulaire de rayon r (r < a), comportant N tours de fil, est placee `a linterieur
du solenode. De plus, les axes de la bobine et du solenode sont confondus.
Quel est le flux de B ~ `a travers la bobine?

En deduire la valeur du rapport M = . Quelle est le nom de la grandeur M ?
I

3) Le solenode est `
a present parcouru par un courant I(t) sinusodal, de frequence = /2 et dam-
plitude I0 :
I(t) = I0 sin t. (1)

~ a
a) Quelle est lexpression du flux de B ` travers la bobine en fonction du temps?

b) En deduire la f.e.m. dinduction e dans la bobine.

c) On relie les extremites de la bobine. On forme ainsi un circuit de resistance electrique R et dauto-
induction negligeable. Determiner le courant Ib qui circule dans la bobine.

d) Quel est le champ electrique Ei induit `


a linterieur du fil de la bobine?

4) Proposer une methode pour mesurer la grandeur M .


91

C. Interf
erom`
etre de Michelson
Lemetteur E gen`ere une onde plane electromagnetique de longueur donde (de lordre du cm) se pro-
pageant suivant laxe Oz et polarisee rectilignement suivant ~uy . Laxe Oy est vertical et perpendiculaire
`a la figure.
Linterferom`etre de Michelson comporte deux plans
reflecteurs identiques P1 et P2 et une plaque de bakelite x

B semi-reflechissante. Le plan reflecteur P1 est perpen-


plan rflecteur P1
diculaire `a Ox et `
a la distance X de O ; le plan reflecteur
P2 est perpendiculaire ` a Oz et `a la distance L de O ; cornet X
metteur
la plaque de bakelite est verticale, centree en O et in-
clinee de 45 sur laxe demission Oz. La distance de E
L L z

lemetteur E ` a O est L . On place le cornet recepteur O

R `a la distance L de O.
L
Dans linterferom`etre, seul le plan reflecteur P1 est plaque de
baklite B
mobile : X est variable, mais L, L et L sont des plan rflecteur P2

constantes. cornet
rcepteur
R

1) Il arrive sur le recepteur R deux ondes ayant suivi des chemins differents. On ecrit leurs champs
electriques en R sous la forme
~ 1 = a cos(t 1 ) ~uy ,
E ~ 2 = a cos(t 2 ) ~uy .
E (2)

Decrire les chemins de ces deux ondes. Justifier que les deux ondes ont la meme amplitude. Montrer que

= 1 2 = X + (3)

o`
u et sont des constantes et exprimer en fonction de .

2) Montrer, en utilisant la notation complexe, que la somme E~ =E~1 + E


~ 2 est une vibration harmonique
de la forme
~ = A cos(t + ) ~uy .
E (4)
Nota : on ne demande pas de calculer A et dans cette question.

3) Calculer lintensite I = A2 en fonction de a et . Decrire succinctement un dispositif experimental


permettant de mesurer I.

4) Tracer la courbe de I en fonction de X. Comment peut-on mesurer la longueur donde ?


92 8. EXAMEN (2E SESSION) 2003-2004

Corrig
e
A. Onde harmonique stationnaire dune corde

1) Transversale : les points de la corde se deplacent perpendiculairement `a Ox (comparer `a onde


longitudinale).
Harmonique : londe est sinusodale.
Stationnaire : tous les points de la corde vibrent en phase ou en opposition de phase (comparer `a onde
progressive).

2) La forme generale dune onde stationnaire harmonique est ( : longueur donde)


 
2x
s(x,t) = A sin(2t + ) sin + (5)

Londe presente deux fuseaux. On a donc les conditions : s(x,t) = 0 t pour x = 0, x = l/2 et x = l ;
pour les autres valeurs de x (0 < x < l/2 ou l/2 < x < l), s(x,t) nest pas identiquement nul. Puisque
s(x,t), `a tout instant t, est une sinusode en x, ces conditions impliquent que s(x,t) est proportionnel `a
sin(2x/l). On en deduit que = 0 et = l dans lequation (5). La condition `a t = 0 donne = 0 (mod
). On a donc, le signe ne pouvant etre determine par les conditions de lenonce,
 
2x
s(x,t) = A sin(2t) sin . (6)
l

3) La figure ci-contre represente la forme de la corde aux ins-


nT A B
tants tn = (mod T ) pour n = 0, 1, . . . , 15. La periode
16
T = 1/ = 0,002 ms est tr`es petite devant le temps de per-
sistance des images retiniennes. La corde apparat comme la
superposition de toutes les positions de la corde.

s
T
4) = l = 50 cm ; v = = 250 m s1 ; v = donne T = v 2 = 625 N.
T

B. Induction

1.a) Tout plan perpendiculaire ` ~ k ~u.


a ~u est plan de symetrie (pour b = ). Donc B

b) Le syst`eme est invariant par translation parall`ele `a ~u et par rotation autour de laxe du solenode.
~ ) = B(P )~u ne depend que de la distance de P `a laxe du solenode.
Donc B(P

c) Appliquons le theor`eme dAmp`ere au pourtour du rectangle P QRS. On prend P Q k ~u et de longueur


 
d. La circulation de B ~ le long de ce pourtour est d B(P ) B(R) . Pour P et R `a linterieur du solenode,
le courant `a travers le rectangle est nul et B(P ) = B(R). Le champ magnetique est donc uniforme `a
linterieur du solenode.
93

d) On montre de meme que le champ magnetique est uniforme `a lexterieur du solenode. Si on seloigne
du solenode jusqu` ~ tend vers 0. On suppose que cela reste
a linfini, pour un solenode fini, le champ B
~ ~ =0
vrai pour le solenode infini. Comme B = Cte `a lexterieur du solenode cette constante est nulle et B
`a lexterieur du solenode.

e) Orientons ~u et I selon la r`egle du tire-bouchon et soit B~u le champ magnetique `a linterieur du


solenode. Le rectangle P QRS traversant le solenode, P `a linterieur du solenode, S `a lexterieur du
solenode, le theor`eme dAmp`ere donne Bd = 0 nId, do`u B = 0 nI et B ~ = 0 nI~u.

f ) Le flux du champ magnetique ` a travers le solenode est s = Ba2 nb = 0 n2 a2 bI. Le coefficient


dauto-induction du solenode est L = s /I = 0 n2 a2 b. Le coefficient dauto-induction par unite de
longueur du solenode est = L/b = 0 n2 a2 .

~ a
2) Le flux de B ` travers la bobine est

= Br2 N = 0 nr2 N I. (7)

Le signe est si les sens denroulements du solenode et de la bobine sont opposes.



Le rapport M = = 0 nN r2 sappelle coefficient de mutuelle-induction solenode-bobine.
I

~ a
3.a) Le flux de B ` travers la bobine est

= M I0 sin t = 0 nN r2 I0 sin t. (8)

b) La f.e.m. dinduction dans la bobine est

d
e= = M I0 cos t = 0 nN r2 I0 cos t. (9)
dt

c) Le courant qui circule dans la bobine est

e M I0 0 nN r2 I0
Ib = = cos t = cos t. (10)
R R R
Cest un courant sinusodal damplitude
|M |I0
Ib0 = (11)
R

d) La circulation du champ electrique induit Ei le long du fil de la bobine est la f.e.m. e. Linvariance
du syst`eme par rotation autour de son axe implique que Ei est le meme en tout point du fil. On a donc
e = 2rN Ei et
e 0 nrI0
Ei = = cos t. (12)
2rN 2

4) Envoyer un courant I(t) de la forme (1) dans le solenode et mesurer son amplitude I0 et sa pulsation
. Mesurer lamplitude Ib0 du courant Ib (t) qui circule dans la bobine. Dapr`es (11), le coefficient de
mutuelle-induction solenode-bobine sobtient par
RIb0
|M | = . (13)
I0

On a M > 0 (resp. M < 0) si I(t) et Ib (t) sont en opposition de phase (resp. en phase).
94 8. EXAMEN (2E SESSION) 2003-2004

C. Interf
erom`
etre de Michelson

M1

1) La plaque de bakelite se comporte comme un milieu dindice n.


Les lois de Descartes sappliquent aux rayons comme EC qui donne
un rayon reflechi CM1 et un rayon transmis CD. La premi`ere onde C
E M2
suit le chemin ECM1 CF R et la deuxi`eme le chemin ECDM2 DR.
F D
Chaque chemin comporte une reflexion sur la plaque bakelite, une
reflexion sur un plan reflecteur et une transmission `a travers la plaque plaque de
baklite B
bakelite qui se font dans les memes conditions. Cela justifie que les
deux ondes ont la meme amplitude. R

2
Si au point E, la vibration est en cos(t 0 ), les dephasages en R sont 1 = 0 + longueur optique

2
du chemin ECM1 CF R et 2 = 0 + longueur optique du chemin ECDM2 DR. Le segment CM1 ,

parcouru deux fois par londe 1, est le seul qui varie. Sa longueur etant X+Cte, on a

4
= 1 2 = X + o`
u = et = Cte. (14)

2) La somme des representations


 complexes E1 = aei(t1 ) et E2 = aei(t2 ) donne E = Aei(t+)
i i1 i2
o`
u Ae = a e +e est un nombre complexe independant de t. En prenant sa partie reelle on
~ ~
obtient que la somme E = E1 + E ~ 2 est une vibration harmonique de la forme

~ = A cos(t + ) ~uy .
E (15)

2 2
3) Lintensite est I = A2 = Aei = a2 ei1 + ei2
  2 1 2
I = a2 ei(1 2 )/2 + ei(1 2 )/2 ei(1 +2 )/2 = 4a2 cos2

. On a donc
2


I = 4a2 cos2 = 2a2 (1 + cos ). (16)
2

On peut mesurer I avec un detecteur constitue par une diode capable de redresser le courant U.H.F.
~ de londe. Le signal recu par ce detecteur peut sobserver sur
produit par le champ electrique oscillant E
un millivoltm`etre digital comme en TP.

I
  
2 2
2 4X 4a2
4) Lintensite I = 2a 1 + cos + est periodique


de periode en X. On peut mesurer la longueur donde
2
en mesurant cette periode. Noter que I varie entre la valeur
minimale 0 et la valeur maximale 4a2 . 0
X
95

9. Premier partiel 2004-2005


Enonc
e
15 octobre 2004
duree : 1 heures 30
sans documents, calculatrices autorisees

Bar`eme indicatif : A=9, B=11.

A. Onde triangulaire
y

Q a

P R B
x a O a b x

Fig. 23 Corde a
` linstant t = 0.

Une corde homog`ene tr`es longue, de masse lineique = 10 g m1 , est tendue le long de laxe x Ox. Une
de ses extremites est fixee au point B dabscisse b (b = 1 m). On considerera que la corde au repos occupe
le demi-axe x B des points dabscisse x : < x b.
` linstant t = 0 une onde transverse progressive de forme triangulaire P QR se propage sur cette corde
A
sans se deformer vers les x croissants (cf. figure 23).
La forme de la corde `
a linstant t = 0 est donnee par la fonction

(a + x)
si a x 0
s0 (x) = (a x) si 0 x a (1)


0 sinon

o`
u a = 20 cm et 1.
La vitesse de propagation de ces ondes est c = 10 m s1 .

1) Questions de cours (utiles pour la resolution du probl`eme)

Enoncer le theor`eme de dAlembert donnant la forme generale du deplacement s(x, t) des ondes dune
corde vibrante.
96 9. PREMIER PARTIEL 2004-2005

Quelle est la signification physique de la condition

s(b, t) = 0 t ? (2)

2) Donner la forme generale dune onde progressive qui se propage vers la droite.
En deduire une expression du deplacement s(x, t) des points de la corde valable aux temps precedant
linstant o`
u londe commence ` a se reflechir en B. On conseille decrire cette expression en termes de la
fonction s0 definie par lequation (1).

3) En utilisant lexpression de s(x, t) obtenue, calculer la vitesse v0 (x) du point dabscisse x de la corde
a linstant t = 0.
`
Tracer la courbe de v0 (x) en fonction de x.

4) Lenergie cinetique totale de la corde `


a linstant t est donnee par
Z b  2
1 s
Ec (t) = (x, t) dx. (3)
2 t

Justifier cette formule et calculer sa valeur Ec (0) `a linstant t = 0 en fonction de a, , et c.


Application numerique. Calculer Ec (0) pour = 0,05.

5) On admettra que lenergie potentielle de la corde `a linstant t est

Ep (t) = T0 L (4)

o`
u T0 est la tension de la corde et L lallongement de la corde par rapport `a sa position de repos.
`
A partir de la figure 23, determiner lallongement L `a linstant t = 0 en fonction de a et . On donnera
une expression approchee simple de L limitee `a lordre le plus bas en .
En deduire la valeur approchee correspondante de lenergie potentielle Ep (0) `a linstant t = 0 et comparer
sa valeur `a Ec (0).

6) Pour un temps quelconque londe a la forme

s(x, t) = s0 (x ct) s0 (x + ct 2b). (5)

Justifier le plus compl`etement possible cette expression.

a linstant t1 = 0,2 s. On marquera sur la figure les axes x Ox et Oy,


7) Representer lallure de la corde `
le point B ainsi que suffisamment dindications (cotes) pour determiner compl`etement la forme de la
corde.
Dans quel sens londe se propage-t-elle `
a cet instant?

8) Representer lallure de la corde `


a linstant t2 = 0,1 s.
Quelle est lenergie mecanique de la corde `
a cet instant?
97

B. Ondes dune membrane


On etudie les ondes transverses dune membrane de tambour. Au repos la membrane est tendue dans le
plan Oxy dun rep`ere cartesien Oxyz. En presence dune onde le point M de coordonnees (x, y, 0) de
la membrane au repos se deplace perpendiculairement au plan Oxy ; ses coordonnees `a linstant t sont
alors (x, y, s(x, y, t)).
La fonction s(x, y, t) satisfait `
a lequation donde `a deux dimensions

2s 2s 1 2s
+ =0 (6)
x2 y 2 u2 t2
o`
u u est une constante.

1) En utilisant lequation (6), determiner la dimension de u.


Expliquer le mot transverse dans onde transverse .

2) On consid`ere la fonction
s1 (x, y, t) = A sin(kx x + ky y t) (7)
o`
u A, kx , ky et sont des constantes reelles (avec > 0 et A > 0).

Ecrire la representation complexe s1 (x, y, t) de s1 (x, y, t).

3) Montrer que la fonction (7) est une solution de lequation donde (6) (ecrite avec s1 au lieu de s) `a
condition que kx , ky et verifient la relation
q
= u kx2 + ky2 . (8)

Verifier lhomogeneite de lequation (8).


Decrire les caracteristiques de londe (7) (est-elle progressive? stationnaire? plane? harmonique? quelle
est sa direction, son sens et sa vitesse de propagation?).

4) La membrane de tambour est tendue sur un cadre rec- y


tangulaire de dimension a b (cf. figure 24). Au repos, la b
membrane a la forme du rectangle du plan Oxy

0 x a, 0 y b. (9)

Les bords de la membrane sont maintenus fixes, de sorte


quune onde s(x, y, t) de la membrane doit satisfaire aux
conditions aux limites
O a x
s(0, y, t) = 0, s(x, 0, t) = 0, (10)
s(a, y, t) = 0, s(x, b, t) = 0 (11) Fig. 24 Forme de la membrane.
pour tout x [0, a], tout y [0, b] et a` tout instant t.

On consid`ere une onde de la forme

s(x, y, t) = A sin(kx x + x ) sin(ky y + y ) sin(t + ) (12)

o`
u A > 0, kx > 0, ky > 0, > 0, x , y et sont des constantes reelles.
Montrer que s(x, y, t) est une solution de lequation donde (6) `a condition que kx , ky et verifient la
relation (8).
98 9. PREMIER PARTIEL 2004-2005

5) Montrer que lon doit avoir sin(kx a) = 0 et sin(ky b) = 0.


En deduire que kx et ky ne peuvent prendre que certaines valeurs et determiner toutes ces valeurs.

6) Quelles sont alors les frequences des ondes correspondantes? Determiner, en fonction de u, a et b la
frequence 0 qui est la plus petite de ces frequences.

7) On donne a = 20 cm, b = 15 cm et u = 60 m s1 . Determiner numeriquement 0 ainsi que les quatre


frequences plus basses suivantes 1 , 2 , 3 , 4 (0 < 1 < 2 < 3 < 4 ) des ondes stationnaires de la
membrane.

8) Quels sont les nuds de londe stationnaire de frequence 4 ? Les representer sur un schema.
99

Corrig
e
A. Onde triangulaire

1) Th
eor`
eme de dAlembert (1747) La solution generale de lequation de la corde vibrante

1 2s 2s
=0
c2 t2 x2
est la superposition de deux ondes progressives arbitraires allant en sens opposes

s(x, t) = F (x ct) + G(x + ct). (13)

Lequation (2) signifie que le point dabscisse x = b est immobile ce qui exprime la condition que la corde
est fixee au point B.

2) On peut utiliser lexpression


s(x, t) = s0 (x ct). (14)
Cette expression represente une onde progressive se propageant vers les x croissants (elle est de la
forme (13), avec G 0 et F s0 ). Cette expression verifie s(x, 0) = s0 (x) et s(b, t) = s0 (b ct) = 0
pour b ct a soit pour t tb = (b a)/c, tb = 0,08 s etant linstant o`
u londe commence `a se reflechir
en B.

3) La derivee de la fonction s0 (u) est




si a < u < 0

s0 (u) = si 0 < u < a (15)


0 si u < a ou a < u.

s
On en deduit v0 (x) = = cs0 (x ct)|t=0 soit
t t=0

c
si a < x < 0
v0 (x) = c si 0 < x < a (16)


0 si x < a ou a < x.

La vitesse v0 (x) nest pas definie mathematiquement aux points anguleux x = a et x = 0. Physiquement,
v0 (x) est continue et varie tr`es vite au voisinage de ces points.

v0 (x)

B
x a O a b x

Fig. 25 Vitesse v0 (x) (`


a linstant t = 0).
100 9. PREMIER PARTIEL 2004-2005

s
4) Lelement de corde entre les abscisses x et x + dx a la masse dx et la vitesse (x, t). Son energie
 2 t
dx s
cinetique est (x, t) . Lequation (3) pour lenergie cinetique totale de la corde `a linstant t
2 t
sobtient par sommation de ces energies cinetiques elementaires sur toute la longueur de la corde (x
variant de `a b).
Lenergie cinetique totale de la corde `
a linstant t = 0 vaut
Z b Z a
1 2 1
Ec (0) = v (x) dx = (c)2 dx = a2 c2 (17)
2 0 a 2

soit
Ec (0) = a2 c2 = 5 104 J. (18)


5) La longueur de chacun des segments P Q et QR etant a 1 + 2 et celle du segment P R etant 2a,
lallongement L de la corde `
a linstant t = 0 est
 
p  2
L = 2a 2
1 + 1 2a 1 + 1 = a2 . (19)
2

a linstant t = 0 est donc, puisque T0 = c2 ,


Lenergie potentielle `

Ep (0) = T0 a2 = a2 c2 (20)

qui est egale `a Ec (0).

6) Pour justifier le plus compl`etement possible lexpression donnee nous allons montrer quon arrive `a
cette forme en utilisant le theor`eme de dAlembert, les conditions initiales et les conditions aux limites.
Dapr`es les questions 1 et 2, londe est de la forme

s(x, t) = s0 (x ct) + G(x + ct). (21)

Londe progressive G(x + ct) se propageant vers les x decroissants napparat quapr`es linstant tb . La
condition (2) donne s0 (b ct) + G(b + ct) = 0 t, soit, posant ct = u b, G(u) = s0 (2b u) =
s0 (u 2b) u (on utilise le fait que s0 est paire).
Londe est est donc, `
a tous les instants t, donnee par

s(x, t) = s0 (x ct) s0 (x + ct 2b). (22)

Cest la superposition de londe progressive s0 (x ct) allant vers la droite et de londe progressive
s0 (x + ct 2b) se propageant vers la gauche. Cette deuxi`eme onde napparat quapr`es linstant tb
(s0 (x + ct 2b) = 0 pour x b et t tb car x + ct 2b b + c(b a)/c 2b = a). Physiquement cest
la reflexion, avec changement de signe, de londe incidente s0 (x ct). Londe incidente disparat apr`es
linstant tb = (b + a)/c = 0,12 s o`
u la reflexion sest terminee (s0 (x ct) = 0 pour x b et t tb ).

7) A` linstant t1 = 0,2 s (t1 > t ) londe se propage vers la gauche apr`es setre reflechie en B.
b
Algebriquement, le terme s0 (x ct1 ) est nul pour x b et londe est decrite par (on utilise ct1 = 2b)

s(x, t1 ) = s0 (x + ct1 2b) = s0 (x). (23)


101

La forme de la corde `
a linstant t1 est donnee par la figure 26.
y

a a B
x O b x

Fig. 26 Corde a
` linstant t1 .

8) Comme ct2 = b, lequation (22) donne `a linstant t2 = 0,1 s

s(x, t2 ) = s0 (x ct2 ) s0 (x + ct2 2b) = s0 (x b) s0 (x b) = 0. (24)

Cette equation sinterpr`ete de la facon suivante. Londe sur la corde est la superposition de deux ondes
progressives, lune allant vers la droite (representee par la figure 23 `a linstant t = 0) et lautre allant vers
la gauche (representee par la figure 26 `a linstant t1 ). Ces ondes sont virtuelles pour x > b. A ` linstant t2 ,
ces deux ondes sont centrees en B et sannulent. La corde est alors dans sa position de repos (figure 27).
Londe se propage dans les deux sens a` cet instant.
y

B
x O b x

Fig. 27 Corde a
` linstant t2 .

Lenergie mecanique Em est la somme de lenergie cinetique et lenergie potentielle de la corde. Cest la
meme aux instants t = 0 et t2 par suite de la conservation de lenergie mecanique de la corde. Elle vaut
donc :
Em = Ec (0) + Ep (0) = Ec (t2 ) + Ep (t2 ) = 2a2 c2 . (25)

On peut aussi calculer cette energie mecanique comme suit.


a) On calcule la vitesse v2 (x) du point dabscisse x de la corde `a linstant t2 = 0,1 s.
Elle sobtient en derivant (22) par rapport `a t.

v2 (x) = cs0 (x ct2 ) cs0 (x + ct2 2b) = 2cs0 (x b) (26)

soit
2c
si b a < x < b
v2 (x) = indefini si x = b a (27)


0 sinon.

v2 (x)

B
x O ba b x

2c

Fig. 28 Vitesse v2 (x) (`


a linstant t2 ).
102 9. PREMIER PARTIEL 2004-2005

b) On determine lenergie cinetique et lenergie potentielle de la corde `a linstant t2 en utilisant les


expressions (3) et (4).
Lenergie cinetique totale de la corde `
a linstant t2 vaut
Z b Z b
1 2 1
Ec (t2 ) = v (x) dx = (2c)2 dx = 2a2 c2 . (28)
2 2 ba 2

La corde netant pas allongee, son energie potentielle est nulle. On a obtenu

Ec (t2 ) = 2a2 c2 et Ep (t2 ) = 0 (29)

ce qui redonne bien (25).

B. Ondes dune membrane

2s 1 2s
1) Legalite des dimensions des termes 2
et 2 2 de lequation (6) donne
x u t
 
u = LT 1 (30)

cest-`a-dire que u est homog`ene `


a une vitesse.
Le mot transverse dans onde transverse signifie que le mouvement des points materiels de la
membrane est perpendiculaire `
a la direction de propagation de londe (qui est necessairement contenue
dans le plan Oxy). Ce mouvement est en effet parall`ele `a Oz.

2) La representation complexe de s1 (x, y, t) est

s1 (x, y, t) = Aiei(kx x+ky yt) . (31)

3) On calcule, en utilisant lequation (7) ou (31),

2 s1 2 s1 2 s1
= kx2 s1 , = ky2 s1 et = 2 s1 .
x2 y 2 t2

Lequation (6) secrit  


2 2 2
kx ky s1 = 0.
u2
Puisque s1 6= 0 et > 0 il en resulte la relation demandee
q
=u kx2 + ky2 . (32)

     
Elle est homog`ene : = T 1 , kx = ky = L1 et (30) donnent bien
  h q i
= u kx2 + ky2 .

Londe (7) est une onde plane, harmonique, progressive. Elle nest pas stationnaire. Elle se propage
dans la direction et le sens du vecteur ~k = (kx , ky , 0). Comme = u k, sa vitesse de propagation est
u.
103

4) On calcule, en utilisant lequation (12),


2s 2s 2s
= kx2 s, = ky2 s et = 2 s.
x2 y 2 t2
Ces relations sont semblables ` a celles de la question 3 et impliquent de meme que s(x, y, t) est une
solution de lequation donde (6) `
a condition que kx , ky et verifient la relation (8).

5) La condition s(0, y, t) = 0 y,t secrit A sin(x ) sin(ky y + y ) sin(t + ) = 0 y,t. Comme A 6= 0,


cela implique sin x = 0. On peut prendre x = 0 , les autres valeurs x = n (n Z) redonnant la
meme onde, eventuellement en changeant en + .
De meme, la condition s(x, 0, t) = 0 x,t conduit `a y = 0.
Londe est donc de la forme
s(x, y, t) = A sin(kx x) sin(ky y) sin(t + ). (33)
La condition s(a, y, t) = 0 y,t secrit A sin(kx a) sin(ky y) sin(t + ) = 0 y,t et implique

sin(kx a) = 0. (34)

De meme, la condition s(x, b, t) = 0 x,t donne

sin(ky b) = 0. (35)

On en deduit que kx a = n, ky b = m avec n et m entiers strictement positifs (dapr`es lhypoth`ese


kx > 0, ky > 0). Cela montre que kx et ky ne peuvent prendre que les valeurs
n m
kx = , ky = (n Z+ , m Z+ ). (36)
a b
Remarque. Si on enl`eve lhypoth`ese kx > 0, ky > 0, on nobtient pas de nouvelles solutions : pour
kx = 0 ou ky = 0 lexpression (33) est nulle ; les valeurs
n m
kx = , ky =
a b
pour les quatre choix de signes donnent la meme expression (33), eventuellement en changeant en
+ .

uq 2
6) La frequence du mode (n, m) correspondant aux valeurs (36) est nm = = kx + ky2 soit
2 2
r
u n2 m2
nm = 2
+ 2. (37)
2 a b
La plus petite de ces frequences sobtient pour n = m = 1 :
r
u 1 1
0 = 2
+ 2. (38)
2 a b

q p
60 n2 m2
7) La frequence en hertz est donnee par nm = 2 (0,15)2 + (0,2)2 = 50 (4n)2 + (3m)2 . Le tableau
donne les plus basses frequences des modes

n m (4n)2 + (3m)2
1 1 25 0 = 250 Hz
1 2 52 1 = 361 Hz
2 1 73 2 = 427 Hz
1 3 97 3 = 492 Hz
2 2 100 4 = 500 Hz
104 9. PREMIER PARTIEL 2004-2005

y
8) Londe de frequence 4 est le mode n = m = 2 qui est de
b
la forme
   
2x 2y
s(x, y, t) = A sin sin sin(t + ). (39)
a b b
2

Les nuds de cette onde stationnaire sont les points  (x, y)


 tels
2x
que s(x, y, t) = 0 t, cest-`
a-dire qui verifient sin =0
  a O a a x
2y 2
ou sin = 0. Ce qui donne x = 0, x = a/2, x = a,
b
y = 0, y = b/2 ou y = b. Les nuds occupent donc les cotes Fig. 29 Lignes de nuds.
et les medianes du rectangle (cf. figure 29).
105

10. Deuxi`
eme partiel 2004-2005


Enonc
e
24 novembre 2004
duree : 3 heures
sans documents, calculatrices autorisees

Les parties A, B et C sont independantes.


Bar`eme indicatif : A=3, B=2, C I=3, C II=12.
Valeurs numeriques des constantes physiques
Vitesse de la lumi`ere dans le vide : c = 3 108 m s1 ;
Constante de Planck : h = 6,63 1034 J s ;
Charge electrique elementaire : e = 1,6 1019 C ;
Permittivite du vide : 0 = 8,85 1012 F m1 ;
Permeabilite du vide : 0 = 4 107 H m1 .

A. Questions de cours

1) Donner sans demonstration quatre proprietes dun champ de vecteur conservatif.


Donner sans demonstration quatre proprietes dun champ de vecteur `a flux conservatif.

2) Donner lequation locale exprimant la conservation de lenergie electromagnetique.


Commenter la signification physique des differents termes de cette equation.
Donner lequation locale exprimant la conservation de la charge. La comparer `a lequation precedente.

3) Pour chacune des ondes electromagnetiques dont la longueur donde dans le vide est
a) 6 1010 m
b) 6 107 m
c) 600 m
calculer la frequence et lenergie W dun photon (en eV) et indiquer le type de londe.

B. Effet Doppler
Un TGV dont le klaxon est en fonctionnement continu passe `a grande vitesse devant un chef de gare. La
tonalite du son percu par le chef de gare alors que le train seloigne diff`ere dune octave de la tonalite
percue durant la phase dapproche du train. Le son est-il devenu plus grave ou plus aigu? Pourquoi?
106 `
10. DEUXIEME PARTIEL 2004-2005

Application numerique La vitesse du son est cs = 340 m s1 . Quelle est la vitesse v, en km h1 , du TGV?

Rappels

En musique, un saut dune octave correspond `a un doublement de la frequence du son.


Effet Doppler pour un milieu immobile, une source S de vitesse ~v1 et un recepteur M de vitesse ~v2 :

1 ~v2 ~u/cs SM
= 0 avec ~u = . (1)
1 ~v1 ~u/cs SM

z

C. Etude dun condensateur
a
On consid`ere un condensateur plan depaisseur b = 1 mm dont q B
les armatures sont des disques de rayon a = 20 cm. On utilise
un syst`eme de coordonnees cylindriques (z, r, ) dorigine O au b O
centre du condensateur. Laxe z Oz, qui est laxe du conden-
sateur, coupe les armatures inferieure et superieure aux points +q A
A (z = b/2) et B (z = b/2) respectivement. On appelle
interieur du condensateur le volume cylindrique |z| < b/2, r a.
z
Larmature inferieure porte la charge q et larmature superieure
la charge opposee. Fig. 30 Condensateur plan.

I. Cas de l
electrostatique
Dans cette question, le condensateur est isole et `a lequilibre electrostatique.

1) Justifier soigneusement pourquoi on peut considerer que le champ electrique est uniforme

~ r, ) = E~uz
E(z, (2)

a linterieur du condensateur.
`
Exprimer q en fonction de E. Nota : dans cette question et les suivantes, la reponse peut aussi comporter
les dimensions du condensateur et les constantes physiques donnees en tete de lepreuve.

2) Calculer la difference de potentiel U = UA UB entre les points A et B. On exprimera la reponse en


fonction de E.

3) En deduire la capacite C (valeurs litterales et numeriques) du condensateur.

4) Calculer lenergie electrique du condensateur en integrant la densite denergie electromagnetique `a


linterieur du condensateur. On exprimera la reponse en fonction de E.

5) Le champ electrique maximum possible dans lair est Em = 3 106 V m1 . Au del` a lair est ionise et
devient conducteur par decharges. Quelle est numeriquement lenergie electrique maximum qui peut etre
emmagasinee dans le condensateur? Que vaut alors la difference de potentiel U ?
107

II. R
egime variable quasistationnaire
` partir de maintenant on consid`ere un regime variable tel que
A
le champ electromagnetique varie sinusodalement avec la pulsation ;
la repartition des charges et des courants dans les armatures du condensateur est invariante dans
la symetrie par rapport `a tout plan contenant laxe z Oz.
Le champ electrique, `
a linterieur du condensateur, est donne approximativement par

~ = A cos(t)~uz
E (3)

u A = 1000 V m1 est une constante.


o`

1) Le condensateur fait partie dun circuit electrique. Les fils electriques du circuit relies au condensateur
sont situes le long de z A et Bz sur la figure 30. Ils sont parcourus par un courant dintensite I(t) qui est
la meme en tous points des fils (approximation du regime quasistationnaire). Lorientation choisie pour
mesurer I(t) est indiquee par une fl`eche sur le fil Bz.
Determiner lintensite I(t) en fonction de A et .

~ nest pas identiquement nul dans le condensateur. Justifier cette affirmation.


2.a) Le champ magnetique B

b) Demontrer en utilisant des considerations de symetrie que le champ magnetique est de la forme

~ z, r, ) = B(t, z, r) ~u
B(t, (4)

o`
u B(t, z, r) est une fonction independante de .

C
c) Soit S le disque daxe z Oz, de rayon r et situe `a la cote z. Son
bord est le cercle C = S oriente dans le sens trigonometrique pour S r
un observateur place en z = (cf. figure 31). z
Appliquer le theor`eme dAmp`ere generalise au contour C et `a la
J
u
~z
~ z, r, ) `a linterieur
surface S. En deduire le champ magnetique B(t,
du condensateur (r a) en fonction de A et .

Fig. 31 C et S.

z
d) En utilisant le resultat de la question precedente determiner a
~ sur la surface laterale du condensateur. B
le champ B C
b

e) Soit C le cercle daxe z Oz, de rayon a et situe `a la cote z et S A


une surface de bord C qui ne passe pas `a linterieur du conden-
sateur (cf. figure 32). En appliquant le theor`eme dAmp`ere
generalise au contour C et `a la surface S , obtenir une nouvelle S
z
~
expression du champ B sur la surface laterale du condensateur.
Fig. 32 C et S .

On supposera que le champ electrique est nul `a lexterieur du condensateur.


Retrouve-t-on la meme valeur qu`a la question precedente?
108 `
10. DEUXIEME PARTIEL 2004-2005

3.a) Determiner le vecteur de Poynting P~ dans le condensateur en fonction de A et .

b) Calculer le flux p du vecteur de Poynting entrant `a travers le cylindre ferme qui delimite le conden-
sateur. Montrer que ce flux sexprime simplement en fonction de la capacite C, de la charge q et du
courant I.

c) En utilisant les resultats des questions I.4) et II.3.b) faire le bilan denergie du condensateur.

z
` partir de cette question, on utilisera la representation complexe
4) A r
B C
~ B)
(E, ~ du champ electromagnetique.
N
~
u
b O
4.a) Soient les points C = (b/2, r,0) et D = (b/2, r,0). On designe
par la surface du rectangle ABCD orientee suivant ~u (cf. fi- A D

gure 33). Son bord = forme un contour oriente ferme qui suit z

les c
otes du rectangle ABCD.
Fig. 33 et .
t) du vecteur B
Calculer, pour 0 r a, le flux (r, ~ `a travers la
surface en fonction de A et .


b) Enoncer la loi de Faraday appliquee au circuit .

c) Verifier que champ electrique donne par lequation (3) est incompatible avec cette loi.

5.a) Pour que la loi de Faraday soit satisfaite, on va corriger le champ electrique (3). On le remplace par
le champ electrique

~ = E
E ~0 + E
~1 avec ~0 = Aeit ~uz
E et ~1 = Af (r) eit ~uz
E (5)

o` ~0 est le champ electrique au centre du


u f (r) est une fonction qui sannule pour r = 0 (le champ E
condensateur).
Determiner, pour 0 r a, la fonction f (r) telle que la loi de Faraday appliquee au circuit soit
satisfaite.

b) Quelle condition la frequence du champ doit-elle verifier pour que lerreur relative commise soit
~1 du champ electrique (5) `a linterieur du condensa-
inferieure `a 1% lorsquon ignore le terme correctif E
teur?

c) La correction du champ electrique effectuee ci-dessus entrane `a son tour une correction du champ
magnetique. La determiner.
109

Corrig
e
A. Questions de cours

~ r , t) un champ de vecteur defini dans tout lespace. Un champ de vecteur conservatif peut
1) Soit E(~
etre defini par une des quatre proprietes suivantes. Le champ de vecteur satisfait alors aux trois autres
proprietes.
~ derive dun potentiel, cest-`a-dire quil existe un champ scalaire (~r, t) tel que
1. Le champ E

~ = .
E ~ (6)

~ le long dun chemin AB ne depend que des positions des extremites A et B du
2. La circulation de E
chemin.
~ le long dun chemin ferme est nulle :
3. La circulation de E
I
E~

dl = 0. (7)

~ est nul :
4. Le rotationnel du champ E
~ E
~ = 0. (8)
~ r , t) un champ de vecteur defini dans tout lespace. Un champ de vecteur `a flux conservatif peut
Soit B(~
etre defini par une des quatre proprietes suivantes. Le champ de vecteur satisfait alors aux trois autres
proprietes.
~ derive dun potentiel vecteur, cest-`a-dire quil existe un champ de vecteur A(~
1. Le champ B ~ r , t) tel
que
~ =
B ~ A.~ (9)
~ a
2. La flux de B ` travers toute surface fermee orientable S est nul :
ZZ
~ ~n dS = 0.
B (10)
S

~ `
3. La flux de B a travers une surface orientable avec bord ne depend pas de la forme de la surface
mais seulement de la forme du bord.
4. La divergence du champ B ~ est nulle :
~ B
~ = 0. (11)

2) Lequation locale exprimant la conservation de lenergie electromagnetique est

u ~ +
~ P~ = 0.
+ ~ E (12)
t

La grandeur
~2
0 E ~2
B
u(~r, t) = + (13)
2 20
est la densite denergie electromagnetique et

~ B
E ~
P~ (~r, t) = (14)
0
110 `
10. DEUXIEME PARTIEL 2004-2005

~ est la puissance par unite de


est le vecteur de Poynting (vecteur courant denergie). La grandeur ~ E
volume fournie par le champ electromagnetique aux charges.
Dans une region sans charges (~ = 0) lequation (12) se reduit `a
u ~ ~
+ P = 0. (15)
t
Lequation locale exprimant la conservation de la charge est lequation de continuite

~
+ ~ = 0 (16)
t

qui a la meme forme que (15). La densite volumique de charge y remplace la densite denergie
electromagnetique u et le courant volumique ~ y remplace le vecteur de Poynting P~ . Il ny a pas de
~ dans (16).
terme analogue `a ~ E

3) Soit la longueur donde dans le vide de londe. Sa frequence est = c/ et lenergie dun photon
est W = h = hc/. Pour le calcul, on peut utiliser hc = 12,4 107 eVm.

(m) (Hz) W (eV) type


a 6 1010 5 1017 2 103 rayons X
b 6 107 5 1014 2 lumi`ere visible (jaune-orange)
c 600 5 105 2 109 radiofrequence (ondes moyennes, MF)

B. Effet Doppler
Soit 0 la frequence du klaxon. La frequence percue par le chef de gare est
0
1 = (17)
1 v/cs
quand le train sapproche et
0
2 = (18)
1 + v/cs
quand le train seloigne. Le son est devenu plus grave (2 < 1 ).
Le rapport des equations (17) et (18) donne

1 1 + v/cs
= (19)
2 1 v/cs
do`
u, en resolvant en v/cs ,
v 1 /2 1 1
= = . (20)
cs 1 /2 + 1 3
La vitesse du TGV est donc
cs
v= = 408 km h1 . (21)
3


C. Etude dun condensateur

I.1) Comme b a, le condensateur est beaucoup plus etendu horizontalement que verticalement. Loin
des bords, il peut etre considere comme invariant par translations horizontales. Il en resulte quon peut
considerer que le champ est le meme que celui cree par deux plans infinis, uniformement charges : le plan
z = b/2 qui porte la charge surfacique = q/a2 et le plan z = b/2 qui porte la charge surfacique .
111

Le champ electrique cree par le premier plan est

~ a = sign(z + b/2)~uz
E (22)
20
avec
1 si u > 0
sign(u) = 1 si u < 0 (23)

indefini si u = 0.
Celui cree par le second plan est
E~ b = sign(z b/2)~uz (24)
20
Le champ resultant de leur superposition,
(
~ =E ~a + E ~b = ~uz si |z| < b/2
E 0 (25)
0 si |z| > b/2

montre que le champ est uniforme et parall`ele `a Oz dans le condensateur. Ce resultat nest quapproche
a des distances de lordre de b des bords des armatures. La charge q = a2 secrit
et nest plus valable `

q = 0 a2 E. (26)

~ le long du segment AB (ou le


2) La difference de potentiel U = UA UB est egale `a la circulation de E
long de toute courbe qui va de A ` a B).
U = Eb. (27)

3) La capacite du condensateur est

q 0 a2
C= = = 1,11 109 F. (28)
U b

4) Lenergie electrique du condensateur est


ZZZ
0 E 2 0 E 2 a2 b
We = d = . (29)
2 2

5) Lenergie electrique maximum qui peut etre emmagasinee dans le condensateur est

2
0 Em a2 b
Wm = = 5 103 J. (30)
2

La difference de potentiel vaut alors U = Em b = 3 kV.

II.1) La charge est dapr`es lequation (26)

q = 0 a2 A cos t. (31)

Avec lorientation choisie


dq
I(t) = = 0 a2 A sin t. (32)
dt
112 `
10. DEUXIEME PARTIEL 2004-2005

2.a) Considerons lequation de Maxwell-Amp`ere

~
~ = 0~ + 0 0 E .
~ B (33)
t
~
Dans le condensateur ~ = 0 et E/t 6 0 pour le champ electrique variable donne par (3). Lequation
de Maxwell-Amp`ere implique donc que ~ B ~ 6 0, et donc B ~ 6 0. Il doit donc exister un champ
~ non identiquement nul dans le condensateur.
magnetique B
Remarque. Lexpression (3) du champ electrique est une approximation telle que ~ E~ = 0 de sorte
que lequation de Maxwell-Faraday nest pas satisfaite (cf. question 4.c). Il serait faux de deduire de
~
lequation de Maxwell-Faraday que B/t = 0.

b) La symetrie du syst`eme par rapport au plan = z OzM implique que le champ magnetique en M
est orthogonal au plan . Avec lhypoth`ese que le champ electromagnetique varie sinusodalement `a la
pulsation , cela montre que le champ est de la forme
~ = B(t, z, r, ) ~u .
B (34)

Un rotation dangle autour de laxe z Oz est le produit de deux symetries par rapport `a deux plans
contenant la droite z Oz et faisant langle /2 entre eux. Cest donc une symetrie du syst`eme et il sen
suit que B(t, z, r, ) = B(t, z, r) ne depend pas de . Cela justifie lequation (4).

c) Lorientation de S compatible avec celle de C correspond au vecteur normal ~uz . Le theor`eme dAmp`ere
generalise secrit I
~
B

dl = 0 (IS + ISd ). (35)
C

Dans cette equation,


le courant de deplacement `
a travers S est
ZZ ~
E
ISd = 0 ~uz dS = 0 r2 A sin(t) ; (36)
S t

le courant de conduction `a travers S est nul IS = 0 ;


~ le long de C est
la circulation de B
I
~
B

dl = 2rB(t, z, r). (37)
C

On en deduit
r2 A sin(t)
2rB(t, z, r) = 0 0 r2 A sin(t) = ,
c2
r
B(t, z, r) = A sin(t) (38)
2c2
et
~ = r A sin(t)~u .
B (39)
2c2

d) Sur le bord lateral du condensateur on a r = a et

~ = a A sin(t)~u .
B (40)
2c2
113

a travers S est nul (E


e) Le courant de deplacement ` ~ = 0 sur S ) et le courant de conduction `a travers
S est I(t). Le theor`eme dAmp`ere generalise pour le contour C et la surface S donne

~ = 0 I(t) ~u
B (41)
2a
qui est identique ` a (40) dapr`es lequation (32). Ce champ est identique au champ cree en r = a par un
fil infini rectiligne parcouru par le courant I(t).
Nota : pour r < a, le champ (39) secrit en fonction du courant I(t) :

~ = 0 rI(t) ~u .
B (42)
2a2

3.a) Le vecteur de Poynting dans le condensateur sobtient `a partir des champs


~ = A cos(t)~uz
E (43)
et (39) :
~ B
E ~ r
P~ = = 2 A2 sin(t) cos(t)~uz ~u (44)
0 2c 0
soit
0 A2 r
P~ = sin(t) cos(t) ~ur . (45)
2

b) Sur le haut et le bas du cylindre (les disques de rayon a de cotes z = b/2), le vecteur de Poynting
est parall`ele `
a la surface et son flux est nul. Sur la surface laterale (r = a) le vecteur de Poynting vaut
0 A2 a
P~ = sin(t) cos(t) ~ur . (46)
2
Son flux entrant (surface laterale orientee suivant ~ur ) est
p = 0 A2 a2 b sin(t) cos(t). (47)
Dapr`es les equations (28), (31) et (32) on a en fonction de la charge q(t), du courant I(t) et de la capacite
du condensateur
I(t)q(t)
p = . (48)
C
Le flux p est la puissance
electromagn
etique fournie au condensateur.

c) Lenergie electromagn etique du condensateur est, dans lapproximation du regime quasista-


tionnaire,
q2
W = . (49)
2C
La derivee de cette energie est bien egale `a p :
dW q dq Iq
= = = p . (50)
dt C dt C

4.a) Utilisons la representation complexe en eit . La representation complexe conjuguee en eit serait
incompatible avec les donnees de la question 5.a). Le champ B ~ est la partie reelle de

~ = ir Aeit ~u .
B (51)
2c2
114 `
10. DEUXIEME PARTIEL 2004-2005

~ r, ) `
Le flux du vecteur B(z, a travers la surface est
Z b/2 Z r Z r
t) =
(r, dz ~ r , ) ~u = ib Aeit
dr B(z, r dr (52)
b/2 0 2c2 0

soit
2
t) = ibr Aeit .
(r, (53)
4c 2

b) La loi de Faraday exprime la circulation du champ electrique E ~ le long du circuit en fonction du flux
~
(r, t) du vecteur B `
a travers la surface :
I
~
E

dl =
d(r, t)
. (54)
dt

On peut aussi lecrire en representation complexe


I t)
~
d(r, 2 br2 it
E dl = = Ae . (55)
dt 4c2

c) Le champ electrique (3) est un champ conservatif (il est homog`ene). Sa circulation le long de tout
circuit est nulle. Ce champ est donc incompatible avec (55).

5.a) La circulation du champ electrique (5) le long de est


I I
E~

dl = ~1
E

dl = bf (r) Aeit . (56)
CD

En comparant `a (55) il vient


2 r2
f (r) = . (57)
4c2
Le champ electrique est
 2 2

~ = A 1 r
E eit ~uz . (58)
4c2

b) On a dans le condensateur
   2
E1 2
= r 2a . (59)
E0 2c 2c
~ 1 est inferieure `a 1% si la frequence
Lerreur relative que lon commet lorsquon ignore le terme correctif E
verifie
0,1c
= 48 MHz. (60)
a

c) Le champ magnetique corrige est de la forme

~ = B
B ~0 + B
~1 avec ~0 = ir Aeit ~u
B et ~1 = b1 (z, r)eit ~u .
B (61)
2c2
115

Le terme B~0 est le champ magnetique (51). Le calcul de la question 2.c) sest base sur le theor`eme
dAmp`ere generalise applique `
a S et C (cf. figure 31). On avait
I ZZ ~0
~0
B
1
dl = 2
E
~uz dS. (62)
C c S t
On a pour les termes correctifs
I ZZ ~1
~1
B
1
dl = 2
E
~uz dS (63)
C c S t
soit Z r
1 iA 3 r2 it iA 3 r4 it
2rb1 (z, r)eit = 2 2r dr e = e . (64)
c 0 4c2 8c4
On a donc
3 3
b1 (z, r) = iA r , (65)
16c4
qui ne depend pas de z, et
 
~ = A ir 2 r2
B 1 eit ~u . (66)
2c2 8c2

Le terme correctif B~ 1 entrane ` ~ 2 qui entrane un terme correctif B


a son tour un terme correctif E ~ 2 qui
~
entrane un terme correctif E3 qui entrane . . .
On obtient ainsi le champ electromagnetique sous forme de series

~ = B
B ~0 + B
~1 + B
~2 + et ~ = E
E ~0 + E
~1 + E
~2 + (67)

avec (theor`eme dAmp`ere generalise applique `a S et C)


Z r
Bn = i n (r )r dr
E (68)
rc2 0
et (loi de Faraday applique `
a et )
Z r
n+1 = i
E n (r )dr .
B (69)
0

n r2n et B
On en tire en remarquant que E n r2n+1

n = 1 ir
B En (70)
(n + 1) 2c2
1 1  r 2
n+1 =
E n =
(ir)B n
E (71)
2(n + 1) (n + 1)2 2c
 
~ = A 1 1  r 2 1  r 4 1  r 6
E + + eit ~uz (72)
(1!)2 2c (2!)2 2c (3!)2 2c
 
~ = iA 1  r  2  r 3 3  r 5
B + eit ~u . (73)
c (1!)2 2c (2!)2 2c (3!)2 2c
Ces champs secrivent `a laide des fonctions de Bessel Jn (u) de 1re esp`ece dordre n (n Z est un entier
positif ou nul) (voir aussi le cours dOptique Ondulatoire, section 5.6). On obtient, avec le developpement
en serie k

 u n X 41 u2
Jn (u) = , (74)
2 k! (k + n)!
k=0
 r   
~ = AJ0
E eit ~uz , ~ = iA J1 r eit ~u
B (75)
c c c
116 `
10. DEUXIEME PARTIEL 2004-2005

On pourra verifier directement que le champ electromagnetique donne par (72) et (73) verifie les equations
de Maxwell locales. Formules utiles pour cette verification :

~ = 1 (rB) ~uz ,
~ B ~ = E ~u
~ E
(76)
r r r
J0 = J1 , (uJ1 ) = uJ0 (77)
117

11. Examen 2004-2005


Enonc
e
11 janvier 2005
duree : 3 heures
sans documents, calculatrices autorisees

Durees conseillees : 2 heures pour la partie A et 1 heure pour la partie B.


Valeurs numeriques des constantes physiques
Vitesse de la lumi`ere dans le vide : c = 3 108 m s1 ;
Permittivite du vide : 0 = 8,85 1012 F m1 ;
Permeabilite du vide : 0 = 4 107 H m1 .

A. Ligne coaxiale
On consid`ere une ligne coaxiale cylindrique daxe Oz, constituee
dun conducteur central plein de rayon r1 ( l ame ) separe par z
le vide dun conducteur creux de rayon interieur r2 et de rayon r1
exterieur r3 ( gaine ).
r2
On utilise les coordonnees cylindriques r, , z et le rep`ere local
r3
associe.
Dans tous les cas envisages dans ce probl`eme, le syst`eme poss`ede
la symetrie cylindrique (invariance des charges et des courants
dans les rotations daxe Oz et dans la symetrie par rapport `a
tout plan contenant laxe Oz).
On negligera les effets de bords aux extremites de la ligne co- O
axiale.
gaine

gaine

vide a
me vide
Pour les applications numeriques on prendra
r1 = 1 mm, r2 = 2 mm et r3 = 3 mm.
Fig. 34 Ligne coaxiale.


I. Electrostatique
Lame et la gaine sont isolees. L
ame porte une charge electrique lineique = Cte (charge par unite de
longueur selon Oz) et la gaine la charge electrique lineique opposee . La ligne coaxiale constitue un
condensateur cylindrique ` a lequilibre electrostatique.

1) Preciser la repartition des charges.


118 11. EXAMEN 2004-2005

Justifier que le champ electrique est de la forme


~ z, ) = E(r)~ur
E(r, (1)

Determiner E(r) pour tout r (0 r < ).


Tracer la courbe E(r) en fonction de r.
Verifier que le condensateur est `
a lequilibre electrostatique.

2) Calculer la difference de potentiel U = Ur1 Ur2 entre l


ame et la gaine.
En deduire la capacite Cl par unite de longueur (valeurs litterales et numeriques) du condensateur.

3) Calculer lenergie electrique Wl du condensateur par unite de longueur en integrant la densite denergie
electrique `a linterieur du condensateur. On exprimera la reponse en fonction de , r1 et r2 .

z
I
II. Magn
etostatique
L
ame est parcourue par un courant dintensite I = Cte, la gaine r1
par un courant dintensite I. I r2

1) Determiner le champ magnetique `


a linterieur de la ligne (pour
r1 < r < r2 ).

2) Calculer lenergie magnetique Wm par unite de longueur gaine O

gaine
contenue dans lespace entre lame et la gaine (r1 < r < r2 ). vide a
me vide
Ll I 2
Le resultat est de la forme Wm = .
2
Fig. 35 Courant constant.

En deduire le coefficient Ll (valeurs litterales et numeriques).


Calculer la valeur litterale du produit Cl Ll .

III. R
egime variable
On consid`ere un regime variable `a symetrie cylindrique. Dans un tel regime le courant et la charge
lineique dans les conducteurs sont fonctions du temps t et de la cote z. On notera I(z, t) lintensite du
courant dans lame, dans le sens des z croissants, `a la cote z, et (z, t) la charge lineique de l
ame `a la
cote z.
On se propose de chercher un champ electromagnetique de la forme
~ z, , t) = E(r, z, t) ~ur ,
E(r, ~ z, , t) = B(r, z, t) ~u .
B(r, (2)
` linterieur du c
A able coaxial (r1 < r < r2 ) on ecrira

~ z, , t) = a(z, t) ~ur ,
E(r, ~ z, , t) = 0 b(z, t) ~u
B(r, (r1 < r < r2 ) (3)
20 r 2 r
o`
u a(z, t) et b(z, t) sont des fonctions de z et t.


1) Ecrire les equations de Maxwell locales dans le vide `a linterieur de la ligne (pour r1 < r < r2 ) et
pour le champ electromagnetique (3).
119

Montrer que deux de ces equations sont satisfaites.


Montrer que les autres equations impliquent que a(z, t) et b(z, t) satisfont `a deux equations couplees aux
derivees partielles.
On utilisera les expressions de la divergence et du rotationnel du champ A ~ = Ar ~ur + A ~u + Az ~uz en
coordonnees cylindriques :
~ A
~ = 1 (rAr ) + 1 A + Az (4)
r r r z
et      
~ A
~ = 1 Az A ~ur + Ar Az ~u + 1 (rA ) Ar ~uz . (5)
r z z r r r

Nota. On ne demande pas decrire les equations de Maxwell dans les regions r > r2 et r < r1 . Vous ne
chercherez pas `
a determiner le champ dans ces regions.


2) Enoncer le theor`eme de Gauss et le theor`eme dAmp`ere generalise en regime variable.
En utilisant ces theor`emes exprimer a(z, t) et b(z, t) en fonction de I(z, t) et (z, t).

3) A ` partir des resultats des deux questions precedentes, ecrire deux equations aux derivees partielles
verifiees par (z, t) et I(z, t). Commenter.

Etablir lequation de propagation du courant I(z, t). Commenter.

IV. Propagation dondes


electromagn
etiques
On consid`ere des ondes harmoniques de pulsation se propageant dans la ligne coaxiale. On utilise la
representation complexe dans laquelle les diverses grandeurs comportent le facteur eit .

1) On pose Z r2
V (z, t) = ~ ~ur dr
E (6)
r1

o` ~ est donne par lequation (3).


u le champ E
Calculer V (z, t) en fonction de a(z, t), r1 et r2 .

2) Montrer que la representation complexe V (z, t) de la fonction V (z, t) sexprime en fonction de la


t) du courant dans l
representation complexe I(z, ame par

i c Z0 I
V = (7)
z
o`
u Z0 est une constante que lon exprimera en fonction de r1 , r2 , 0 et 0 .
Calculer la valeur numerique de Z0 .
Dans la suite, on appelera impedance `
a la cote z le nombre complexe

V
Z(z) = . (8)
I
Limpedance Z(z) depend de londe consideree.

3) On consid`ere londe
t) = I0 ei(tkz)
I(z, avec k = . (9)
c
120 11. EXAMEN 2004-2005


Justifier la relation k = .
c
Londe est-elle une onde progressive?
Determiner limpedance Z(z) pour cette onde.

4) On consid`ere londe
t) = I0 eit cos(kz)
I(z, avec k = . (10)
c

Justifier la relation k = .
c
Londe est-elle une onde progressive?
Determiner limpedance Z(z) pour cette onde.

5) Lame et la gaine dune ligne coaxiale de longueur L = 75 cm sont mises en court-circuit `a une
extremite (z = 0). Un generateur HF de frequence est branche `a son autre extremite (z = L) (cf.
figure 36).

gaine a
me

court-circuit
g
en
erateur
O L z HF

Fig. 36 Ligne de longueur L en court-circuit.

Justifier quil setablit dans la ligne une onde de la forme (10).


On mesure limpedance Z(L) `
a lextremite z = L de la ligne.
Pour quelles valeurs de la frequence mesure-t-on une impedance Z(L) infinie?
Quelle est la valeur numerique de la plus petite frequence pour laquelle cela a lieu?
121

B. Franges dYoung et observation dune


etoile double

trous dYoung lentille L


ecran
x
A1
O1
H

1 M
rayon

z
C O
etoile
l
ion de
direct
A2
vers S O2
K
f
2
rayon

Fig. 37 Interferom`etre des trous dYoung.

Linterferom`etre des trous dYoung est utilise pour observer une etoile (source ponctuelle S tr`es eloignee).
Le dispositif comporte une lentille convergente L daxe Oz et de distance focale f , un ecran dobservation
place dans le plan focal de cette lentille et un ecran opaque perce de deux trous circulaires identiques
O1 et O2 , distants de s, de rayon tr`es petit compare `a s, et symetriques par rapport `a laxe Oz (cf.
figure 37). Un filtre (non represente) permet lobservation de letoile en lumi`ere monochromatique de
longueur donde 0 dans le vide.
Londe lumineuse emise par letoile arrive suivant la direction par rapport `a laxe Oz est diffractee par
les trous O1 et O2 et produit une figure dinterferences sur lecran dobservation.
On designe par rayon 1 (respectivement rayon 2) le rayon lumineux SO1 A1 M (respectivement SO2 A2 M )
issu de letoile qui arrive au point M de lecran dobservation apr`es diffraction par le trou O1 (respecti-
vement O2 ). Le point M est situe ` a labscisse x sur laxe Ox (axe parall`ele `a O1 O2 ).
La lentille est utilisee dans les conditions de Gauss. On supposera donc que 1, |x| f et que la
lentille est parfaitement stigmatique.
Les amplitudes complexes des deux ondes au point M secrivent

A1 = Aei1 (x) , A2 = Aei2 (x) (11)

o`
u A est une constante reel et 1 (x) et 2 (x) sont des fonctions reelles dependant de x. Les champs
~ 1 et E
electriques correspondants, E ~ 2 , sont parall`eles entre-eux et de la forme E
i = Ai eit .

1) On ecrit pour la difference de phase des amplitudes (11)

2(x)
(x) = 2 (x) 1 (x) = . (12)
0

Comment se nomme la quantite (x)?

2) Soient K la projection de O1 sur la droite O2 A2 et H la projection de O2 sur la droite SO1 .


Montrer que (x) secrit 
(x) = n HO1 O2 K (13)
o`
u n est lindice de lair

3) Exprimer (x) en fonction de n, s, , x et f . (Faire un schema justificatif.)


122 11. EXAMEN 2004-2005

4) Determiner les abscisses x pour lesquelles les amplitudes (11) sont en phase.
Repondre en fonction de n, s, , f et 0 .
Definir et calculer linterfrange en fonction de n, s, f et 0 .

5) O`
u se trouve la frange brillante dordre 0 (pour laquelle = 0)?
Comment peut-on determiner experimentalement sa position?

6) Definir leclairement. En quelle unite est-il mesure ?


On admettra que leclairement I(x) au point M est proportionnel au carre du module de lamplitude de
londe resultante en M .
Montrer que leclairement I(x) secrit
(x)
I(x) = I0 cos2 (14)
2
o`
u I0 est une constante puis lexprimer en fonction de I0 , n, s, f , , x et 0 .

7) On utilise le dispositif pour observer une etoile double symetrique assimilee `a deux sources lumineuses
ponctuelles de meme puissance mutuellement incoherentes S et S . La distance angulaire entre les etoiles
est egale `a .
Le point S est sur laxe optique du syst`eme, le point S est dans le plan Oxz et place comme sur la
figure 37.
Determiner leclairement total It (x) produit au point M par letoile double en fonction de n, s, f , , x,
0 et de leclairement maximal I0 produit par la source S (ou S ).

8) Montrer que pour certaines valeurs particuli`eres de la distance s leclairement devient uniforme.
Determiner la plus petite de ces valeurs, notee s1 , en fonction de n, 0 , et .
A. N. Pour n = 1 et 0 = 580 nm on a trouve s1 = 1,32 m. En deduire en seconde darc.

9) On admet quune lunette astronomique de diam`etre D permet de separer les etoiles S et S seulement
si leur distance angulaire est plus grande que 1,220 /D.
Quel est la valeur numerique minimum du diam`etre D qui permettrait de separer les composantes S et
S de letoile double?
Commenter linteret du dispositif interferentiel etudie.
123

Corrig
e
A. Ligne coaxiale

I.1) Les charges dun conducteur ` a lequilibre se repartissent sur sa surface. Les hypoth`eses de lenonce
(symetrie cylindrique et = Cte) impliquent que la charge portee par l ame est repartie uniformement
sur sa surface. La charge portee par la gaine est repartie uniformement sur sa surface interieure.
On neglige les effets de bords. Cela signifie quon consid`ere que le syst`eme est egalement invariant dans
les translations parall`eles `
a Oz et dans la symetrie par rapport `a tout plan perpendiculaire `a Oz.
Pour tout point M (r > 0), le plan OzM et le plan perpendiculaire `a Oz passant par M sont des plans
de symetrie. Il en resulte que le champ electrique au point M est parall`ele `a ces deux plans et donc dirige
suivant leur intersection : E ~ = E(r, z, )~ur .
La symetrie par translation et par rotation implique que E(r, z, ) = E(r) ne depend pas de z et de .
Appliquons le theor`eme de Gauss ` a un cylindre daxe Oz, de
rayon r et de hauteur unite. Le flux du champ E ~ sortant de ce E(r)
cylindre est 2rE(r).
20 r1
La charge `
a linterieur du cylindre est nulle pour r < r1 ou r > r2
et pour r2 > r > r1 . On a donc

20 r2

~ ~ur si r2 > r > r1


E(r, z, ) = 2 0r (15)
0 si r < r1 ou r > r2 .
0 r1 r2 r
Le condensateur est bien ` a lequilibre electrostatique : le champ
electrique est nul dans les conducteurs.
Fig. 38 Courbe E(r).

2) La difference de potentiel est egale `a la circulation de E ~ le long dun segment de droite radial qui va
de l
ame `
a la gaine :
Z r2 Z r2
ln(r2 /r1 )
U= E(r)dr = dr = . (16)
r1 r1 2 0 r 20
On en deduit la capacite Cl = /U par unite de longueur

20
Cl = = 80 pF m1 . (17)
ln(r2 /r1 )

0 E 2 2
3) La densite denergie electrique `
a linterieur du condensateur est u = = 2 . En lintegrant
2 8 0 r2
sur un volume de hauteur unite on obtient
Z r2 Z r2
2 2 ln(r2 /r1 )
Wl = 2 rdr u = dr = . (18)
r1 r1 40 r 40

2
On a la relation Wl = .
2Cl
124 11. EXAMEN 2004-2005

II.1) Pour tout point M (r > 0), le plan OzM est un plan de symetrie. Il en resulte que le champ
magnetique au point M est perpendiculaire ` ~ = B(r, z, )~u .
a ce plan : B
La symetrie par translation et par rotation implique que B(r, z, ) = B(r) ne depend pas de z et de .
Le theor`eme dAmp`ere applique `
a un cercle daxe Oz et de rayon r (avec r2 > r > r1 ) donne

~ z, ) = 0 I ~u .
B(r, (19)
2 r

Remarque. On ne connat pas la repartition des courants dans l ame et la gaine (on sait seulement
quelle est `a symetrie cylindrique). On ne sait donc pas calculer le champ magnetique pour r > r2 et
r < r1 sans hypoth`ese supplementaire.

B2 0 I 2
2) La densite denergie magnetique `
a linterieur de la ligne est v = = . En lintegrant sur un
20 8 2 r2
volume de hauteur unite on obtient
Z r2 Z r2
0 I 2 0 I 2 ln(r2 /r1 )
Wm = 2 rdr v = dr = . (20)
r1 r1 4 r 4

Le coefficient Ll est le coefficient dauto-induction par unite de longueur de la ligne (si on neglige lenergie
magnetique dans les conducteurs). Il est mesure en H m1 et vaut

0 ln(r2 /r1 )
Ll = = 138 nH m1 . (21)
2
On calcule
1
Cl Ll = 0 0 = . (22)
c2

III.1) Lequation de Maxwell-Gauss est verifiee ( = 0) :

~ = 1 (rE) = 0.
~ E (23)
r r
Lequation de Maxwell-flux aussi :
~ = 1 B = 0.
~ B (24)
r
Lequation de Maxwell-Amp`ere (~ = 0)
~
E
0 0 ~ B
~ =0 (25)
t
donne
E B 1
0 0 ~ur + ~ur (rB) ~uz = 0 (26)
t z r |r {z }
=0

a b
+ = 0. (27)
t z
Lequation de Maxwell-Faraday
~
~ = B
~ E (28)
t
donne
E 1 E B
~u ~uz = ~u (29)
z r |{z}
t
=0
125

1 a 0 b
= (30)
20 r z 2 r t

a b
+ 0 0 = 0. (31)
z t

2) Appliquons le theor`eme de Gauss `a un cylindre daxe Oz, de rayon r (r2 > r > r1 ) et de hauteur dz.
La charge electrique dans ce cylindre est (z, t) dz. Le theor`eme donne a(z, t) = (z, t).
Appliquons le theor`eme dAmp`ere generalise au disque daxe Oz et de rayon r (avec r2 > r > r1 ). Le
courant de deplacement ` ~ parall`ele au disque) et le courant de conduction `a
a travers le disque est nul (E
travers le disque est I(z, t). Le theor`eme donne b(z, t) = I(z, t).
Le theor`eme de Gauss applique `
a un cylindre daxe Oz, de rayon r > r3 et de hauteur dz donne
g (z, t) = (z, t).
Le theor`eme dAmp`ere generalise applique au disque daxe Oz et de rayon r > r3 donne

Ig (z, t) = I(z, t).

3) Les resultats du 1) secrivent en fonction du courant et de la charge lineique

I I
+ = 0, + 0 0 = 0. (32)
t z z t
La premi`ere equation exprime la conservation de la charge.
On obtient lequation de propagation du courant en eliminant :

2I 2I
2
0 0 2 = 0. (33)
z t

Cest lequation de dAlembert. La vitesse de propagation des ondes est la vitesse de la lumi`ere dans le
1
vide c =
0 0

IV. 1) Le calcul est le meme que celui effectue `a lequation (16) si on remplace par a(z, t) :

a(z, t) ln(r2 /r1 ) a(z, t)


V (z, t) = = . (34)
20 Cl


2) Pour la representation complexe en eit , la derivee est identique `a la multiplication par i. Les
t
equations (34) (o`
u a = ) et (32) donnent

ln(r2 /r1 )
I
V = et +
i = 0. (35)
20 z
On en tire
ln(r2 /r1 ) I
V = i (36)
20 z
126 11. EXAMEN 2004-2005

soit
i c Z0 I
V = (37)
z
avec (c2 0 0 = 1)
r
ln(r2 /r1 ) 0
Z0 = = 42 . (38)
2 0

3) On ecrit que I est solution de lequation de propagation (33) :


 
2 I 2 I 2
2
0 0 2 = k 2 + 2 I0 ei(tkz) = 0 (39)
z t c

La valeur k = correspond ` a une solution de cette equation.
c
Londe est progressive et se propage vers les z croissants.
Limpedance sobtient en utilisant lequation (7)

i c Z0 I k c Z0
V = =
I = Z0 I. (40)
z

Limpedance est constante et vaut Z0 .

4) On ecrit que I est solution de lequation de propagation (33) :


 
2 I 2 I 2
0 0 = k 2 + I0 eit cos(kz) = 0 (41)
z 2 t2 c2

La valeur k = correspond ` a une solution de cette equation.
c
Londe nest pas progressive ; cest une onde stationnaire.
Lequation (7) donne

i c Z0 I i k c Z0
V = = I0 eit sin(kz) = i Z0 I0 eit sin(kz). (42)
z
On en deduit limpedance
Z(z) = i Z0 tan(kz). (43)

5) Londe de frequence la plus generale dans la ligne est la somme dune onde progressive allant vers
les z croissants et dune onde progressive allant vers les z decroissants :

I = I1 ei(tkz) + I2 ei(t+kz) (44)

Lequation (7) donne

i c Z0 I  
V (z, t) = = Z0 I1 ei(tkz) I2 ei(t+kz) . (45)
z

La ligne etant en court-circuit en z = 0, on a V (0, t) = 0 et donc I1 = I2 . Posant I0 = I1 /2, londe est


bien de la forme 
t) = I1 eit eikz + eikz = I0 eit cos(kz).
I(z, (46)
127

Limpedance Z(L) est donnee par lequation (43)


2L
Z(L) = i Z0 tan(kL) = i Z0 tan . (47)
c
Elle est infinie pour les frequences telles que
2L
= (2k + 1) avec k Z (48)
c 2
soit pour
c
= (2k + 1) = (2k + 1) 100 MHz avec k = 1, ,2, 3, . . . avec k Z (49)
4L

La plus petite frequence est = 100 MHz. La ligne est alors de longueur /4 ( = c/ = 3 m).

V. Question non pos


ee
Question On utilisera le champ electromagnetique donne par lequation (3).
Soit le rectangle ABCD du demi-plan = 0 avec
z
A = (r1 , 0, z + dz), B = (r2 , 0, z + dz), r2

D = (r1 , 0, z), C = (r2 , 0, z). r1

A
Calculer la circulation du champ E ~ le long du bord oriente z + dz B

ABCDA du rectangle de deux facons differentes.


En deduire une relation differentielle entre a(z, t) et b(z, t). z C
D
a
me vide gaine
O

Fig. 39 Rectangle ABCD.

R ~ etant nulle le long de BC et DA,


eponse Le calcul direct donne, la circulation de E
ln(r2 /r1 ) a
= V (z + dz, t) V (z, t) = dz. (50)
20 z
Le calcul apr`es transformation en integrale sur la surface du rectangle ABCD orientee selon ~u et
utilisation de lequation de Maxwell-Faraday donne
ZZ ZZ ~ ZZ
~ E
~ ~u dS = B 0 b
= ~u dS = dS
ABCD ABCD t ABCD 2 r t
Z r2
b 0 dr 0 ln(r2 /r1 ) b
= dz = dz (51)
t r1 2 r 2 t
La comparaison des deux calculs donne la relation

a b
+ 0 0 = 0. (52)
z t
identique `
a lequation (31). Cela na rien detonnant : les deux calculs qui conduisent `a cette relation sont
tous deux bases sur lequation de Maxwell-Faraday.

B. Franges dYoung et observation dune


etoile double

1) La quantite est la diff


erence de marche.
128 11. EXAMEN 2004-2005

2) La quantite est la difference des chemins optiques = [SHO1 A1 M ] [SO2 KA2 M ].


Montrons dabord que [O1 A1 M ] = [KA2 M ]. Pour cela, on peut imaginer quon place en M une source
ponctuelle. Cette source produit une onde dont M A1 O1 et M A2 KO2 sont des rayons. A ` gauche de la
lentille, cette onde est plane et les points O1 et K sont situes sur une meme surface donde. En utilisant le
principe de legalite des chemins optiques entre deux surfaces donde, on obtient alors legalite des chemins
optiques [O1 A1 M ] = [KA2 M ]. Nota. Dans lexperience, les rayons O1 A1 M et O2 A2 M correspondent `a
deux ondes differentes, londe diffractee par le trou O1 et londe diffractee par le trou O2 respectivement.
Dans lexperience, les points O1 et K ne sont pas situes sur une meme surface donde.
On a egalite des distances SO2 = SH et des chemins optiques [SO2 ] = [SH] (en admettant que lindice
varie de 1 (vide) `a n (air autour de lappareil) de la meme facon le long des deux rayons). On en deduit
que (x) secrit

(x) = [HO1 ] [O2 K] = n HO1 O2 K . (53)

La longueur O2 K etant comptee algebriquement dans le sens de la propagation de la lumi`ere (O2 K > 0
sur la figure 37), cette expression reste valable quand x < 0 (on a alors (O2 K < 0).

3)

trous dYoung lentille L


ecran
x
A1
O1
H

1 M
rayon

z
C O
etoile
l
ir e c t ion de
d
A2
vers S O2
K
f
2
rayon

Fig. 40 Calcul de .

Le point M etant dans le plan focal objet de la lentille L, le rayon O2 A2 est parall`ele `a CM . Il en resulte
legalite des angles = O\ \
2 O1 K = OCM (leurs c otes sont deux `a deux orthogonaux). On a, en tenant
compte de || 1,

O2 K x sx
= u O2 K =
(dans le triangle O1 KO2 ) et = (dans le triangle COM ) do` .
s f f

Langle en O2 du triangle O1 HO2 vaut . On en deduit HO1 s.


La difference de marche vaut donc
 
ns x
(x) = (f x) = ns . (54)
f f

4) Les amplitudes (11) sont en phase pour (x) = k0 avec k Z. Cela a lieu aux abscisses

k0 f
x = f , k Z. (55)
ns
129

Ce sont les points declairement maximal (franges brillantes). Linterfrange i est la distance entre deux
franges brillantes voisines. Lequation (55) secrit x = f ki avec

0 f
i= . (56)
ns

5) La frange brillante dordre 0 passe par le point M0 dabscisse x = f . Elle est parall`ele `a laxe Oy. Le
point M0 est limage de S par la lentille. On peut determiner experimentalement sa position en enlevant
lecran des trous O1 et O2 .
Une autre methode pour determiner la position de la frange brillante dordre 0 consiste `a supprimer le
filtre de longueur donde. La frange brillante dordre 0 est une frange brillante pour toutes les longueurs
donde. En lumi`ere blanche, cest le lieu des points declairement maximum.

6) Leclairement I dun element de surface dS est la puissance electromagnetique moyenne par unite
de surface qui arrive sur dS. Pour dS perpendiculaire `a laxe optique et dans les conditions de Gauss,
I = hP i (P~ est le vecteur de Poynting).
Leclairement est mesure en W m2 .
Lamplitude resultante en x est A = A1 + A2 . Le carre de son module est
2 2 2
i1 (x)
+ Aei2 (x) = A2 1 + ei = A2 ei/2 + ei/2 = 4A2 cos2 .

Ae
2
Leclairement I(x) est donc de la forme
(x)
I(x) = I0 cos2 (57)
2
o`
u I0 est leclairement maximum. En remplacant par 2/0 on obtient

ns(f x)
I(x) = I0 cos2 . (58)
f 0

7) Leclairement du`a letoile double est la somme de deux expressions de la forme (58), une avec = 0
pour S et lautre identique ` a (58) pour S.
 
nsx ns(x f )
It (x) = I0 cos2 + cos2 (59)
f 0 f 0

8) Leclairement It (x) est, `


a une constante additive pr`es, la somme de deux sinusodes dephasees de
meme amplitude :  
I0 2nsx 2ns(x f )
It (x) = 2 + cos + cos (60)
2 f 0 f 0
Cette somme devient independante de x lorsque les sinusodes sont en opposition de phase, cest-`a-dire
lorsque
2ns
= (2k + 1), k Z. (61)
0
Leclairement est donc uniforme pour
(2k + 1)0
s= , k entier positif ou nul. (62)
2n
130 11. EXAMEN 2004-2005

La plus petite de ces valeurs est


0
s1 = . (63)
2n

0
A. N. = = 0,0906
2ns1

9) Le diam`etre minimum qui permettrait de separer les composantes S et S de letoile double est

1,220
D= = 2,44ns1 = 3,22 m. (64)

Le dispositif interferentiel etudie dans ce probl`eme ne necessite quune lentille de diam`etre 1,32 m.
Lidee du dispositif est due `
a Fizeau (1868). Michelson a construit et monte sur le telescope de lobserva-
toire du Mont Wilson un appareil base sur ce principe, le stellar interferometer. La mesure du contraste
des franges en fonction de s lui a permis de mesurer le diam`etre de quelques etoiles. En 1920, il a obtenu
le diam`etre de Betelgeuse (540 fois celui du Soleil). Letoile, de couleur orange, est avec Rigel, lune des
deux etoiles les plus brillantes de la constellation Orion.
131

12. Examen (2e session) 2004-2005


Enonc
e
23 juin 2005
duree : 3 heures
sans documents, calculatrices autorisees

Durees conseillees : 1 heure pour chacune des parties (A, B et C). Il sera tenu compte de la presentation
et de la concision dans la notation.

A. Induction
Un solenode de longueur b tr`es grande devant son rayon a comporte n spires circulaires jointives par
unite de longueur. Il est parcouru par un courant continu dintensite I. On designe par ~u un vecteur
unitaire parall`ele `
a laxe Oz du solenode.

z
1) On supposera que le solenode est de longueur infinie (b = ) pour a

repondre `
a cette question. On designe par P un point quelconque de lespace.
~ ) au point P est parall`ele `a ~u.
a) Montrer que le champ magnetique B(P
~ ) ne depend que de la distance de P `a laxe du
b) Montrer que B(P
solenode.
c) Montrer que le champ magnetique est uniforme `a linterieur du u
~
b
solenode. O
d) Justifier que le champ magnetique est nul `a lexterieur du solenode.
e) Determiner le champ magnetique `a linterieur du solenode en fonction I
des donnees de lenonce.
f ) Quel est le coefficient dauto-induction par unite de longueur du
solenode?

2) Une bobine de section circulaire de rayon r (r < a), comportant N tours de fil, est placee `a linterieur
du solenode. De plus, les axes de la bobine et du solenode sont confondus.
Quel est le flux de B ~ `a travers la bobine?

En deduire la valeur du rapport M = . Quelle est le nom de la grandeur M ?
I

3) Le solenode est `
a present parcouru par un courant I(t) sinusodal, de frequence = /2 et dam-
plitude I0 :
I(t) = I0 sin t. (1)
132 12. EXAMEN (2E SESSION) 2004-2005

~ a
a) Quelle est lexpression du flux de B ` travers la bobine en fonction du temps?

b) En deduire la f.e.m. dinduction e dans la bobine.

c) On relie les extremites de la bobine. On forme ainsi un circuit de resistance electrique R et dauto-
induction negligeable. Determiner le courant Ib qui circule dans la bobine.

d) Quel est le champ electrique Ei induit `


a linterieur du fil de la bobine?

4) Proposer une methode pour mesurer la grandeur M .

B. R
eflexion dune onde plane
electromagn
etique
Notations et valeurs numeriques pouvant etre utiles a
` la resolution du probl`eme :

On utilise un syst`eme cartesien Oxyz de vecteurs unitaires ~ux , ~uy , ~uz .


Vitesse de la lumi`ere dans le vide : c = 3,00 108 m s1 ;
Permittivite du vide : 0 = 8,85 1012 F m1 ;
Permeabilite du vide : 0 = 4 107 H m1 .

Une onde plane (onde incidente) electromagnetique de pulsation se propageant dans le vide est donnee
par le champ electrique
~ i = A cos (t + kx) ~uz ,
E (2)
u k > 0 et A = 100 V m1 . La frequence de londe est = 5 1014 Hz.
o`

1) Calculer numeriquement la longueur donde .


Quel est le domaine spectral de cette radiation ?
Quelle est la relation entre et k ?


2.a) Ecrire les equations de Maxwell dans le vide en absence de charges et courants volumiques.

~ i de londe.
b) En deduire le champ magnetique B

3) Determiner la densite denergie electromagnetique ui et le vecteur de Poynting P~i de cette onde.


Quelle est la relation entre ui et P~i ?
Calculer litteralement et numeriquement leurs moyennes temporelles < ui > et < P~i >.

4) Londe incidente (2) se propage dans le demi-espace (vide) x > 0. Le demi-espace x 0 est forme
dun metal parfaitement conducteur de sorte que le plan x = 0 forme un miroir metallique. Il apparat
une onde reflechie qui se superpose `
a londe incidente et dont le champ electrique est de la forme
~ r = A cos (t kx) ~uz .
E (3)

~ r de londe reflechie.
Determiner le champ magnetique B

5) Representer sur le meme schema les 6 vecteurs suivants : le vecteur donde et les champs electrique et
magnetique de chacune des ondes reflechie et incidente au point x = y = z = 0 `a linstant t = 0.
133

6.a) Le miroir etant suppose parfait, les champs electrique et magnetique sont nuls dans le metal. Justifier
cette affirmation.


b) Ecrire les relations de passage que verifient les champs electrique et magnetique `a la traversee du
plan x = 0. On ecrira ces relations en fonction de E(0 ~
~ + , t) = limx0+ E(x, ~ + , t) = limx0+ B(x,
t), B(0 ~ t),
de la densite surfacique de charge (t) et du courant surfacique ~ (t).

c) En deduire la densite surfacique de charge (t) et la densite surfacique de courant ~ (t).


Verifier que les relations de passage sont satisfaites.
Calculer numeriquement lamplitude de ~ (t).

C. Pouvoir s
eparateur dune lunette astronomique
On consid`ere une lunette astronomique dont lobjectif est constitue par une lentille mince convergente,
supposee parfaitement stigmatique de distance focale f = 900 mm. La lentille est limitee par un dia-
phragme circulaire, centre sur laxe optique, de diam`etre D = 60 mm. La lunette est precedee dun filtre
qui ne laisse passer que la lumi`ere dont la longueur donde dans le vide est voisine de 0,65 m. La
lunette est placee dans lair dindice na = 1.
A laide de cet objectif on desire observer une etoile double, assimilee `a deux sources ponctuelles S1 et
S2 situees `
a linfini, de meme intensite, separees par langle 1.

R1
La source S1 se trouve sur laxe optique de la lu-
nette. Les images des deux etoiles S1 et S2 dans
le plan focal image de lobjectif de la lunette sont C O
designees par I1 et I2 respectivement.
La figure represente lobjectif, son plan focal image,
son centre C, son axe optique CO et deux rayons R2
parall`eles R1 et R2 provenant de letoile S2 .
f

1.a) Reproduire ce schema et y placer les images I1 et I2 . On demande une construction geometrique
de I2 .
Tracer le trajet des rayons R1 et R2 apr`es leur passage `a travers lobjectif.
Representer deux surfaces dondes et de la lumi`ere issue de S2 , avant que la lumi`ere natteigne
lobjectif et apr`es que la lumi`ere lait traverse.

b) Exprimer en fonction de et de f la distance d separant les images I1 et I2 .


Calculer numeriquement d en microns (m) pour = 1,2 104 rad.

2) Questions de cours

a) Quappelle-t-on lentille parfaitement stigmatique? Comment cette propriete apparat-elle sur votre
schema?


b) Enoncer le theor`eme de Malus-Dupin.
Quelles proprietes du schema deduisez-vous de ce theor`eme?
134 12. EXAMEN (2E SESSION) 2004-2005

c) Donner la definition du chemin optique le long dun chemin allant dun point A `a un point B.

3) En fait, lorsque la lunette est dirigee vers une etoile, assimilee `a une source ponctuelle S placee `a
linfini, on observe dans le plan focal image de la lentille une tache circulaire centree en I, appelee tache
dAiry.

ES ()
La figure ci-contre represente leclairement ES () observe
le long dune droite I du plan focal image. Le premier
minimum nul, autour du centre brillant, correspond `a un
f
rayon = 1,22 .
D
a) Quel est le nom du phenom`ene physique observe ?
Calculer numeriquement, en m, la valeur du rayon du
disque dAiry dans le plan focal image de lobjectif pour
la lunette etudiee.

I

b) La lunette est `a present dirigee vers letoile double. Soit E(x) leclairement observe dans le plan
focal image de lobjectif sur la droite x x qui passe par les points I1 (dabscisse x = 0) et I2 (dabscisse
x = d). Representer lallure de leclairement E(x) lorsque

) d ) d = ) d .

c) Quel est, en radian, le pouvoir separateur de lobjectif, cest-`a-dire la valeur minimale de la


distance angulaire separant les deux composantes dune etoile double pour que les deux etoiles puissent
etre separees `a laide de la lunette etudiee? On supposera que les deux images sont encore distinctes si
le maximum central de la tache dAiry correspondant `a lune des deux etoiles concide avec le premier
minimum nul de la tache dAiry correspondant `a lautre etoile.
Peut-on esperer, avec cet objectif, separer les deux composantes S1 et S2 lorsque = 1,2 104 rad?

4) Pour observer limage obtenue dans le plan focal de lobjectif on utilise un oculaire que lon assimilera
a une lentille mince convergente, supposee parfaitement stigmatique de distance focale f = 10,0 mm.
`

a) Comment faut-il placer cette lentille pour que lon puisse observer sans accommoder limage de
letoile double? (cette image doit etre `
a linfini).
Si les deux composantes dune etoile double sont distantes angulairement de , elles sont visuellement
separees de langle apr`es traversee de la lunette. Faire un schema representant la position des deux
lentilles, le trajet `
a travers ces lentilles des rayons lumineux R1 et R2 issus de la source S2 ainsi que les
angles et et les longueurs f et f . On utilisera des angles et et des longueurs f et f differentes
des valeurs numeriques de lenonce pour que la figure represente clairement ces donnees.


b) En deduire, `
a laide de cette construction geometrique, la valeur du grossissement G = de la

lunette en fonction de f et f .
Calculer numeriquement lorsque = 1,2 104 rad.

5) Sachant que la limite angulaire de resolution de lil est 1 (une minute dangle) dans les meilleures
conditions, le pouvoir separateur de la lunette est-il limite par le pouvoir de resolution de lil ou par le
pouvoir separateur de lobjectif?
135

Pourquoi le pouvoir separateur de la lunette est-il limite si les composantes de letoile double ont des
intensites dordres de grandeurs differents?
Existe-t-il dautres facteurs susceptibles de limiter ce pouvoir separateur? Quelles methodes utilise-t-on
pour y remedier?
136 12. EXAMEN (2E SESSION) 2004-2005

Corrig
e
A. Induction

1.a) Tout plan perpendiculaire ` ~ k ~u.


a ~u est plan de symetrie (pour b = ). Donc B

b) Le syst`eme est invariant par translation parall`ele `a ~u et par rotation autour de laxe du solenode.
~ ) = B(P )~u ne depend que de la distance de P `a laxe du solenode.
Donc B(P

c) Appliquons le theor`eme dAmp`ere au pourtour du rectangle P QRS. On prend P Q k ~u et de longueur


 
d. La circulation de B ~ le long de ce pourtour est d B(P ) B(R) . Pour P et R `a linterieur du solenode,
le courant `a travers le rectangle est nul et B(P ) = B(R). Le champ magnetique est donc uniforme `a
linterieur du solenode.

d) On montre de meme que le champ magnetique est uniforme `a lexterieur du solenode. Si on seloigne
du solenode jusqu` ~ tend vers 0. On suppose que cela reste
a linfini, pour un solenode fini, le champ B
~ ~ =0
vrai pour le solenode infini. Comme B = Cte a` lexterieur du solenode cette constante est nulle et B
a lexterieur du solenode.
`

e) Orientons ~u et I selon la r`egle du tire-bouchon et soit B~u le champ magnetique `a linterieur du


solenode. Le rectangle P QRS traversant le solenode, P `a linterieur du solenode, S `a lexterieur du
solenode, le theor`eme dAmp`ere donne Bd = 0 nId, do`u B = 0 nI et B ~ = 0 nI~u.

f ) Le flux du champ magnetique ` a travers le solenode est s = Ba2 nb = 0 n2 a2 bI. Le coefficient


dauto-induction du solenode est L = s /I = 0 n2 a2 b. Le coefficient dauto-induction par unite de
longueur du solenode est = L/b = 0 n2 a2 .

~ `a travers la bobine est


2) Le flux de B

= Br2 N = 0 nr2 N I. (4)

Le signe est si les sens denroulements du solenode et de la bobine sont opposes.



Le rapport M = = 0 nN r2 sappelle coefficient de mutuelle-induction solenode-bobine.
I

~ a
3.a) Le flux de B ` travers la bobine est

= M I0 sin t = 0 nN r2 I0 sin t. (5)

b) La f.e.m. dinduction dans la bobine est


d
e= = M I0 cos t = 0 nN r2 I0 cos t. (6)
dt

c) Le courant qui circule dans la bobine est

e M I0 0 nN r2 I0
Ib = = cos t = cos t. (7)
R R R
Cest un courant sinusodal damplitude
|M |I0
Ib0 = (8)
R
137

d) La circulation du champ electrique induit Ei le long du fil de la bobine est la f.e.m. e. Linvariance
du syst`eme par rotation autour de son axe implique que Ei est le meme en tout point du fil. On a donc
e = 2rN Ei et
e 0 nrI0
Ei = = cos t. (9)
2rN 2

4) Envoyer un courant I(t) de la forme (1) dans le solenode et mesurer son amplitude I0 et sa pulsation
. Mesurer lamplitude Ib0 du courant Ib (t) qui circule dans la bobine. Dapr`es (8), le coefficient de
mutuelle-induction solenode-bobine sobtient par
RIb0
|M | = . (10)
I0
On a M > 0 (resp. M < 0) si I(t) et Ib (t) sont en opposition de phase (resp. en phase).

B. R
eflexion dune onde plane
electromagn
etique

1) La longueur donde est = c/ = 0,6 m. Le domaine spectral de cette radiation est le visible. La
relation entre et k est = kc.

2.a) Les equations de Maxwell secrivent

1 E ~
~ B
~ = (11)
2
c t
~ E
~ = 0 (12)
B~
~ E
~ = (13)
t
~ B
~ = 0. (14)

~ dependant du temps par le facteur eit ,


2.b) Lequation (13) donne, pour B

x
B /x 0 0
~i = /y

i y =
B ~ E 0 = ikAeit+ikx . (15)

z
B /z Aeit+ikx 0

On en tire
B~i = A eit+ikx ~uy . (16)
c
A
~ i = cos(t + kx)~uy .
B (17)
c

3) La densite denergie electromagnetique est

0 Ei2 B2
ui = + i = 0 A2 cos2 (t + kx).
2 20
Le vecteur de Poynting est
~i B
E ~i A2
P~i = = cos2 (t + kx)~ux .
0 0 c
138 12. EXAMEN (2E SESSION) 2004-2005

La relation entre ui et P~i est P~i = cui ~ux .


Les moyennes temporelles sont < ui >= 0 A2 /2 = 4,42 108 J m3 et < P~i >= c < ui > ~ux =
13,3 ~ux W m2 .

4) Le vecteur donde de londe reflechie est ~kr = k~ux et son champ magnetique

~ ~
~ r = kr Er = A cos (t kx) ~uy .
B
c

5) Les 6 vecteurs sont ~ki = k~ux , ~kr = ~ki = k~ux , E


~ i = A~uz , E
~ r = E ~ i = A ~uy et
~ i = A~uz , B
c
~ ~ A
Br = Bi = ~uy .
c

6.a) La loi dOhm secrit ~ = E ~ o`


u ~ est le courant volumique et la conductivite. Un conducteur
parfait correspond ` a la limite . On doit avoir E ~ = 0 sinon la puissance par unite de volume
dPc ~ = E 2 , serait infinie, ce qui est absurde. Les equations de Maxwell
dissipee par effet Joule, = ~ E
d
restent valables dans un metal. Lequation (13) donne i B ~ =
~ E ~ = 0 et B
~ =0.

6.b) Les relations de passage en x = 0 secrivent :

~ + , t) =
~ux B(0 0~ (18)
~ + , t) =
~ux E(0 (19)
0
~ + , t) =
~ux E(0 0 (20)
~ + , t) =
~ux B(0 0. (21)

On peut aussi les ecrire de facon equivalente

~ + , t) =
E(0 ~ux (22)
0
~ + , t) =
B(0 0~ ~ux . (23)

~ + , t) = E
6.c) On a E(0 ~ i (0+ , t) + E
~ r (0+ , t) = 0 et B(0
~ + , t) = B ~ r (0+ , t) = 2A cos(t)~uy . Les
~ i (0+ , t) + B
c
equations (1821) sont verifiees avec
2A 2A
= 0 et ~ = cos(t) ~uz . Lamplitude de ~ (t) est = 0,53 A m1 .
0 c 0 c
139

C. Pouvoir s
eparateur dune lunette astronomique

I2
1.a) Limage I1 est au foyer image O. Le rayon R3 R1
provenant de S2 qui passe par C nest pas devie par
la lentille. R3 est parall`ele `
a R1 et R2 . Limage I2 C I1
est lintersection de ce rayon et du plan focal.
R3

Les rayons R1 et R2 convergent en I2 apr`es tra-
versee de lobjectif. R2
La surface donde est un plan perpendiculaire aux
rayons dentree et la surface donde une sph`ere f

centree en I2 .

b) Comme 1, d = f = 108 m.

2.a) Une lentille est parfaitement stigmatique lorsque les rayons issus dun point objet S convergent vers
un point I (ou semblent provenir dun point I) apr`es traversee de la lentille.
Cette propriete explique lexistence des images ponctuelles I1 et I2 . Elle implique la convergence des
rayons Ri au point I2 .

b) Th eor`
eme de Malus-Dupin. Dans un milieu isotrope les rayons lumineux sont orthogonaux aux
surfaces donde. est un plan perpendiculaire `a la droite CS2 et est une sph`ere centree en I2 .

c) Chemin optique. Le chemin optique [AB] le long dun chemin allant dun point A `a un point
B est lintegrale Z
[AB] = ndl (24)

o`
u n est lindice du milieu. Lindice n nest pas suppose constant. Une particule qui se meut `a la vitesse
c dl ndl
(de phase) de la lumi`ere v = parcourt lelement dl dans le temps dt = = . Une autre definition
n v c
du chemin optique [AB] est
[AB] = ctAB (25)
Z
o`
u tAB = dt est le temps que mettrait une telle particule pour parcourir le chemin (ce chemin nest

pas necessairement un rayon lumineux).

f
3.a) La diffraction. Le rayon du disque dAiry est = 1,22 = 12 m.
D

b) E(x) E(x) E(x)

) d ) d = ) d

x x x
0 d 0 d 0d
140 12. EXAMEN (2E SESSION) 2004-2005


c) Le pouvoir separateur de lobjectif est = = 1,22 = 1,32 105 rad. Les deux composantes
f D
peuvent donc etre separees lorsque = 1,2 104 rad.

4.a) Il faut placer laxe optique de loculaire selon laxe optique de lobjectif, avec le foyer objet de
loculaire confondu avec le foyer image de lobjectif. Le syst`eme est alors afocal (un objet `a linfini donne
une image `a linfini).

I2

C
I1 C

f f

f
b) G = = = 90. = G = 1,08 102 rad= 37 .
f

5) Le rayon du cercle de diffraction est vu sous langle = G = 1,2 104 rad = 4 . Cet angle est
superieur `a la limite de resolution de lil (1 ). Le pouvoir separateur de la lunette est donc limite par
le pouvoir separateur de lobjectif.
Si les composantes de letoile double ont des intensites dordres de grandeurs differents, le disque dAiry
de la composante la moins intense peut etre masque par les pieds de la tache dAiry de la composante
la plus intense. Les images ne sont pas toujours distinctes lorsque < d et le pouvoir separateur de la
lunette sen trouve limite.
Le pouvoir separateur peut etre limite par la turbulence atmospherique. On a avantage `a choisir un
endroit o`u latmosph`ere est le plus calme possible. Une solution extreme consiste `a placer la lunette dans
un satellite.
141

13. Premier partiel 2005-2006


Enonc
e
8 octobre 2005
duree : 1 heure 30
sans documents, calculatrices autorisees

Bar`eme indicatif : A=12 et B=8.

A. Ondes de types S et A
Une corde homog`ene tr`es longue (infinie) est tendue le long de laxe x Ox. On considerera que la corde
au repos occupe la droite x Ox. La vitesse de propagation des ondes le long de la corde est c = 10 m s1 .

1) Questions de cours

Ecrire lequation de dAlembert verifiee par le deplacement s(x, t) des ondes dune corde vibrante.

Enoncer le theor`eme de dAlembert donnant la forme generale du deplacement s(x, t) des ondes dune
corde vibrante.

2) On appelle onde de type S une onde de la forme

1 
s(x, t) = (x + ct) + (x ct) (1)
2

o`
u (u) est une fonction arbitraire.
Justifier que lexpression (1) decrit une onde de la corde.

2
/a2
3) On consid`ere londe de type S correspondant `a (u) = Aeu avec a = 1 cm et A = 2 cm :

1 h (x+ct)2 /a2 2 2
i
s(x, t) = Ae + Ae(xct) /a . (2)
2

Tracer lallure de la corde aux instants t0 = 0 et t1 = 5 103 s.


142 13. PREMIER PARTIEL 2005-2006

Decrire la propagation de cette onde aux instants t > t1 .

s (cm)

x 0 x (cm)

Fig. 41 Mod`ele de diagramme pour tracer lallure de la corde.

4) On appelle onde de type A une onde de la forme


1 
s(x, t) = (x + ct) (x ct) (3)
2
o`
u (u) est une fonction arbitraire.
Justifier que lexpression (3) decrit une onde de la corde.
Tracer lallure de la corde aux instants t0 = 0 et t1 = 5 103 s lorsque
2
/a2
(u) = Aeu avec a = 1 cm et A = 2 cm (4)

et decrire la propagation de cette onde aux instants t > t1 .

5.a) On consid`ere deux ondes de type S,


1  1 
s1 (x, t) = 1 (x + ct) + 1 (x ct) et s2 (x, t) = 2 (x + ct) + 2 (x ct) . (5)
2 2
Montrer que leur superposition S(x, t) = s1 (x, t) + s2 (x, t) est une onde de type S.

b) Que pouvez-vous dire de la superposition de deux ondes de type A?

6) On consid`ere une onde de la forme


s(x, t) = G(x + ct) (6)
o`
u G(u) est une fonction arbitraire.
Comment sappelle une telle onde? Comment se propage-t-elle?
Montrer que cette onde peut sobtenir par superposition de deux ondes, une de type S et lautre de type
A.

7) On consid`ere une onde de la forme


s(x, t) = F (x ct) (7)
o`
u F (u) est une fonction arbitraire.
Comment sappelle une telle onde? Comment se propage-t-elle?
143

Montrer que cette onde peut sobtenir par superposition de deux ondes, une de type S et lautre de type
A.

8) Montrer que londe la plus generale est est la somme de deux ondes

s(x, t) = S(x, t) + A(x, t) (8)

o`
u
1 
S(x, t) = (x + ct) + (x ct) (9)
2
est une onde de type S et
1 
A(x, t) = (x + ct) (x ct) (10)
2
est une onde de type A.
s
Exprimer (u) et (u) en fonction de s(x, 0) et (x, 0).
t

B. Onde harmonique stationnaire dune corde


On cree dans une corde tendue le long de laxe x Ox une onde transversale, harmonique, stationnaire de
frequence = 420 Hz.

1) Expliquer le sens des mots transversale, harmonique et stationnaire.

2) Donner lexpression generale s(x, t) du deplacement du point dabscisse x `a linstant t dune onde
stationnaire de frequence . On ecrira la reponse en fonction de x, t, , (longueur donde), a (la valeur
maximum de s(x, t)) et de deux phases arbitraires et .

3) La corde a la longueur l = 70 cm et est tendue entre les deux points fixes A (dabscisse xA = 0) et
B (dabscisse xB = l) (cf. figure 42). La masse lineique de la corde est = 10 g m1 et la tension de la
corde est T .
s

B
A
0 l x

Fig. 42 Corde au repos.

Preciser la forme precedente de lexpression de s(x, t) sachant que


la corde prend laspect de la figure 43 ;
`a t = 0 la corde est dans sa position dequilibre (cf. figure 42) ;
juste apr`es linstant t = 0 la corde se deforme vers le haut de la figure 42.
On ecrira s(x, t) en fonction de x, t, , l et a.
s

B
A x

Fig. 43 Onde creee.


144 13. PREMIER PARTIEL 2005-2006

4) Representer la forme de la corde `


a divers instants. Expliquer pourquoi la corde prend laspect de la
figure.

5) Determiner numeriquement la longueur donde , la vitesse c des ondes de la corde et la tension T .


145

Corrig
e

A. Ondes de types S et A

1) Lequation de dAlembert est


1 2s 2s
2 2
2 = 0. (11)
c t x

Theor`
eme de dAlembert (1747) La solution generale de lequation de la corde vibrante (11) est la
superposition de deux ondes progressives arbitraires allant en sens opposes

s(x, t) = F (x ct) + G(x + ct). (12)

2) Lexpression (1) est de la forme (12) avec F (u) = G(u) = (u). Dapr`es le theor`eme de dAlembert,
elle decrit une onde de la corde.

(cm)
u2 /a2 2
3) La fonction (u) = Ae est une gaussienne qui a une
` linstant t = t0 = 0 on a
largeur de lordre de a = 1 cm. A
2
1
/a2
s(x, t0 ) = (x) = Aex . (13)
` linstant t = t1 = 5 103 s, on a, posant b = ct1 = 5 cm,
A 2 1 0 1 u (cm)

Ah 2
/a2 2 2
i
s(x, t1 ) = e(x+b) + e(xb) /a
. (14) Fig. 44 Courbe (u).
2

s (cm)

0 x (cm)

Fig. 45 Londe de type S `a linstant t = t0 = 0.

s (cm)

b 0 b x (cm)

Fig. 46 Londe de type S `a linstant t = t1 = 5 103 s.


146 13. PREMIER PARTIEL 2005-2006

Aux instants t > t1 , londe est formee de deux perturbations de forme en cloche qui seloignent lune de
lautre avec la vitesse c. La hauteur des deux cloches est la moitie de la hauteur de la cloche `a linstant
t = t0 . Les deux cloches sont symetriques par rapport `a laxe Os.

4) Lexpression (3) est de la forme (12) avec F (u) = (u) et G(u) = (u). Dapr`es le theor`eme de
dAlembert, elle decrit une onde de la corde.
` linstant t = t0 = 0 on a
A
s(x, t0 ) = 0. (15)
` linstant t = t1 = 5 103 s, on a (b = ct1 = 5 cm),
A
A h (x+b)2 /a2 2 2
i
s(x, t1 ) = e e(xb) /a . (16)
2
s (cm)

0 x (cm)

Fig. 47 Londe de type A `a linstant t = t0 = 0.


s (cm)

b 0 b x (cm)

Fig. 48 Londe de type A `a linstant t = t1 .

Aux instants t > t1 , londe est formee de deux perturbations, une bosse et un creux, qui seloignent lune
de lautre avec la vitesse c. Le creux et la bosse sont symetriques par rapport au point O.

5.a) La somme
S(x, t) = s1 (x, t) + s2 (x, t) (s1 et s2 de type S) secrit
1 
S(x, t) = s1 (x, t) + s2 (x, t) = (x + ct) + (x ct) avec (u) = 1 (u) + 2 (u). (17)
2
Cest donc une onde de type S.

5.b) La superposition des deux ondes de type A


1  1 
s3 (x, t) = 1 (x + ct) 1 (x ct) et s4 (x, t) = 2 (x + ct) 2 (x ct)
2 2
secrit
1 
A(x, t) = s3 (x, t) + s4 (x, t) = (x + ct) (x ct) avec (u) = 1 (u) + 2 (u). (18)
2

Cest donc une onde de type A.


147

6) Londe s(x, t) = G(x + ct) est une onde progressive.


Elle se propage vers les x decroissants.

Ecrivons
1  1 
G(x + ct) = G(x + ct) + G(x ct) + G(x + ct) G(x ct) .
2 2
Cela montre que cette onde est la superposition

G(x + ct) = s1 (x, t) + s3 (x, t) (19)

de londe de type S
1 
s1 (x, t) = 1 (x + ct) + 1 (x ct) , avec 1 (u) = G(u),
2
et de londe de type A
1 
s3 (x, t) = 1 (x + ct) 1 (x ct) , avec 1 (u) = G(u).
2

7) Londe s(x, t) = F (x ct) est une onde progressive.


Elle se propage vers les x croissants.

Ecrivons
1  1 
F (x ct) = F (x + ct) + F (x ct) F (x + ct) F (x ct) .
2 2
Cela montre que cette onde est la superposition

F (x ct) = s2 (x, t) + s4 (x, t) (20)

de londe de type S
1 
s2 (x, t) = 2 (x + ct) + 2 (x ct) , avec 2 (u) = F (u),
2
et de londe de type A
1 
s4 (x, t) = 2 (x + ct) 2 (x ct) , avec 2 (u) = F (u).
2

8) Le theor`eme de dAlembert, equation (12), et les equations (19) et (20) montrent que londe la plus
generale secrit

s(x, t) = F (x ct) + G(x + ct) = s1 (x, t) + s2 (x, t) + s3 (x, t) + s4 (x, t) .


| {z } | {z }
ondes de type S ondes de type A

En utilisant les resultats de la question 5 (equations (17) et (18)) il vient le theor`eme suivant.
Theor`
eme La solution generale de lequation de la corde vibrante (11) est la superposition de deux
ondes
s(x, t) = S(x, t) + A(x, t) (21)
   
u S(x, t) = 12 (x + ct) + (x ct) est une onde de type S et A(x, t) = 21 (x + ct) (x ct) est
o`
une onde de type A.
Autre d emonstration La solution generale (12) de lequation de la corde vibrante (11) est la super-
position des deux ondes (810) avec (u) = F (u) + G(u) et (u) = F (u) G(u).
s
On tire des equations (810) s(x, 0) = (x) et (x, 0) = c (x).
t
148 13. PREMIER PARTIEL 2005-2006

On en deduit
Z u
1 s
(u) = s(u, 0) et (u) = (x, 0) dx + K. (22)
0 c t

Remarques
La fonction (u) est definie `
a la constante K pr`es. Londe de type A associee ne depend pas du choix
de K.
En portant les equations (22) dans (810) on obtient le deplacement s(x, t) au temps t en fonction des
s
conditions initiales, le deplacement s(x, 0) et la vitesse (x, 0) des points de la corde `a linstant t = 0.
t
On a donc obtenu une solution explicite du probl`eme de Cauchy :

Z x+ct
1  1 1 s
s(x, t) = s(x + ct, 0) + s(x ct, 0) + (u, 0) du. (23)
2 2 xct c t

Le deplacement s(x, t) du point M (abscisse x) au temps t ne depend que de la valeur des deplacements
des points A (abscisse x c|t|) et B (abscisse x + c|t|) ainsi que des vitesses des points du segment AB
a linstant initial t = 0. La valeur de s(x, t) est independante de londe en dehors du segment AB `a
`
linstant initial. Une perturbation ne peut pas se propager plus loin que la distance ct pendant le temps
t. Cela correspond bien ` a une vitesse de propagation des ondes progressives egale `a c.

B. Onde harmonique stationnaire dune corde

1) Transversale : les points de la corde se deplacent perpendiculairement `a Ox (comparer `a onde


longitudinale).
Harmonique : londe est sinusodale.
Stationnaire : tous les points de la corde vibrent en phase ou en opposition de phase (comparer `a onde
progressive).

2) La forme generale dune onde stationnaire harmonique damplitude a est (avec = c)


 
2x
s(x, t) = a sin(2t + ) sin + . (24)

3) Londe presente un fuseau. On a donc les conditions : s(x, t) = 0 t pour x = 0 et x = l ; pour les
autres valeurs de x (0 < x < l), s(x, t) nest pas identiquement nul. Puisque s(x, t), `a tout instant t,
est une sinusode en x, ces conditions impliquent que s(x, t) est proportionnel `a sin(x/l). On en deduit
que = 0 et = 2l dans lequation (24). La condition s(x, 0) = 0 (corde dans sa position dequilibre `a
linstant t = 0) donne = 0 (mod ). On a donc s(x, t) = a sin(2t) sin (x/l).
Le signe est determine par la condition de lenonce juste apr`es linstant t = 0 la corde se deforme vers
le haut :
 x 
s(x, t) = a sin(2t) sin . (25)
l
149

s
4) La figure ci-contre represente la forme de la corde aux
nT
instants tn = (mod T ) pour n = 0, 1, . . . , 15. La B
16 A x
periode T = 1/ = 2,381 ms est tr`es petite devant le temps
de persistance des images retiniennes. La corde apparat
comme la superposition de toutes les positions de la corde.
Fig. 49 Corde `a divers instants.

s
T
5) = 2l = 140 cm ; c = = 588 m s1 ; c = donne T = c2 = 3457 N.
T
150 `
14. DEUXIEME PARTIEL 2005-2006

14. Deuxi`
eme partiel 2005-2006


Enonc
e
9 novembre 2005
duree : 3 heures
sans documents, calculatrices autorisees

Bar`eme indicatif : A = 8, B = 5 et C = 7.

Valeurs numeriques des constantes physiques


Permittivite du vide : 0 = 8,85 1012 F m1
Permeabilite du vide : 0 = 4 107 H m1
Vitesse de la lumi`ere dans le vide : c = 3,00 108 m s1

A. Magn
etostatique : bobinage torodal, inductance
Soit O le point de laxe Ox dabscisse a = 20 cm et C le cercle du plan Oxz de centre O et de rayon
b = 1 cm (cf. figure 50). On consid`ere le tore T engendre par la rotation de ce cercle autour de laxe z Oz
(cf. figure 51). Un fil de cuivre est enroule sur ce tore pour former un bobinage torodal B de N = 1250
spires jointives.
On utilisera les coordonnees cylindriques (r, , z) et le rep`ere local associe (~ur , ~u , ~uz ).
z
z T

C
b O
O
O x

a
I
z
z
Fig. 51 Tore T et un bobinage torodal. Le bobinage B est
Fig. 50 Cercle C. plus serre que sur la figure.

Le fil est parcouru par un courant dintensite I constante.


151

Pour determiner le champ magnetique B ~ cree par le courant, on fera lapproximation de remplacer le
bobinage torodal par N boucles circulaires jointives. Chacune de ces boucles est parcourue par le courant
I. Une de ces boucles est le cercle C. Le sens du courant est indique par une fl`eche sur les figures 50 et 51.

1) Justifier que le champ magnetique est de la forme

~ , z) = B(r, z) ~u .
B(r, (1)


2.a) Enoncer le theor`eme dAmp`ere.

~ ext = 0.
b) Montrer que le champ magnetique `a lexterieur du tore est nul : B

~ int `a linterieur du tore.


c) Determiner le champ magnetique B


3) Ecrire les relations de passage du champ magnetique en un point M = (r, , z) du tore.
En deduire que le bobinage equivaut `
a une nappe de courant surfacique et determiner le vecteur courant
surfacique ~ au point M .
Faire une figure pour preciser la direction et le sens de ce vecteur.
En quelle unite le courant surfacique ~ est-il mesure ?

4) Le module Bint nest pas uniforme. Pour simplifier, on utilisera lapproximation

~ int B0 ~u
B (2)

u B0 est choisi pour que lexpression soit exacte au point O .


o`
Quelle est lexpression de B0 en fonction de 0 , N , a et I ?
Quelle est lerreur relative maximale

Bint B0
= max
(3)
B0

que lon commet lorsquon utilise lapproximation (2) `a linterieur du tore?

` partir de cette question on supposera que 1 et on utilisera lapproximation (2).


5) A
Determiner le flux du champ magnetique `a travers le bobinage B.
En deduire le coefficient dauto-induction L du bobinage B.
152 `
14. DEUXIEME PARTIEL 2005-2006

Calculer L numeriquement.
z
B
6.a) Determiner la densite denergie magnetostatique
um . I
R B

b) En deduire lenergie magnetostatique totale W en T


H
fonction de 0 , N , a, b et I. O x
Exprimer W en fonction de L et I.
Calculer la valeur numerique de W pour I = 10 A.
I

7) Un deuxi`eme fil en cuivre, parcouru par le courant


constant I , forme une boucle B enlacant le tore T
(cf. figure 52). Cette boucle est situee dans le demi- A

plan (y = 0, x 0) et formee dun segment rectiligne z

AB le long de laxe z Oz et dun demi-cercle BHA de


Fig. 52 Boucle B = ABH
rayon R = 5 m et de centre O.
enlacant le tore T .

a) En supposant que R a b, determiner de facon approchee le champ magnetique B ~ cree par le


~
courant I `a linterieur du tore T . En deduire le flux 1 du champ magnetique B `a travers le bobinage
B.

b) En utilisant (2), calculer le flux 2 du champ magnetique cree par le courant I `a travers la boucle B .

c) Determiner le coefficient dinductance mutuelle M des circuits B et B . Calculer sa valeur numerique.


z
B

8) On translate la boucle B de D ~ux avec D = 10 cm I


B
(cf. figure 53). Determiner le coefficient dinductance
mutuelle M des circuits B et B dans leur nouvelle
position. T
H
O x

9) Dans cette question, on enl`eve la boucle B (I = I

0) et on consid`ere le vrai bobinage torodal B, sans D


faire lapproximation de le remplacer par N boucles
jointives.
Montrer que le champ magnetique B ~ ext `
a lexterieur A
z
du tore ne peut pas etre identiquement nul. Nota : on
ne demande pas de calculer B ~ ext .
Fig. 53 Nouvelle position de la boucle
B .

B. Ondes
electromagn
etiques
On consid`ere la propagation dans le vide dune onde electromagnetique UHF emise par un relais emetteur
de telephonie mobile. On utilise un syst`eme cartesien Oxyz de vecteurs unitaires ~ux , ~uy , ~uz , le vecteur
~uz etant vertical et dirige vers le haut.
153

Au voisinage du point O, qui est ` a la distance L = 450 m de lemetteur, le champ magnetique de londe
au point ~r (x, y, z) et `
a linstant t est donne par
~ r , t) = B0 cos(t ky)~ux
B(~ (4)
avec B0 = 1,25 109 T et k = 17,0 m1 .

1) Determiner numeriquement la longueur donde et la frequence de cette onde. Quelle est la position
de lemetteur? Justifier la reponse et donner les coordonnees de lemetteur (x0 , y0 , z0 ).


2) Ecrire ~ r , t) de londe.
lexpression du champ electrique E(~
Faire un schema representant le vecteur donde ~k ainsi que les champs E
~ et B
~ en O et `a linstant t = 0.

5 cm 5 cm
3) Un observateur place en O mesure lamplitude E0 de ce
A A B B
champ electrique `
a laide dune antenne dipolaire AB formee
de 2 fils conducteurs de 5 cm de long (cf. figure 54).
Quelle est la valeur numerique de E0 ? signal

Comment lobservateur doit-il disposer lantenne pour obser-


ver le signal maximum? Fig. 54 Antenne dipolaire.

Z
4) Un fil DD , enroule sur un cylindre circulaire de rayon R = 7 mm
et daxe Z Z, forme une bobine plate de n = 5 spires (cf. figure 55).
Lobservateur place le centre de la bobine en O et mesure la force R
electromotrice dinduction e qui apparat entre ses bornes D et D .
D

a) Justifier que le champ B~ peut etre considere comme uniforme `a


D
linterieur de la bobine.
Z

b) Comment lobservateur doit-il orienter laxe Z Z pour obtenir
un signal maximum? Fig. 55 Bobinage.
Quelle est alors lexpression de e?

c) Calculer la valeur numerique de lamplitude U0 de e.

d) Comment lobservateur doit-il orienter laxe Z Z pour obtenir le signal le plus petit possible?

5) Determiner la densite denergie electromagnetique u de londe ainsi que sa moyenne temporelle u = hui.
Calculer la valeur numerique de u.

D E
6) Determiner le vecteur de Poynting P~ de londe ainsi que sa moyenne temporelle P~m = P~ .

Calculer la valeur numerique de P~m .

7) Quelle relation existe-t-il entre le vecteur de Poynting P~ et la densite denergie electromagnetique u?

8) Estimer numeriquement la puissance moyenne rayonnee W par lemetteur (on pourra supposer que
lemission est isotrope).
154 `
14. DEUXIEME PARTIEL 2005-2006

C. Blindage
electromagn
etique
On desire eviter la perturbation dun appareil sensible par londe (4). Une solution couramment utilisee
consiste `a placer lappareil dans un botier metallique faisant ecran. Pour pouvoir determiner les dimen-
sions du botier et evaluer lattenuation du champ electromagnetique, nous allons etudier tout dabord la
propagation des ondes de frequence dans un conducteur. Les divers champs en representation complexe
~ B,
(E, ~ , ~) dependront du temps par le facteur eit .


1) Ecrire ~ et B
les equations de Maxwell dans le vide verifiees par E ~ en presence de la charge volumique
et du courant volumique ~.
Demontrer lequation de continuite de la charge `a partir de ces equations.

2) Hypoth`eses : (a) dans le conducteur, les equations demandees `a la question 1) sont valables ; (b) le
courant volumique ~ et le champ E~ satisfont `a la loi dOhm locale
~
~ = E (5)

u = 2,25 107 S m1 est la conductivite ohmique du conducteur.


o`
En utilisant ces hypoth`eses, montrer que verifie une equation differentielle de la forme

+ a = 0 (6)
t
o`
u a est une constante.
En deduire que = 0.

3) A` partir des equations de Maxwell, avec = 0 et ~ 6= 0, determiner lequation de propagation de E


~
(cette equation contient ~).
Verifier que cette equation de propagation donne lequation de dAlembert pour ~ = 0.
On pourra utiliser la formule :
 ~   
~ E.
~
rot rot E = grad div E (7)

4) Le conducteur est limite par le plan y = 0 et occupe tout le demi-espace 0 y < . On etudie dans
le conducteur une onde de la forme
E~ = Aey ei(ty) ~u (8)
o`
u A, , sont des constantes reelles non nulles (avec > 0) et ~u un vecteur unitaire constant.


a) Ecrire ~ de la representation complexe (8).
le champ reel E
~ au point M = (x, y, z)?
Quelle est lamplitude Ar (y) du champ electrique reel E
Pourquoi doit-on avoir > 0?
Tracer la courbe Ar (y) en fonction de y.
Londe se propage-t-elle dans la direction de Oy vers les y croissants?
Quel est le phenom`ene physique `
a lorigine de la variation de Ar (y) avec y ?

~ pour le champ (8).


~ E
b) Calculer
155

En deduire une condition que doit verifier le vecteur ~u.


5.a) Ecrire que londe (8) est solution de lequation trouvee a` la question 3) en tenant compte de
lequation (5).
En deduire une equation de la forme

2 2i
( + i)2 = + 2. (9)
c2 L

Quelle est lexpression litterale de L en fonction de 0 , et ?


Calculer la valeur numerique de L.

2 2i
b) Verifier numeriquement que le terme 2
est negligeable par rapport `a 2 .
c L
Calculer les valeurs numeriques de et . On rappelle que (1 + i)2 = 2i.

6) On consid`ere un blindage forme dune tole metallique depaisseur d, de meme conductivite ohmique
que le milieu conducteur precedent et qui occupe la region 0 y d.
Londe (4) (dans le milieu y < 0) tombe sur cette t
ole et y produit une onde de la forme (8).
Ar (d)
On estime que le blindage sera suffisant si le rapport des amplitudes du champ est inferieur `a
Ar (0)
= 1012 .
Comment doit-on choisir lepaisseur du blindage?
156 `
14. DEUXIEME PARTIEL 2005-2006

Corrig
e

A. Magn
etostatique : bobinage torodal, inductance

1) Tout plan contenant le point M et laxe z Oz est un plan de symetrie du syst`eme. Le champ B ~ en M
~
est normal `a ce plan et donc de la forme B(r, , z) = B(r, , z) ~u . Le syst`eme est invariant dans toute
rotation autour de z Oz : B(r, , z) ne depend pas de . On a donc

~ , z) = B(r, z) ~u .
B(r, (10)

~
n
2.a) Th eor`
eme dAmp` ere. Soit un circuit et S une surface
AAA
AAA
AAAA
AAA
AAA S

AAA
AA
AAA
AAAA
AAA
AAA
AA
orientee de bord = S (cf. figure 56). Lorientation du circuit
correspond `a celle de la surface S (r`egle du tire-bouchon). La
~ le long de est
circulation de B
I
~

AAA AAA
AA AA
B dr = 0 IS . (11)

Fig. 56 Theor`eme
o`
u ZZ dAmp`ere.
IS = ~ ~n dS (12)
S

est le courant qui traverse la surface S (le courant enlace par ).


Remarques. 1. Pour un circuit donne, la valeur de IS est independante de la forme de la surface S.
2. Le theor`eme dAmp`ere nest plus valable en regime variable. On doit alors le remplacer par le theor`eme
dAmp`ere generalise.

b) Appliquons le theor`eme dAmp`ere en prenant pour contour le cercle de rayon r, daxe z Oz, de cote
~ le long de est 2rB(r, z).
z et oriente selon ~u . La circulation de B
Dans cette question on prend le cercle exterieur au tore. Pour S on peut toujours choisir une surface
exterieure au tore. Le courant qui traverse la surface S est nul : IS = 0. Le theor`eme dAmp`ere implique
que le champ magnetique est nul ` a lexterieur du tore :

~ ext = 0.
B (13)

c) Dans cette question on prend le cercle ` a linterieur du tore. Pour S on prendra le disque de bord
; son orientation est suivant ~uz . Le courant qui traverse la surface S est : IS = N I. Le theor`eme
dAmp`ere implique :

~ int = 0 N I ~u .
B (14)
2r
157

z
3) Soit ~n le vecteur unitaire normal au tore en M et dirige ~

de linterieur vers lexterieur du tore. ~
n
Le champ magnetique au voisinage de M est nul `a lexterieur
r
du tore (B ~ ext = 0) et B ~ int = 0 N I ~u `a linterieur du tore M
2r ~ int
B
(o`
u r est la distance de M ` a laxe z Oz). Les relations de O
C X
passage du champ magnetique en M secrivent
 
~n B~ ext B
~ int = 0 (15)
 
~n B ~ ext B
~ int = 0~ (16)
z

Fig. 57 Relations de passage.

La premi`ere equation est verifiee et la deuxi`eme donne

NI
~ = ~n ~u . (17)
2r

Le sens et la direction du vecteur ~ sont precises sur la figure 57. Ce vecteur est tangent au tore et dans
le plan M z z. Le bobinage equivaut `
a une nappe de courant surfacique torodale dintensite totale N I.
Lunite de mesure de courant surfacique est lamp`
ere par m`
etre.

4) En O , r = a. On pose donc
0 N I
B0 = . (18)
2a
0 N I
La valeur Bint (r) = ne depend pas de z. La variation maximale de r `a linterieur du tore autour
2r
de sa valeur r = a au point O est r = b a. On a, en notant Bint la variation maximale de Bint
Bint r b
autour de B0 , = = . Lerreur relative maximale commise est donc
B0 r a

Bint B0 b
= max = = 0,05. (19)
B0 a

5) Dans lapproximation (2) le champ B~ int est uniforme sur tout disque section de linterieur du tore et

dun demi-plan de bord z Oz. Le flux de B ~ int a` travers une spire du bobinage est

0 N b2 I
= b2 B0 = .
2a
Le flux = N du champ magnetique `a travers le bobinage B est

0 N 2 b2 I
= . (20)
2a

On en deduit le coefficient dauto-induction L = /I du bobinage B :

0 N 2 b2
L= = 491 H. (21)
2a
158 `
14. DEUXIEME PARTIEL 2005-2006

6.a) La densite denergie magnetostatique est



0 N 2 I 2
B2
uint = `a linterieur du tore
um = = 8 2 a2 (22)
20 0 `a lexterieur du tore.

b) La densite denergie magnetostatique etant uniforme a` linterieur du tore, lenergie magnetostatique


totale sobtient en multipliant uint par le volume V 2 2 ab2 (valeur approchee du volume interieur au
tore pour b a) :
0 N 2 b2 I 2
W = . (23)
4a
En fonction de L et I on a
LI 2
W = . (24)
2
La valeur numerique de W pour I = 10 A est W = 0,0245 J.

7.a) Comme R a on peut determiner le champ magnetique B ~ cree par le courant I `a linterieur du
tore T comme celui cree par un courant infini I le long de z Oz. On a donc

~ (r, , z) = 0 I ~u .
B (25)
2r
~ est uniforme `a linterieur du tore (dapr`es a b). On en deduit
On peut considerer que le module de B
2
le flux 1 = N b B du champ magnetique B ~ `a travers le bobinage B :

0 N b2 I
1 = . (26)
2a

~ cree par le courant I est nul en dehors du tore. Son flux `a travers la boucle
b) Le champ magnetique B
2
B est donc 2 = B0 b :
0 N b2 I
2 = . (27)
2a

c) Le coefficient dinductance mutuelle des circuits B et B est

1 2 0 N b2
M= = = = 0,392 H. (28)
I I 2a

8) Le flux du champ magnetique B ~ cree par le courant I `a travers la boucle B ne depend pas de la
position de la boucle B (tant que celle-ci enlace le tore). Le coefficient dinductance mutuelle M garde

donc la meme valeur :


M = M. (29)

9) Le circuit ferme ABHA de la figure 52 enlace le courant I (le signe depend du sens de lenroulement
du bobinage B). Dapr`es le theor`eme dAmp`ere la circulation du champ magnetique le long de ce circuit
nest pas nulle (elle vaut 0 I). Le champ magnetique `a lexterieur du tore ne peut donc pas etre
identiquement nul.
159

B. Ondes
electromagn
etiques

2 c
1) La longueur donde est = = 0,37 m et la frequence = = 812 MHz.
k
Londe est une onde progressive se propageant le long de Oy vers les y croissants. Lemetteur se trouve
donc sur laxe y Oy en y = L. Ses coordonnees sont

(x0 , y0 , z0 ) = (0, L, 0).

2) Londe est une OPPH. Les vecteurs (E,~ B,~ ~k = k~uy ) forment un
~
E
tri`edre orthogonal direct et E = cB. On a donc

~ r , t) = cB0 cos(t ky)~uz .


E(~ (30)
O ~ y
k
x
~
B
3) Lamplitude du champ electrique est
~ ~k).
~ B,
Fig. 58 (E,
E0 = cB0 = 0,375 V m1 .

Pour observer le signal maximum lobservateur doit placer les fils parall`element au champ electrique. Il
doit donc placer AB le long de laxe Oz.

` linterieur
4.a) Les dimensions de la bobine sont petites par rapport `a la longueur donde (R ). A
de la bobine, on peut ecrire en negligeant la phase ky (|ky| 2R/ = 0,13)

~ r , t) = B0 cos(t ky)~ux B0 cos(t)~ux


B(~ (`
a linterieur de la bobine). (31)

~ est ainsi considere comme uniforme dans la bobine.


Le champ B

4.b) Soit langle que fait laxe Z Z avec laxe ~ux . Le flux du champ magnetique `a travers le bobinage
est = nR2 cos B0 cos(t). La f.e.m. dinduction est

d
e= = nR2 cos B0 sin(t). (32)
dt
On observera un signal maximum pour cos = 1, cest `a dire lorsque Z Z est dans la direction x Ox.
On a alors
e = nR2 B0 sin(t) (33)

a Z Z de meme sens que x x et le signe au sens oppose.


le signe + correspondant `

4.c) Lamplitude de e est


U0 = nR2 B0 = 5,63 mV. (34)

4.d) Le signal est nul pour cos = 0. Lobservateur doit orienter laxe Z Z perpendiculairement `a x x
pour obtenir un signal nul.
160 `
14. DEUXIEME PARTIEL 2005-2006

5) La densite denergie electromagnetique de londe est

0 E 2 B2
u= + = 0 c2 B02 cos2 (t ky). (35)
2 20

Sa moyenne temporelle est


0 c2 B02
u= = 0,622 1012 J m3 . (36)
2

6) Le vecteur de Poynting est

~ B
E ~ cB02
P~ = = cos2 (t ky) ~uy = 0 c3 B02 cos2 (t ky) ~uy (37)
0 0

Sa moyenne temporelle est

cB02 0 c3 B02
P~m = ~uy = ~uy = (0,187 103 W m2 ) ~uy . (38)
20 2

7) La relation entre le vecteur de Poynting P~ et la densite denergie electromagnetique u est P~ = c u ~uy .

8) La puissance moyenne rayonnee est estimee `a W = 4L2 Pm = 475 W.

C. Blindage
electromagn
etique

1) Les equations de Maxwell sont


~ = ,
~ E (39)
0
~ B
~ = 0, (40)
~
~ = B ,
~ E (41)
t
!
~
~ = 0 ~ + 0 E
~ B . (42)
t
Prenons la divergence de lequation (42). Il vient, la divergence dun rotationnel etant nulle,
~
~ E = 0.
~ ~ + 0 (43)
t
Derivons lequation (39) par rapport ` ~ on a
a t. Cette derivee permutant avec loperateur ,
~
~ E = .
0 (44)
t t
En portant dans (43) on obtient lequation de continuite de la charge

~ ~ + = 0.
(45)
t
161

2) En utilisant (45), (39) et (5), on a

~ ~ = ~ =
~ E
=
t 0
soit

+ = 0. (46)
t 0
 

Pour des ondes de pulsation , cela implique i + = 0. Do`
u = 0. Noter que lequation de
0
Maxwell-Gauss (39) secrit
~ E
~ = 0. (47)

3) En utilisant (41), (42) et 0 0 c2 = 1, il vient


2~
~ (
~ =
~ E) ~ = 0 ~ 1 E .
~ B
t t c t2
2

~ (
Avec (47) et (7) on ecrit ~ E)
~ = E.
~ Lequation de propagation de E
~ est donc :

2~
~ = 1 E E
E ~ = 0 ~ . (48)
c2 t2 t

Cette equation redonne bien lequation de dAlembert ~ = 0 pour ~ = 0.


E
Autre m
ethode. Partons des equations des potentiels en jauge de Lorenz :
~ = 0~.
V = =0 et A
0
~
A
Appliquons loperateur aE
` ~ = ~ . On obtient, loperateur
V commutant avec les operateurs
t
~ :
et
t
~ =(
E ~
A) ~ ( V ) = 0
~
.
t t
On retrouve lequation de propagation (48).

Ar
4.a) Le champ reel est A

~ = Aey cos(t y) ~u.


E (49)

Lamplitude au point M = (x, y, z) est Ar (y) = Aey . Pour


0 y
y cette amplitude doit rester finie. On a donc > 0.

Fig. 59 Ar (y).

Pour = 0, londe (8) serait une onde plane sinusodale progressive se propageant dans la direction de
Oy vers les y croissants. Sa phase t y = (t y/v ) correspondrait `a la vitesse de phase v = /.
Dans le cas envisage, > 0, la phase est toujours t y. Londe se propage donc dans la direction de
Oy vers les y croissants, mais sattenue (lamplitude Ar (y) decrot pour y croissant).
162 `
14. DEUXIEME PARTIEL 2005-2006

Cette attenuation resulte de labsorption de londe par le milieu conducteur, lenergie de londe se trans-
formant en chaleur par effet Joule.
Nota. Londe (8) nest pas progressive pour > 0 (le signal sattenue). Londe ne depend ni de x ou z :
londe est plane et homog`ene. Les plans dondes sont les plans y = Cte.

4.b) On calcule ~ E~ = ( i)~u ~uy . Dapr`es (47), cette divergence est nulle. Le vecteur ~u doit donc
etre orthogonal `a ~uy .
~ est orthogonal `
Londe est transverse (E a la direction de propagation).

5.a) Portons (8) dans lequation de propagation (48) :



2
 ~
2
2 ( i) E ~ = 0 ~ = 0 E = i0 E
~
c t t

o`
u on a utilise la loi dOhm (5). Simplifions :

2 2 2i
( + i)2 = + i0 = + 2 (50)
c2 c2 L
r
2
avec L = = 3,72 106 m.
0

5.b) On peut negliger 2 /c2 = 289 m2 par rapport `a 2/L2 = 1,44 1011 m2 . On a donc approximative-
ment
2i
( + i)2 = 2 .
L
En prenant la racine carree, on a
1+i
+ i = .
L
u = = 1/L = 269 103 m1 .
Comme > 0, on doit prendre le signe +. Do`

Ar (d)
6) On exige que = ed ou, en prenant le logarithme, d ln . Le blindage doit donc
Ar (0)
avoir au moins lepaisseur
dmin = L ln = 0,103 mm. (51)
163

15. Examen 2005-2006


Enonc
e
15 decembre 2005
duree : 3 heures
sans documents, calculatrices autorisees

On rappelle quun resultat numerique donne sans unite est faux. Les parties A, B et C sont independantes.
Bar`eme indicatif : A = 7, B = 6 et C = 7.

Valeurs numeriques des constantes physiques


Permittivite du vide : 0 = 8,85 1012 F m1
Permeabilite du vide : 0 = 4 107 H m1
Vitesse de la lumi`ere dans le vide : c = 3,00 108 m s1
Charge electrique de lelectron : e avec e = 1,60 1019 C
Masse de lelectron : m = 0,911 1030 kg
Dans le syst`eme cartesien Oxyz, les vecteurs unitaires sont notes ~ux , ~uy , ~uz .

A. Ondes electromagn
etiques dans le milieu interstellaire et dis-
tance dun pulsar
Le milieu interstellaire est un plasma extremement rarefie constitue de ne electrons par unite de volume
et de divers ions et molecules.
On consid`ere une onde electromagnetique qui se propage dans ce milieu et donnee en notation complexe
par
~ = E0 ei(tkz) ~ux ,
E ~ = B0 ei(tkz) ~uy ,
B (1)
o`
u , k, E0 et B0 sont des constantes reelles et positives.
On demande de montrer, question 6), que et k sont lies par la relation de dispersion (7). Dans les
questions 1) `
a 5), vous repondrez en fonction de et k sans utiliser cette relation (7).
Les questions 7) et 8) peuvent etre resolues sans avoir repondu aux questions 1) `a 6).


1) Ecrire ~ et B.
les champs reels E ~
Decrire londe electromagnetique (1).
Quelle est sa longueur donde dans le milieu?
Quels sont le sens et la direction de propagation, le vecteur donde, les surfaces donde, la polarisation?
164 15. EXAMEN 2005-2006

2) Calculer le vecteur de Poynting P~ (M, t) du champ electromagnetique (1), au point M = (x, y, z) et `a


linstant t, en fonction de E0 , B0 , , k, c et 0 .


3) Ecrire les equations de Maxwell dans le vide.
Sachant que le champ electromagnetique (1) verifie ces equations de Maxwell, en deduire :

a) la densite de charge electrique (M, t) dans le milieu ;

b) la valeur de B0 en fonction de , k et E0 ;

c) que la densite de courant ~ (M, t) dans le milieu secrit en notation complexe

~
~ = E (2)

o`
u est un nombre complexe que lon exprimera en fonction de , k, c et 0 .

4) On note ~v (x, y, z, t) = vx ~ux + vy ~uy + vz ~uz le champ de vitesse des electrons du milieu (un electron
situe en M = (x, y, z) ` a linstant t a pour vitesse ~v (x, y, z, t)). On fait lhypoth`ese que les electrons ont
un mouvement oscillatoire parall`ele ` a laxe Ox et que leur champ de vitesse ne depend ni de x ni de y.
On ecrira donc, en notation complexe,

~v(z, t) = v0 ei(tkz) ~ux . (3)

o`
u v0 est une constante complexe.
On utilisera dans la suite lequation suivante

~v ~
m = eE. (4)
t
Cette equation est lequation du mouvement dun electron (masse m, charge e) soumis `a la force
electrique F~e = eE.
~ On a ecrit, pour lacceleration de lelectron,

d~v ~v ~v ~v ~v ~v
= + vx + vy + vz = (5)
dt t x y z t

ce qui est exact pour le champ de vitesse (3).

a) En realite, lelectron subit egalement une force magnetique F~m due au champ B
~ de londe.
Quelle est lexpression de F~m ?

b) Quelle condition la constante v0 doit-elle satisfaire pour que la force magnetique F~m soit negligeable
par rapport `a la force electrique F~e ?

c) En utilisant lequation (4), determiner v0 en fonction de m, , e et E0 .

5) La densite de courant est due au mouvement des electrons :

~ = ne e~v . (6)
165

a) Justifier cette affirmation : on expliquera lequation (6) et pourquoi on peut negliger les courants dus
aux ions.

~
b) En utilisant lequation (6), exprimer ~ en fonction de ne , e, m, et E.

6.a) Montrer que londe etudiee verifie une relation de dispersion de la forme

2 = k 2 c2 + p2 (7)

et exprimer la valeur de p en fonction de ne , e, m et 0 .

b) Que pouvez-vous dire de londe pour 0 < < p ?

c) Application numerique. On mesure p = 1,0 104 s1 . Calculer ne .

7) Calculer la vitesse de phase v et la vitesse de groupe vg de londe dans le milieu en fonction de c, p


et .
Quelle relation existe-t-il entre v et vg ?
Comparer les vitesses vg et v avec la vitesse de la lumi`ere dans le vide.
Donner une valeur numerique de qui correspond `a de la lumi`ere visible rouge.
Calculer numeriquement les vitesses vg et v pour cette lumi`ere.

8) Un pulsar situe `a la distance L nous envoie des impulsions electromagnetiques denviron 50 milli-
secondes et se repetant `
a une periode voisine de la seconde. Chaque impulsion est une superposition
dondes du type (1) pour diverses frequences. A lemission dune impulsion par le pulsar, ces diverses
ondes sont emises simultanement quelle que soit leur frequence. Mais, sur Terre, apr`es propagation `a la
vitesse de groupe vg dans le plasma interstellaire, il y a un retard entre les temps darrivee des differentes
frequences.
Exprimer le retard t entre les temps darrivee des composantes de frequences 1 et 2 (1 < 2 ). On
supposera que ne est constant sur le trajet de londe et on simplifiera lexpression en tenant compte du
fait que 1 p et 2 p . On donnera t en fonction de c, p , 1 , 2 et L.
Quelle est la composante qui arrive en premier?
On mesure un retard t = 0,11 s pour les frequences 1 = 400 MHz et 2 = 500 MHz.
En deduire la distance L en parsecs (1 pc = 3,09 1016 m).
Le pulsar est-il dans notre galaxie? Notre galaxie a un diam`etre de 30 kpc.

B. Propagation guid
ee : ligne `
a rubans
Une ligne ` a rubans est constituee de deux rubans conducteurs parfaits, de longueur infinie, de largeur
b = 2 cm et de faible epaisseur e (e b). Les deux rubans sont disposes parall`element lun au dessus de
lautre `a la distance a = 2 mm (cf. figure 60).
Soit un referentiel cartesien Oxyz, le demi-axe Ox etant dirige vers le haut. On appelle interieur de la
ligne la region I de lespace situee entre les rubans :

I = (x, y, z) R3 : 0 < x < a, |y| b/2, z + . (8)
166 15. EXAMEN 2005-2006

Les faces des rubans qui bordent linterieur de la ligne sont


appelees face A (sur le ruban inferieur) et face B (sur le x
ruban superieur).
La face A occupe la bande |y| b/2 du plan x = 0. La face b Face B
cach
ee
B, cachee sur la figure, occupe la bande |y| b/2 du plan
z
x = a. Lespace autour des rubans (y compris linterieur
Face A
de la ligne) est vide. e
La face A est parcourue par des courants de densite surfa-
cique ~ (z, t) = j (z, t) ~uz et porte une densite surfacique a
de charge (z, t). Les densites j (z, t) et (z, t) au point O
e y
P = (0, y, z) de la face A ne dependent pas de y.

Fig. 60 Ligne `a rubans.

La face B est parcourue par des courants de densite surfacique ~ (z, t) = j (z, t) ~uz et porte la densite
surfacique de charge (z, t).
Il ny a aucun autre courant ou charge dans le syst`eme.
~ et magnetique B
On etudiera les champs electrique E ~ uniquement `a linterieur de la ligne. On suppose
~ et B
que dans cette region E ~ ne dependent que de z et de t (on neglige les effets de bord) et sont donnes
au point M = (x, y, z) I par :

~ t) = E(z, t) ~ux ,
E(z, ~ t) = B(z, t) ~uy
B(z, (9)


1) Ecrire les relations de passage sur les faces A et B.
En deduire lexpression des densites surfaciques de charge (z, t), (z, t) et de courant j (z, t), j (z, t)
en fonction de E(z, t) et B(z, t).

2) On consid`ere une tranche T depaisseur infinitesimale dz de la region I. La tranche T est le pa-


rallelepip`ede rectangle de dimensions a b dz entre les cotes z et z + dz :
T = ]0, a[ [b/2, b/2] [z, z + dz] . (10)
Determiner, en fonction de E(z, t), lenergie electrique dUe contenue dans la tranche T
Quelle est la charge surfacique dq sur la face A entre les cotes z et z + dz ?
(dq)2
Montrer que lenergie electrique dUe secrit dUe = et exprimer C en fonction de a, b et 0 .
2Cdz
Quelle est la signification physique de C ? En quelle unite C est-il mesure ?
Calculer la valeur numerique de C.

3) Determiner, en fonction de B(z, t), lenergie magnetique dUm contenue dans la tranche T .
Quelle est lintensite totale I(z, t) du courant dans le ruban inferieur?
Ldz(I(z, t))2
Montrer que lenergie magnetique dUm secrit dUm = et exprimer L en fonction de a, b et
2
0 .
Quelle est la signification physique de L? En quelle unite L est-il mesure ?
Calculer la valeur numerique de L.
Quelle relation existe-t-il entre L et C ?


4) Ecrire les equations de Maxwell dans le vide, en labsence de charges et courants.
167

Que deviennent ces equations pour les champs donnes par lequation (9)?
En deduire, en utilisant les resultats de la question 1), deux equations aux derivees partielles satisfaites
par (z, t) et j (z, t).

5) Demontrer que (z, t) verifie une equation de dAlembert.


Quelle est la vitesse de propagation des ondes correspondant `a cette equation?
En utilisant le theor`eme de dAlembert, ecrire la solution generale de cette equation.
Quelle est la signification physique de cette solution generale?

6) On suppose que la densite surfacique de charge est donnee par

(z, t) = A cos(t kz) (11)

o`
u A, et k sont des constantes positives.
Exprimer et k en fonction de la frequence de londe et de c (on justifiera le calcul de k).
~ et B
Calculer, en fonction de A, la densite surfacique de courant j (z, t) et les champs E ~ `a linterieur de
la ligne.
E
Que vaut le rapport ? Decrire la structure de londe electromagnetique entre les rubans.
B
Londe est-elle TEM (transverse electrique et magnetique)?

C. Dispositif interf
erentiel des trous dYoung
x
L L
E
O1

d M
z S z
O

O2

f f

Fig. 61 Dispositif des trous dYoung. Laxe z Oz est laxe optique des lentilles L et L. Le point O est
le foyer image de la lentille L.

Le dispositif interferentiel des trous dYoung est schematise sur la figure 61 : il comporte une source lumi-
neuse ponctuelle et monochromatique, placee au foyer objet S dune lentille convergente L , de distance
focale f = 2,6 m. La lumi`ere diffractee par les trous O1 et O2 est recue par une lentille convergente L de
distance focale f = 1,5 m. On designe par 0 = 579,1 nm la longueur donde dans le vide de la lumi`ere
emise par la source, par d = 5,000 mm la distance des deux trous et par n = 1,00029 lindice de lair dans
lequel lensemble du dispositif est plonge.

1) Effectuer la construction geometrique des deux rayons issus de S qui arrivent au point M du plan
focal image de la lentille L. On peut utiliser la page 169 pour la reponse.

2) On suppose que le point M est situe sur laxe Ox parall`ele `a la ligne O2 O1 . On notera x labscisse du
point M .
168 15. EXAMEN 2005-2006

Calculer la difference de marche des deux ondes qui interf`erent au point M en fonction de x, f , d et n.

3) Decrire bri`evement le phenom`ene observe sur un ecran E occupant le plan focal image Oxy de la
lentille L.
Calculer litteralement et numeriquement linterfrange i.

4) On rappelle que leclairement de lecran au point M est donne par

I(x) = 2I0 (1 + cos ) (12)

o`
u est le dephasage entre les deux ondes qui interf`erent au point M .
Exprimer en fonction de x et i.
Tracer la courbe de I(x) en fonction de x.

5) On suppose que la longueur donde dans le vide de la lumi`ere emise est 0 = 576,9 nm au lieu de 0 .
Calculer la valeur numerique i du nouvel interfrange.

6) La source lumineuse est maintenant dichromatique : la lumi`ere emise est composee des deux rayonne-
ments monochromatiques envisages ci-dessus et dont les longueurs donde dans le vide sont 0 et 0 . Ces
deux composantes monochromatiques ont la meme puissance demission. Leurs longueurs donde etant
tr`es voisines, elles paraissent de la meme couleur (jaune) sans que lil puisse distinguer lune de lautre.
On appelle ordre dinterference de la composante de longueur donde 0 (respectivement 0 ) le rapport

p= (respectivement p = ).
0 0

a) En quels points de lecran les ordres dinterference p et p sont-ils egaux?


1
En quels points de lecran les ordres dinterference p et p diff`erent-ils de 2?
En quels points de lecran les ordres dinterference p et p diff`erent-ils de 1?
Dans chacun des cas demandes, on exprimera les abscisses des points de lecran en fonction de i et
i et on calculera leurs valeurs numeriques. On donnera egalement les valeurs numeriques des ordres
dinterference p et p .

b) On observe les interferences sur lecran E.


Exprimer leclairement I1 (x) de lecran E au point M dabscisse x en fonction de x, i, i et de sa valeur
maximale Imax .
Expliquer pourquoi on peut dire que leclairement presente le phenom`ene des battements.

c) Decrire les trois franges brillantes et les deux franges sombres les plus proches de O (donner leurs
positions et estimer leurs eclairements). Que vaut le contraste
Imax Imin
C= (13)
Imax + Imin
des franges en O, Imin etant leclairement des franges sombres les plus proches de O ?

1
u les ordres dinterference p et p diff`erent de
d) Que se passe-t-il au voisinage des points de lecran o` 2?
u les ordres dinterference p et p diff`erent de 1?
Que se passe-t-il au voisinage des points de lecran o`
Num
ero danonymat :

Feuille `
a rendre avec la copie
C.1) Construction g
eom
etrique des deux rayons qui arrivent au point M

L L

E
O1

z S z
O

O2

f f

169
170 15. EXAMEN 2005-2006

Corrig
e
A. Ondes
electromagn
etiques dans le milieu interstellaire et distance dun
pulsar

1) Les champs reels sont

~ = E0 cos(t kz)~ux, et B
E ~ = B0 cos(t kz)~uy . (14)

Londe electromagnetique (1) est une onde plane progressive harmonique homog`ene (OPPH). Le vecteur
donde est ~k = k~uz . Londe est TEM (transverse electrique et magnetique) et les trois vecteurs (~k, E,
~

~ forment un tri`edre orthogonal direct. La longueur donde dans le milieu est = 2 . Elle nest pas
B)
k
2c
egale `a la longueur donde dans le vide dune onde de meme frequence 0 = car la vitesse de phase

v = /k nest pas egale `
a c (voir plus bas la question 7).
Londe se propage selon Oz, vers les z croissants. Les surfaces donde sont les plans z = Cte (les plans
perpendiculaires `
a Oz). Londe est polarisee rectilignement suivant Ox.

2) Pour calculer le vecteur de Poynting P~ = 0 c2 E


~ B,
~ il faut utiliser les champs reels (14) :

P~ = 0 c2 E0 B0 cos2 (t kz) ~uz . (15)

3) Les equations de Maxwell dans le vide secrivent

~ = ,
~ E (16)
0
~ B
~ = 0, (17)
~
~ E ~ = B , (18)
t
~
~ B ~ = 0~ + E . (19)
2
c t
~ ~
Pour le champ electromagnetique (1), on calcule E = 0, ~ B
~ = 0,
~ ~
~ E
~ = ikE0 ei(tkz) ~uy ,
~ B~ = ikB0 ei(tkz) ~ux , B = i B,~ E = i E.
~
t t
On peut noter que lequation (17) est satisfaite.

a) Lequation (16) donne = 0 (le milieu est electriquement neutre).

b) Lequation (18) donne ikE0 ei(tkz) ~uy = iB0 ei(tkz) ~uy do`
u

kE0
B0 = . (20)

171

i ~ k ~ ~ i ~
c) Lequation (19) donne ikB0 ei(tkz) ~ux = 0~ + 2 E. Avec (20) il vient ik E = 2 + 2E do`
u
c c 0 c

~ k 2 c2 2
~ = E avec = i0 . (21)

4.a) La force est F~m = e~v B.


~ Posons v0 = |
v0 |ei . On a v = |
v0 | cos(tkz +), B = B0 cos(tkz)
et
F~m = ev~ux B~uy = evB~uz = e| v0 |B0 cos(t kz + ) cos(t kz)~uz .

b) Lordre de grandeur de la force magnetique est donc e|


v0 |B0 . Lordre de grandeur de la force electrique
e|
v0 |B0 |
v0 |k
etant eE0 , on pourra negliger la force magnetique si leur rapport = est tr`es petit par
eE0
rapport `a 1. La condition est donc

|
v0 | . (22)
k
|
v0 |
On verra plus bas que v = & c. On peut ecrire aussi la condition sous la forme 1 qui signifie
k c
que les electrons ont des vitesses petites par rapport `a c.

c) Portons (1) et (3) dans lequation (4):


v0 ei(tkz) ~ux = eE0 ei(tkz) ~ux .
mi
On a donc
eE0
v0 = i . (23)
m

5.a) Lequation (6) est la relation ~ = e~v qui donne la densite de courant dune classe de particules (les
electrons) de vitesse ~v et de charge volumique e = ne e.
Le mouvement dun ion de charge Ze et masse M sobtient par la meme methode que pour un electron.
On vient de montrer que lamplitude de la vitesse dun electron est |
v0 | = eE0 /m. Celle de lion est
V = ZeE0 /M qui est tr`es petite par rapport `a |
v0 | :
V Zm
= 1.
|
v0 | M

Les charges volumiques des ions et des electrons etant du meme ordre de grandeur (le milieu est neutre),
les courants dus aux ions sont negligeables par rapport `a ceux dus aux electrons. Tout se passe comme
si les ions etaient immobiles.

eE0 i(tkz) ne e2 E0 i(tkz)


b) Portons ~v(z, t) = i e ~ux dans lequation (6) : ~ = i e ~ux . On a donc
m m
2
ne e ~
~ = i E. (24)
m

6.a) Comparons (21) et (24) :


k 2 c2 2 ne e 2
= i0 = i
m
172 15. EXAMEN 2005-2006

En rearrangeant :
ne e 2
k 2 c2 2 + =0
m0
On a donc obtenu la relation de dispersion
s
ne e 2
2 = k 2 c2 + p2 o`
u p = . (25)
m0

q
p2 2
b) Pour 0 < < p , cette relation donne k = i . Londe ne se propage pas (onde evanescente).
c

Seulement les ondes de frequences plus grande que la frequence de coupure c = p /2 peuvent se
propager dans le milieu.

m0 p2
c) Application numerique. On a ne = = 3,15 104 m3 .
e2

c
7) La vitesse de phase est v = =q soit
k 2 p2

c
v = r  2 . (26)
p
1

d d
La vitesse de groupe est vg = . En differentiant (25), il vient 2d = 2c2 kdk soit = c2 . On a la
dk k dk
relation v vg = c2 entre v et vg . On en tire la vitesse de groupe

r  2
p
vg = c 1 . (27)

La vitesse de groupe est inferieure `


a la vitesse de la lumi`ere dans le vide qui est elle-meme inferieure `a
la vitesse de phase :
vg < c < v .

Considerons la lumi`ere visible rouge de longueur donde dans le vide 0 = 8 107 m. La pulsation a pour
valeur
2c
= = 2,3 1015 s1 . (28)
0
On a p et
vg = 3,00 108 m s1 v = 3,00 108 m s1 (29)

8) La composante de frequence met pour arriver jusqu`


a nous le temps
 ! !
L L  2 1/2 L p2 L p2
p
t() = = 1 1+ = 1+ 2 2 .
vg c c 2 2 c 8
173

On a utilise le fait que p . La composante de plus haute frequence (2 ) arrive en premier. Le retard
= temps darrivee de la composante de frequence 1 - le temps darrivee de la composante de frequence
2 est
 
Lp2 1 1
t = 2 2 . (30)
8 c 12 2

8 2 ct 1
Application numerique : L = = 1,2 1019 m = 375 pc. Le pulsar est dans notre galaxie,
p2 1 1
2
12 2
comme la plupart des pulsars connus.

B. Propagation guid
ee : ligne `
a rubans

1) Le champ est nul dans les rubans (conducteur parfait). Les relations de passage donnent E(z, ~ t) =

(z, t) ~ t) = (z, t) ~ux , B(z,
~ t) = 0~ (z, t) ~ux sur la face A et E(z, ~ t) = 0~ (z, t) ~ux sur
~ux , B(z,
0 0
la face B. On en deduit

B(z, t)
(z, t) = (z, t) = 0 E(z, t) et j (z, t) = j (z, t) = . (31)
0

~ est uniforme dans T )


2) Lenergie electrique est (E

0 E 2 abdz
dUe = .
2
2 abdz (dq)2 a (dq)2
La charge surfacique est dq = b dz. On a dUe = = = avec
20 20 bdz 2Cdz

0 b
C= = 8,85 1011 F m1 . (32)
a

On peut remarquer que Cdz est la capacite dun condensateur plan de surface bdz et depaisseur a. C
est donc la capacit
e lin
eique de la ligne (ou capacite par unite de longueur de la ligne) et est mesuree
en farads par m` etre.

~ est uniforme dans T )


3) Lenergie magnetique est (B

B 2 abdz
dUm = .
20

Lintensite totale du courant dans le ruban inferieur est I(z, t) = bj (z, t).
0 j2 abdz 0 adzI 2 LdzI 2
On a dUm = = = avec
2 2b 2
0 a
L= = 1,26 107 H m1 . (33)
b

La grandeur L est l inductance propre lin eique de la ligne (ou autoinductance par unite de longueur
de la ligne) et est mesuree en henrys par m`
etre.
174 15. EXAMEN 2005-2006

1
La relation entre L et C est LC = 0 0 = .
c2

4) En labsence de charges et courants ( = 0, ~ = 0), les equations de Maxwell sont :

~ E
~ = 0, (34)

~ B
~ = 0, (35)
~
~ E ~ = B , (36)
t
~
~ B
~ = E . (37)
2
c t
Pour les champs donnes par lequation (9), ces equations deviennent respectivement :

~ = E(z, t) = 0
~ E qui est verifie pour tout E(z, t). (38)
x

~ = B(z, t) = 0 qui est verifie pour tout B(z, t).


~ B (39)
y

~ = E(z, t) ~uy = B(z, t) ~uy qui implique


~ E E(z, t) B(z, t)
+ = 0. (40)
z t z t
~ B
~ = B(z, t) E(z, t) B(z, t) E(z, t)
~ux = 2
~ux qui implique + = 0. (41)
z c t z c2 t
En utilisant (31) et 0 0 c2 = 1, on obtient :

(z, t) 1 j (z, t)
+ 2 =0 (42)
z c t

et
j (z, t) (z, t)
+ = 0. (43)
z t
On peut remarquer que cette derni`ere equation est lequation de continuite de la charge.

5) En eliminant j dans les equations precedentes, il vient

1 2 (z, t) 2 (z, t)
=0 (44)
c2 t2 z 2

qui est une equation de dAlembert. La vitesse de propagation des ondes correspondant `a cette equation
est c, la vitesse de la lumi`ere dans le vide. La solution generale secrit

(z, t) = F (z ct) + G(z + ct), (45)

o`
u F (u) et G(u) sont des fonctions arbitraires. Cette solution generale est la superposition de deux ondes
planes progressives se propageant lune vers les z croissants et lautre vers les z decroissants.

6) On a = 2. (z, t) etant solution de lequation (44), on a 2 /c2 = k 2 . Do`


u (k > 0)

2
k= = . (46)
c c
175

En representation complexe, on a = Aei(tkz) . La grandeur (z, t) en representation complexe depend


i(tkz)
de z et t par le facteur e . On a, en portant dans lequation (43), ik (z, t) + i
(z, t) = 0 puis
j (z, t) = c(z, t) et
j (z, t) = cA cos(t kz). (47)
Les formules (31) donnent pour les champs

~ t) = A cos(t kz) ~ux ,


E(z, ~ t) = 0 cA cos(t kz) ~uy
B(z, (48)
0

E A E
Le rapport des champs est = soit = c. Les trois vecteurs ~k = k~uz , E
~ et B
~ forment un
B 0 0 cA B
tri`edre orthogonal direct : la structure de londe electromagnetique entre les rubans est la meme que celle
dune OPPH dans le vide. Londe est TEM (transverse electrique et magnetique).

C. Dispositif interf
erentiel des trous dYoung

1) Les rayons arrivent en O1 et O2 parall`element `a laxe z Oz. Ils en partent parall`element `a CM , C


etant le centre de la lentille L.
L L x

O1 E

d M
z S C z
O

H
O2

f f x

Fig. 62 Construction des rayons.

2) Calculons la difference de marche = [SO2 M ] [SO1M ]. Soit H la projection orthogonale de O1 sur


le rayon partant de O2 . Il y a egalite des chemins optiques [HM ] = [O1 M ] par stigmatisme de la lentille
L. On en deduit = [O2 H] = nO2 H. Langle = zCM \ = O\ 2 O1 H est petit par rapport `
a 1 (conditions
de lapproximation de Gauss). On a, dans les triangles rectangles O1 HO2 et COM ,

x O2 H
tan = sin = = .
f d nd
On en deduit
nxd
= . (49)
f

3) On observe le phenom`ene dinterference. Les franges sont rectilignes et parall`eles `a Oy. La frange
brillante dordre p (p entier) correspond `a = p0 et est situee `a labscisse x = pf 0 /nd. Linterfrange
est la distance entre la frange dordre p + 1 et la frange dordre p. Cest donc

f 0
i= = 0,17368 mm (50)
nd

qui ne depend pas de p : linterfrange est constant.


176 15. EXAMEN 2005-2006

I
4) Le dephasage des deux ondes qui interf`erent au point
M est
2 2nxd 2x
= = = .
0 f 0 i
Leclairement de lecran au point M est donne par
 
2x x
I(x) = 2I0 1 + cos = 4I0 cos2 . (51)
i i
2i i 0 i 2i x


Fig. 63 Eclairement I en fonc-
tion de x.

5) Linterfrange devient
f 0
i = = 0,17302 mm. (52)
nd

6.a) Les ordres dinterference en M = (x, y) E sont p = x/i et p = x/i . Les ordres dinterference p
et p sont egaux pour x = x0 = 0, cest-`
a-dire pour la frange centrale dordres dinterference p = p = 0

(laxe y Oy).
On a p p = 1/2 pour x = pi = (p 1/2)i do`
u p(i i ) = i /2 et

ii
x = x1/2 = = 2,277 cm. (53)
2(i i )

Ce sont les deux franges dordres dinterference p = p1/2 = 131,11 et p = p1/2 = 131,61.
On a p p = 1 pour x = pi = (p 1)i do`
u p(i i ) = i et

ii
x = x1 = = 4,554 cm. (54)
(i i )

Ce sont les deux franges dordres dinterference p = p1 = 262,23 et p = p1 = 263,23.

b) Les deux composantes monochromatiques de longueurs donde dans le vide 0 et 0 sont mutuellement
incoherentes. Leclairement en M est la somme des eclairements des deux composantes :
x x
I1 (x) = 2I0 [1 + cos(2p)] + 2I0 [1 + cos(2p )] avec p = , p = . (55)
i i

Les deux composantes ayant la meme puissance on a I0 = I0 . Leclairement est maximum en x = 0 et


vaut Imax = 8I0 . On a donc
 
Imax 2x 2x
I1 (x) = 2 + cos + cos
4 i i
        
Imax 1 1 1 1
= 1 + cos + x cos x (56)
2 i i i i
177

Leclairement en fonction de x est la somme dune constante et de deux sinusodes de periodes spatiales
voisines i et i . Il presente donc le phenom`ene des battements.

I1

x x1 x1/2 O x1/2 x1 x

Fig. 64 Leclairement en fonction de x. En realite, il y a beaucoup plus doscillations rapides que


representees (entre x = 0 et x = x1/2 il devrait y en avoir environ p1/2 131).

Linterfrange (la distance entre deux franges brillantes voisines) est compris entre les valeurs i et i et
varie tr`es leg`erement dune frange `
a lautre.

i + i
c) Les trois franges brillantes les plus proches de O sont les lignes x = 0 et x = i avec i =
2
0,17335 mm. Elles ont toutes trois pratiquement le meme eclairement I1 (i ) I1 (0) = Imax .
Les franges sombres sont approximativement au milieu de deux franges brillantes voisines. Celles qui
i i + i
encadrent O sont en x = = 0,086674 mm. Ces franges sombres sont pratiquement
2 4
noires : leur eclairement est Imin = I1 (i /2) = 0,89364 105Imax et le contraste des franges en O vaut

Imax Imin
C= 1. La visibilite des franges est maximale.
Imax + Imin

d) Au voisinage des points de lecran o`u les ordres dinterference p et p diff`erent de 12 (autour de x = x1/2
et x = x1/2 ) les franges brillantes dune composante monochromatique sont les franges noires de lautre.
Les franges se brouillent et le contraste des franges est pratiquement nul.
Au voisinage des points de lecran o` u les ordres dinterference p et p diff`erent de 1 (autour de x = x1
et x = x1 ) les franges brillantes des deux composantes monochromatiques se superposent. Le contraste
des franges est maximum comme autour de lorigine.
178 16. EXAMEN (2E SESSION) 2005-2006

16. Examen (2e session) 2005-2006


Enonc
e
Mardi 13 juin 2006
duree : 3 heures
sans documents, calculatrices autorisees

Bar`eme indicatif : A = 7 ; B = 6 ; C = 3 ; D = 4.

A. Corde fix
ee aux deux extr
emit
es
On consid`ere une corde tendue de longueur L fixee en ses extremites (les points x = 0 et x = L de laxe
Ox). La vitesse de propagation des ondes transverses de la corde est c.

1) Ecrire lequation donde de dAlembert verifiee par le deplacement s(x, t) des points de la corde.

2) On rappelle que toute deformation transversale de la corde est representee par :



X  
s(x, t) = sn (x, t) avec sn (x, t) = an cos(n t) + bn sin(n t) sin(kn x), (1)
n=1

u n et kn sont des constantes strictement positives telles que n < n+1 n N et o`


o` u les coefficients
an et bn sont determines par les conditions initiales.
En utilisant le fait que sn verifie lequation donde de dAlembert et les conditions aux limites du
probl`eme, determiner les constantes n et kn en fonction de n, L et c.

3) Exprimer
s
s0 (x) = s(x, t = 0) et v0 (x) = (x, t = 0) (2)
t
en fonction des an et bn .

4) Inversement, determiner an et bn en fonction de s0 (x) et v0 (x). On pourra utiliser


Z L (
L/2 si m = n ;
sin(kn x) sin(km x) dx = (3)
0 0 si m 6= n.
179

5) A linstant t = 0, la corde, qui est dans la position dequilibre s0 (x) = 0, est frappee avec un marteau
de largeur 2 L situe en x = . Ce marteau communique `a la corde une vitesse initiale en forme de
creneau :
0, si 0 x ;

v0 (x) = v, si x + ; (4)


0, si + x L.

Determiner les coefficients an et bn en fonction de v, , , n, L et c.


Simplifier ces expressions lorsque n L.

6) Montrer que lorsque = L/2 les harmoniques pairs disparaissent de la serie (1).

7) Montrer que le mouvement de la corde est periodique (s(x, t + T ) = s(x, t)) et determiner la periode
T.

B. R
eflexion dune onde plane
electromagn
etique
Notations et valeurs numeriques pouvant etre utiles a
` la resolution du probl`eme :

On utilise un syst`eme cartesien Oxyz de vecteurs unitaires ~ux , ~uy , ~uz .


Vitesse de la lumi`ere dans le vide : c = 3,00 108 m s1 ;
Permittivite du vide : 0 = 8,85 1012 F m1 ;
Permeabilite du vide : 0 = 4 107 H m1 .

Une onde plane (onde incidente) electromagnetique de pulsation se propageant dans le vide est donnee
par le champ electrique
~ i = A cos (t + kx) ~uz ,
E (5)
u k > 0 et A > 0. La frequence de londe est = 5 1013 Hz.
o`

1) Calculer numeriquement la longueur donde .


Quel est le domaine spectral de cette radiation ?
Quelle est la relation entre et k ?


2.a) Ecrire les equations de Maxwell dans le vide en absence de charges et courants volumiques.
~ i de londe.
b) En deduire le champ magnetique B

3) Determiner la densite denergie electromagnetique ui et le vecteur de Poynting P~i de cette onde.


Quelle est la relation entre ui et P~i ?
Calculer les moyennes temporelles < ui > et < P~i >.
En quelle unite < P~i > est-il mesure ?
On mesure pour le module de < P~i > la valeur 13,3 (unite S.I. demandee ci-dessus). En deduire la valeur
numerique de < ui > et lamplitude A de londe (5).

4) Londe incidente (5) se propage dans le demi-espace (vide) x > 0. Le demi-espace x 0 est forme
dun metal parfaitement conducteur de sorte que le plan x = 0 forme un miroir metallique. Il apparat
une onde reflechie qui se superpose `
a londe incidente et dont le champ electrique est de la forme
~ r = A cos (t kx) ~uz .
E (6)
180 16. EXAMEN (2E SESSION) 2005-2006

~ r de londe reflechie.
Determiner le champ magnetique B

5) Representer sur le meme schema les 6 vecteurs suivants : le vecteur donde et les champs electrique et
magnetique de chacune des ondes reflechie et incidente au point x = y = z = 0 `a linstant t = 0.

6.a) Le miroir etant suppose parfait, les champs electrique et magnetique sont nuls dans le metal. Justifier
cette affirmation.

b) Ecrire les relations de passage que verifient les champs electrique et magnetique `a la traversee du plan
~
x = 0. On ecrira ces relations en fonction de E(0+, ~
t) = limx0+ E(x, ~
t), B(0+, ~
t) = limx0+ B(x, t), de
la densite surfacique de charge (t) et du courant surfacique ~ (t).

c) En deduire la densite surfacique de charge (t) et la densite surfacique de courant ~ (t).


Verifier que les relations de passage sont satisfaites.
Calculer numeriquement lamplitude de ~ (t).

C. Induction

1) Questions de cours A A
p s

a) Enoncer la loi de linduction.

b) Enoncer la loi de Lenz p s


R K R
2) On consid`ere les deux circuits p et s ci-contre. Ap et As sont
deux bobines mutuellement couplees par induction.
On ferme le circuit p (on abaisse linterrupteur K brusquement) `a linstant t = 0. Il apparat juste apr`es
un courant dintensite Is (t) dans le circuit s ; ce courant circule dans le sens de la fl`eche.

a) Comment expliquez-vous lexistence de ce courant?

b) On attend jusqu` a linstant t1 ; on a alors atteint un etat stationnaire (courants constants dans les
circuits, le circuit p etant toujours ferme).
Quel est alors lintensite du courant dans le circuit s? Justifier la reponse.

` linstant t2 (t2 > t1 ), on ouvre le circuit p (on l`eve linterrupteur K brusquement).


c) A
Decrire ce quil se passe.
Dans quel sens le courant circule-t-il dans le circuit s? Justifier la reponse.

D. Appareil photographique
Un appareil photographique a pour objectif une lentille convergente L, de distance focale

f = 50 mm,
181

pourvue dun diaphragme circulaire de diam`etre D variable.


On appelle nombre douverture le rapport
f
N=
D
qui peut prendre les valeurs

N = 4, N = 5,6, N = 8, N = 11, N = 16 et N = 22.

1) Lappareil recoit une onde plane provenant du point `a linfini sur laxe de L.
Comparer entre-elles les puissances lumineuses entrant dans lappareil pour les differentes valeurs de N .
Comparer entre-eux les temps dexposition necessaires pour avoir le meme effet sur la pellicule.

2) Calculer le rayon du premier anneau noir de la figure de diffraction pour les differentes valeurs de
N , en prenant pour longueur donde de la lumi`ere

= 0,55 m (jaune).

En deduire, pour chaque N , le pouvoir separateur theorique th de cet objectif, cest-`a-dire la plus petite
distance angulaire separant deux points `a linfini dont lappareil peut donner des images separees. On
donnera th en minute darc.

3) La pellicule photographique est une emulsion contenant des grains dhalogenure dargent de 10 m de
diam`etre. Quel est le pouvoir separateur de lensemble objectif + pellicule pour chaque N ?
182 16. EXAMEN (2E SESSION) 2005-2006

Corrig
e
A. Corde fix
ee aux deux extr
emit
es

1) Lequation donde de dAlembert est

1 2s 2s
2 2
2 = 0. (7)
c t x

2 sn 2 2 sn
2) On calcule = n s 1 et = kn2 sn .
t2 x2
  2 
2 n
Lequation donde (7) donne kn sn = 0 do`
u on tire n = kn c.
c
La condition aux limites sn (x = 0, t) = 0 est identiquement verifiee. La condition aux limites sn (x =
L, t) = 0 donne kn L = p avec p entier. La suite k1 , k2 , k3 , . . . etant positive et croissante, on doit
prendre p = n et
n nc
kn = , n = .
L L

3)

X
X
s0 (x) = an sin(kn x), v0 (x) = bn n sin(kn x). (8)
n=1 n=1

RL
4) On multiplie (8) par sin(km x) et on int`egre en x de 0 `a L : 0
s0 (x) sin(km x) dx = Lam /2. On a donc
Z L
2
an = s0 (x) sin(kn x) dx.
L 0

De meme, on trouve
Z L
2
bn = v0 (x) sin(kn x) dx.
Ln 0

Z L Z L
2 2
5) On calcule an = s0 (x) sin(kn x) dx = 0 et bn = v0 (x) sin(kn x) dx
L 0 | {z } Ln 0
=0
Z +
2v 2v 4v
= sin(kn x) dx = [cos kn ( ) cos kn ( + )] = sin(kn ) sin(kn ).
Ln Ln kn Lckn2
On a donc
4vL n n
an = 0, bn = sin sin .
n2 2 c L L

Lorsque n L on peut ecrire


4v n
an = 0, bn sin .
nc L
183

n
6) Pour = L/2 et n = 2p pair, on a sin = sin p = 0. On a donc a2p = b2p = 0 et les harmoniques
L
pairs disparaissent de la serie (1).

2 2L
7) Chaque mode sn etant periodique de periode Tn = = , la serie (1) est periodique de periode
n nc
2L
T = qui est le plus petit multiple entier de tous les Tn (T = nTn ).
c

B. R
eflexion dune onde plane
electromagn
etique

1) La longueur donde est = c/ = 6 m. Le domaine spectral de cette radiation est la lumi`ere


infra-rouge. La relation entre et k est = kc.

2.a) Les equations de Maxwell secrivent

1 E ~
~ B
~ = (9)
2
c t
~ E
~ = 0 (10)
B~
~ E
~ = (11)
t
~ B
~ = 0. (12)

2.b) Lequation (11) donne, pour B ~i dependant du temps par le facteur eit ,

/x 0 0
~ i = ~i = /y

i B ~ E 0 = ikAeit+ikx .

it+ikx
/z Ae 0

On en tire
B~i = A eit+ikx ~uy .
c
~ i = A cos(t + kx)~uy .
B
c

3) La densite denergie electromagnetique est

0 Ei2 B2
ui = + i = 0 A2 cos2 (t + kx).
2 20
Le vecteur de Poynting est
E ~i
~i B A2
P~i = = cos2 (t + kx)~ux .
0 0 c

La relation entre ui et P~i est P~i = cui ~ux .


0 A2 c0 A2
Les moyennes temporelles sont < ui >= et < P~i >= ~ux .
2 2
Lunite de < P~i > est le watt par m`etre carre (W m2 ).
184 16. EXAMEN (2E SESSION) 2005-2006
p
On a < ui >= | < P~i > |/c = 4,42 108 J m3 et A = 2 < ui > /0 = 100 V m1 .
x
~
kr
4) Le vecteur donde de londe reflechie est ~kr = k~ux et son champ
magnetique
~ ~
B~ r = kr Er = A cos (t kx) ~uy . ~r
E O
~i
E
c z

~ i =B
B ~r
5) Les 6 vecteurs sont ~ki = k~ux , ~kr = ~ki = k~ux , E
~ i = A~uz ,
~
ki
~ i = A ~uy et B ~ i = A ~uy .
y
~ r = E
E ~ i = A~uz , B ~r = B
c c

6.a) La loi dOhm secrit ~ = E ~ o`


u ~ est le courant volumique et la conductivite. Un conducteur
parfait correspond ` a la limite . On doit avoir E ~ = 0 sinon la puissance par unite de volume
dPc ~ = E 2 , serait infinie, ce qui est absurde. Les equations de Maxwell
dissipee par effet Joule, = ~ E
d
restent valables dans un metal. Lequation (11) donne i B ~ =
~ E ~ = 0 et B
~ =0.

6.b) Les relations de passage en x = 0 secrivent :

~
~ux B(0+, t) = 0~ (13)
~
~ux E(0+, t) = (14)
0
~
~ux E(0+, t) = 0 (15)
~
~ux B(0+, t) = 0. (16)

On peut aussi les ecrire de facon equivalente

~
E(0+, t) = ~ux (17)
0
~
B(0+, t) = 0~ ~ux . (18)

~
6.c) On a E(0+, t) = E~ i (0+, t) + E
~ r (0+, t) = 0 et B(0+,
~ ~ r (0+, t) = 2A cos(t)~uy . Les
~ i (0+, t) + B
t) = B
c
equations (1316) sont verifiees avec
2A 2A
= 0 et ~ = cos(t) ~uz . Lamplitude de ~ (t) est = 0,53 A m1 .
0 c 0 c

C. Induction

1.a) Loi de linduction (ou loi de Faraday)


Lorsque le flux du champ magnetique ` a travers un circuit varie au cours du temps, il apparat dans le
d
circuit une force electromotrice induite e = .
dt

b) Loi de Lenz
185

La force electromotrice induite tend a` produire un courant de sens tel que le flux quil envoie `a travers
le circuit soppose `
a la variation du flux qui lui donne naissance.
Cette loi explique le signe dans la loi de linduction.

2.a) Quand on ferme le circuit p, la bobine Ap cree un champ magnetique dans la bobine As . Choisissons
lorientation du circuit s de sorte que le flux du champ magnetique `a travers la bobine As soit positif.
Ainsi commence ` a crotre `
a partir de 0 : dapr`es la loi de linduction, il apparat dans le circuit s une
d
force electromotrice induite e = < 0 et un courant Is = e/R < 0. Le sens de la fl`eche indique le
dt
sens negatif du circuit s.

b) Dans letat stationnaire (courants constants dans les circuits), est constant et, dapr`es la loi de
linduction, la force electromotrice e induite dans le circuit s est nulle. Le courant Is = e/R est alors
egalement nul.

c) Quand on ouvre le circuit p (on l`eve linterrupteur K brusquement), le flux decrot brusquement
d
et tend vers 0. Il apparat dans le circuit s une force electromotrice induite e = > 0 et un courant
dt
Is = e/R > 0. Le courant circule dans le circuit s dans le sens oppose `a la fl`eche. Ce courant ne dure quun
bref instant apr`es louverture du circuit : la pile ne fournit plus denergie et lenergie electromagnetique
est rapidement dissipee en chaleur dans la resistance.

D. Appareil photographique

1) Soit I leclairement (il est egal au module Z


deZ la moyenne temporelle du vecteur de Poynting) de londe
D2 I f 2 I
plane incidente. La puissance recue est P = IdS = = .
diaphragme 4 4N 2
Les puissances sont inversement proportionnelles `a N 2 .
Pour que leffet (ou lenergie) soit le meme, le temps dexposition doit etre proportionnel `a N 2 . Remarquer
que la suite N 2 est une suite geometrique de raison 2 :

N 4 5,6 8 11 16 22
N2 16 32 64 128 256 512

f
2) Le rayon du premier anneau noir de la figure de diffraction est = 1,22 = 1,22N .
D
Le pouvoir separateur theorique est donne par le crit`ere de Rayleigh : on distingue deux images ponc-

tuelles si elles sont distante de au moins. Le pouvoir separateur theorique est donc th = . On converti
f
180 60
les radians en minutes darc par 1 rad = = 3438.

N 4 5,6 8 11 16 22
(m) 2,7 3,8 5,4 7,6 10,7 15,1
th 0,184 0,26 0,37 0,52 0,74 1,04

10 m
3) Le pouvoir separateur de la pellicule est p = = 0,69 . Le pouvoir separateur de lensemble
f
objectif + pellicule est e = max(p , th ). Pour N 11, cest la pellicule qui limite le pouvoir separateur
186 16. EXAMEN (2E SESSION) 2005-2006

et e = p = 0,69 . Pour N = 16 et N = 22, cest la diffraction de louverture qui limite le pouvoir


separateur et e = th .

N 4 5,6 8 11 16 22
e 0,69 0,69 0,69 0,69 0,74 1,04

S-ar putea să vă placă și